You are on page 1of 130

WWW.CLATPOINT.

COM

INDIA’S ONE OF THE FASTEST GROWING CLAT & OTHER


LAW ENTRANCE PREP INSTITUTE

THE REVOLUTION
Legal
Reasoning CLAT UG
CLAT POINT

ABOUT US

CLAT POINT IS A LAW ENTRANCE EXAMINATION PREPARATION INSTITUTE. WE DO NOT CONFIDE OUR SERVICES
ONLY IN THE NAME OF A COACHING INSTITUTE BUT OUR AIM IS TO HELP THE YOUTH WHO JOIN US TO ACHIEVE A
MOMENTUM THAT CAN HELP THEM EMBARK ON A JOURNEY IN THE FIELD OF LAW AND LEGAL KNOWLEDGE. CLAT
POINT IS AN INITIATIVE AND THE CULMINATION OF EFFORTS PUT IN BY IT’S FOUNDER MANVENDRA PRATAP SINGH
WHO HAS AN EXPERIENCE IN TEACHING IN FIELDS AS VAST AS UPSC AND CLAT. HE IS AN NLU DROPOUT AND
ALUMINI FROM JAMIA MILLIA ISLAMIA UNIVERSITY DELHI. WITH AN ONLINE VIEWERSHIP OF NOT LESS THAN A
MILLION HE IS A PROMINENT FACE IN THE TEACHING INDUSTRY. CLAT POINT AIMS AT PROVIDING EFFICIENT
SERVICES IN THE EXAMINATIONS OF CLAT UG , CLAT PG, NLSAT & JUDICIARY.

Legal Reasoning Practice Sheet 1


CLAT POINT

Legal Reasoning
Passage 1

According to Section 10 of the Indian Contract Act of 1872 (ICA), “All agreements are contracts if they are made by the
free consent of parties…” This implies that free consent is a sine qua non for an agreement to become a contract.
Further, according to Section 14 of the ICA, a ‘free consent’ is when it is not caused by either by coercion, or undue
influence, or fraud, or misrepresentation, or mistake. So, any agreement which is entered under coercion
or misrepresentation or fraud is voidable at the option of the party whose consent was so caused.

Coercion, according to section 15 of ICA, is the committing, or threatening to commit, any act forbidden by the Indian
Penal Code or the unlawful detaining, or threatening to detain, any property, to the prejudice of any person whatever,
with the intention of causing any person to enter into an agreement. What it means is that making a person to do
something under the influence of any threat or force.

Contract under ‘undue influence’ as per Section 16 of the ICA, is said to have been induced when the relationship
between the parties to the contract is such that one party has the ability to control the other's will, and that party has
used that power to gain an unfair advantage over the other. For instance – Relation shared between an employee of
any company and his boss. Here the boss has the ability to control the employee’s will.

Another factor that vitiates the free consent is the presence of fraud. ‘Fraud’ means and includes any of the following
acts committed by a party to a contract, or with his connivance, or by his agent, with intent to deceive another party
thereto of his agent, or to induce him to enter into the contract - (1) the suggestion, as a fact, of that which is not
true, by one who does not believe it to be true; (2) the active concealment of a fact by one having knowledge or belief
of the fact; (3) a promise made without any intention of performing it; (4) any other act fitted to deceive; (5) any such
act or omission as the law specially declares to be fraudulent. Fraud refers to an act done by a party which actively
deceives the other party in order to gain the consent of the said party. However, mere silence as to facts likely to
affect the willingness of a person to enter into a contract is not fraud, unless the circumstances of the case are such
that, regard being had to them, it is the duty of the person keeping silence to speak, or unless his silence, is, in
itself, equivalent to speech.

1. Sagar, with a gun pointed at Samar, coerces him into selling his beloved bike for a mere 1 Rupee. Samar, desperate
to sell his bike for some time, succumbs to the pressure and agrees to the deal. Applying the principle laid out in the
passage, which option would be the most suitable?
(a) It cannot be considered an agreement since the consent of Samar was obtained through coercion.
b) It is a valid contract since Samar wanted to sell his bike and he ultimately agreed to the terms offered by Sagar.
(c) While it is an agreement, it falls under the category of a voidable contract since the consent of Samar was not given
freely due to the threat of violence.
(d) This agreement qualifies as a voidable contract, but not because of the price of the bike. Rather, it is due to the
fact that Samar's consent was obtained under duress.

2. Suppose in the scenario presented in the earlier question, Sagar threatened Samar with a gun to sell him his bike at
the cost of 1 Rupee. However, unknown to Samar, the gun was fake. In this situation, which of the following options
is the most appropriate one, applying the principles outlined in the passage provided?
(a) The transaction constitutes a valid contract, as Samar agreed to sell his bike.
(b) The contract is voidable at the option of Samar, as he did not have the knowledge that the gun was fake, and
therefore, his consent was not free.
(c) The transaction is an agreement, but it represents a voidable contract because the bike was sold at an unreasonably
low price of 1 Rupee.
(d) The transaction constitutes a contract, as Samar had intended to sell his bike.

Legal Reasoning Practice Sheet 2


CLAT POINT

3. Dr. Amitabh is a highly accomplished neurosurgeon who has dedicated his life to finding a cure for brain cancer.
After years of research, he has developed a groundbreaking brain-cell imaging surgery that could potentially save
countless lives. However, he is still waiting for approval to begin practical applications of his theory. One year ago, his
patient, Bacchan, was diagnosed with 2nd stage brain cancer, and his condition has rapidly deteriorated. In an attempt
to test his theory, Dr. Amitabh offers Bacchan the chance to undergo his brain-cell imaging surgery. Bacchan, desperate
for a cure, gives his consent. Which of the following options is the most appropriate?
(a) It is a voidable contract covered under the head of undue influence. As Amitabh was in a position to control the
will of Bacchan, his consent was not entirely free.
(b) Dr. Amitabh is acting nobly and ethically in offering Bacchan the chance to undergo the surgery. His theory has
already received the required approval, and he is using his influence for a good cause.
(c) Bacchan gave his consent freely and without coercion, so this cannot be covered under undue influence.
(d) This is a case of undue influence, not due to the relationship between the parties, but due to Bacchan's diagnosis
of brain cancer, which put him in a vulnerable position.

4. Aman, a seasoned horse trader, sold a horse to Amar without disclosing that the horse had a history of seizures and
erratic behaviour. Amar, a novice horse owner, purchased the horse without inspecting it thoroughly, assuming that
Aman had sold him a healthy horse. After two weeks of owning the horse, Amar realized that the horse was indeed
suffering from a mental ailment, which Aman had intentionally concealed. In the given situation, what is the most
appropriate option?
(a) Aman can be held liable for fraud because he had knowledge of the unsound state of the horse and deliberately
concealed it.
(b) Aman cannot be held liable for fraud because he did not make any positive statements about the horse's mental
state.
(c) Aman cannot be held liable because the buyer should have conducted a thorough inspection of the horse before
making the purchase.
(d) Aman can be held liable for fraud only if it can be proved that he intentionally deceived Amar to make a profit.

5. Rohit, a wealthy businessman, wants to buy a rare piece of art from his friend Rahul, who is a struggling artist. Rahul,
aware of Rohit's interest in the artwork and his financial clout, pressures him to buy the piece of art at an inflated
price. Rohit, afraid of damaging his friendship with Rahul and losing out on the artwork, reluctantly agrees to pay the
inflated price. Which of the following options best describes the situation, based on the principles of the Indian
Contract Act?
(a) This is a case of coercion, as Rahul has used his position of power to force Rohit into buying the artwork at an
inflated price.
(b) This is a case of undue influence, as Rahul has exerted undue pressure on Rohit to buy the artwork.
(c) This is a case of fraud, as Rahul has knowingly and intentionally misled Rohit into paying an inflated price for the
artwork.
(d) This is not a case of any of the above principles, as Rohit agreed to the inflated price of his own free will.

Passage 2
A contract the consent to which is induced by misrepresentation is voidable at the option of the deceived party.
Misrepresentation means misstatement of a fact material to the contract. Misrepresentation is defined in Section 18;
S. 18. "Misrepresentation" defined.—"Misrepresentation" means and includes— (1) the positive assertion, in a
manner not warranted by the information of the person making it, of that which is not true, though he believes it to
be true; (2) any breach of duty which, without an intent to deceive, gains an advantage to the person committing it,
or anyone claiming under him, by misleading another to his prejudice, or to the prejudice of any one claiming under
him; (3) causing, however innocently, a party to an agreement, to make a mistake as to the substance of the thing
which is the subject of the agreement.

Legal Reasoning Practice Sheet 3


CLAT POINT

When a person positively asserts that a fact is true when his information does not warrant it to be so, though he
believes it to be true, this is misrepresentation. A statement is said to be warranted by the information of the person
making it when he receives the information from a trustworthy source. It should not be a mere hearsay.

Where a representation acquires the status of being a term of the contract, and it turns out to be untrue, the
disadvantaged party may, not only avoid the contract but also sue for damages for breach. Where in the course of
negotiations for the sale of lamb, the seller stated that the whole of the lot was fully serviced, whereas this was not
so, the buyer was allowed damages for the breach of the warranty. In another case' the plaintiff was orally assured
that his containers would be carried under the deck, but by mistake they were loaded on deck and were lost. The
defendant was not allowed to claim the protection of a clause in the contract limiting his liability only to wilful
misconduct. The oral assurance had overridden the written clauses.

Any breach of duty which brings an advantage to the person committing it by misleading the other to his prejudice is
a misrepresentation. "This clause is probably intended to meet all those cases which are called in the court of equity—
cases of 'constructive fraud in which there is no intention to deceive, but where the circumstances are such as to make
the party who derives a benefit from the transaction equally answerable in effect as if he had been actuated by motives
of fraud or deceit".

6. A seller, while negotiating the sale of a laptop, falsely claims that the laptop has never been repaired or damaged,
when in reality it had several issues and had been repaired multiple times. The buyer relies on this representation and
agrees to purchase the laptop. Later, the buyer discovers the truth and wishes to void the contract. Which of the
following statements is true regarding the situation described above?
(a) The contract is voidable at the option of the seller since the misrepresentation was not intentional.
(b) The buyer cannot void the contract since the misrepresentation was not intentional.
(c) The contract is voidable at the option of the buyer since the misrepresentation was material to the contract.
(d) The buyer can only sue for damages but cannot void the contract since the misrepresentation was not intentional.

7. In a recent court case, the plaintiff alleged that the defendant made a misrepresentation of facts which induced him
to enter into a contract. According to the plaintiff, the defendant had told him that a certain property was free from
any legal encumbrances. However, it turned out that the property was subject to a mortgage. The defendant argued
that he had obtained this information from a reliable source, and therefore, he had not made any misrepresentation.
Which of the following is true in this case?
(a) The defendant's statement is a clear case of misrepresentation, as he made a positive assertion which was not
warranted by his information.
(b) The defendant cannot be held liable for misrepresentation, as he had relied on information from a reliable source.
(c) The plaintiff cannot claim damages for misrepresentation, as he should have conducted his own due diligence
before entering into the contract.
(d) Both a) and b) are plausible arguments, but the court must determine the credibility of the defendant's source of
information before making a decision.
The correct answer is Option C: B relied only on hearsay information, he was not warranted to assert such information.
Hence this is not misrepresentation.

8. A purchased land from B for constructing a duplex. B, while selling the land, represented that there was no difficulty
in using the land for the intended purpose. However, A later found out that a permission to build the complex was
denied unless a sewage system costing $3000 was installed. A claims that B should have disclosed this fact before
selling the land. Which of the following options is correct?
(a) The contract is voidable at the option of the buyer due to fraudulent misrepresentation by B.
(b) The contract is voidable at the option of the seller due to mutual mistake of fact.
(c) The contract is voidable at the option of the buyer due to innocent misrepresentation by B.
(d) The contract is not voidable as the buyer should have done their due diligence before purchasing the land.

9. Which of the following scenarios amounts to misrepresentation according to this part of the passage ".—
"Misrepresentation" means and includes— (1) the positive assertion, in a manner not warranted by the information

Legal Reasoning Practice Sheet 4


CLAT POINT

of the person making it, of that which is not true, though he believes it to be true; (2) any breach of duty which, without
an intent to deceive, gains an advantage to the person committing it, or anyone claiming under him, by misleading
another to his prejudice, or to the prejudice of any one claiming under him; (3) causing, however innocently, a party
to an agreement, to make a mistake as to the substance of the thing which is the subject of the agreement."
(a) The seller of a car tells the buyer that the car has been driven for only 10,000 miles when in reality it has been
driven for 50,000 miles.
(b) A person tells a buyer that they will be fired if they do not sign a contract with them.
(c) A person tells a buyer that they will release their confidential information if they do not sign a contract with them.
(d) A person sells a property to a buyer, and later files a false report claiming that the buyer had caused damage to the
property in order to extort additional compensation.

10. According to Section 18 of the Indian Contract Act, certain essential requisites must be fulfilled to construe an act
as misrepresentation. Which of the following scenarios does not fulfill one of these requisites?
(a) A person asserts a fact without having any information about it and does not believe that the information is true.
(b) A person asserts a fact which he believes to be true but which is actually false, based on information received from
a source that is generally considered unreliable.
(c) A person asserts a fact that is untrue and does not believe it to be true, but the information is received from a
credible source.
(d) A person asserts a fact which he believes to be true and which is actually true, based on information received from
a source that is trustworthy.

Passage 3
“Indemnity" in English law means a promise to save a person harm less from the consequences of an act. The promise
may be express or it may be implied from the circumstances of the case. The English definition of indemnity is wide
enough to include a promise of indemnity against loss arising from any cause whatsoever, e.g., loss caused by fire or
by some other accident. Indeed, every contract of insurance, other than life assurance, is a contract of indemnity. But
the definition of "indemnity" in Section 124 of the Indian Contract Act is somewhat narrower. It is like this: S. 124.
"Contract of Indemnity" defined.—A contract by which one party promises to save the other from loss caused to him
by the conduct of the promisor himself, or by the conduct of any other person, is called a "contract of indemnity".
Illustration A contracts to indemnify B against the consequences of any proceedings which C may take against him
respect of a certain sum of 200 rupees. This is a contract of indemnity. The only illustration appended to the section
says that if a person promises to save another from the consequences of a proceeding which may be commenced
against him it is a contract of indemnity. The person who gives the indemnity is called the "indemnifier" and the person
for whose protection it is given is called the "indemnity-holder" or "indemnified".

Thus, the scope of "indemnity" is by the very process of definition restricted to cases where there is a promise to
indemnify against loss, caused: [a) by the promisor himself, or {b) by any other person. The definition excludes from
its purview cases of loss arising from accidents like fire or perils of the sea. Loss must be caused by some human
agency. Secondly, situations like those in Adamson v Jarvis where cattle were sold under the instruction of a wrongful
owner, are also outside the scope of this definition. Such cases and the case of a loss arising from an act done at the
request of the promisor are covered by Section 223 of the Act which provides for indemnity between principal and
agent. An indemnity bond which permits an employee to leave the employment earlier than the minimum agreed
period only at the cost of the forfeiture of his bond money is valid provided both the period of restriction and the bond
money are reasonable. Only that part of the bond money can be retained which is necessary to indemnify the
employer-for his loss.

11. The plaintiff, an auctioneer, sold certain horses on the instruction of the defendant. It subsequently turned out
that the livestock did not belong to the defendant, but to another person, who made the auctioneer liable and the
auctioneer in his turn sued the defendant for indemnity for the loss he had thus suffered by acting on the defendant's
directions. Which of the following is the most suitable option?
(a) The defendant would have to indemnify the plaintiff, the plaintiff had assumed that if what he did was wrong, he
would be indemnified by the defendant.
Legal Reasoning Practice Sheet 5
CLAT POINT

(b) The defendant would not have to indemnify the plaintiff because he was not the owner of the horses.
(c) The defendant would not have to indemnify because the plaintiff should have checked whether he was the true
owner of the horses.
(d) None of the above.

12. The plaintiffs were in possession of certain trucks which were claimed both by the defendants and one K.P. Co. The
defendants demanded delivery and the plaintiffs asked for an indemnity bond, but received no reply. Even so they
delivered the trucks to the defendants. K.P. Co. successfully sued the plaintiffs for conversion of their goods. Do the
defendants have to indemnify the plaintiffs?
(a) Defendants would not have to indemnify the plaintiffs because there was no promise for indemnity by them.
(b) Defendants would have to indemnify the plaintiffs because by demanding the indemnity bond, they made it clear
impliedly that there was no intention of delivery unless there was indemnity.
(c) Defendants would have to indemnify because the plaintiffs had suffered a loss due to conduct of the defendants.
(d) Both b and c.

13. Alpha Pvt. Ltd. hires Beta Pvt. Ltd. to transport a consignment of goods from one city to another. Beta Pvt. Ltd.
agrees to indemnify Alpha Pvt. Ltd. against any loss or damage caused to the goods during transit. However, the
consignment gets stolen due to the negligence of Alpha Pvt. Ltd. employees who did not properly secure the goods.
What is the legal position of Beta Pvt. Ltd. in this situation?
(a) Beta Pvt. Ltd. is liable to indemnify Alpha Pvt. Ltd. as per the contract.
(b) Beta Pvt. Ltd. is not liable to indemnify Alpha Pvt. Ltd. as the loss was caused due to the negligence of Alpha Pvt.
Ltd.
(c) This is not a contract of indemnity.
(d) Beta Pvt. Ltd. is liable to indemnify Alpha Pvt. Ltd. only if it can be proven that the theft was caused due to the
negligence of Beta Pvt. Ltd.

14. A contract was made between X, the owner of a warehouse, and Y, a logistics company, for storing Y's goods in the
warehouse. The contract included a clause which stated that X would be indemnified by Y for any loss caused to X's
warehouse due to fire or any other accidents. One day, a fire broke out in the warehouse, causing significant damage
to the property. X sued Y for damages. Which of the following options is correct?
(a) X cannot recover damages from Y because the loss was caused by an accident and not by the conduct of Y.
(b) X can recover damages from Y because the contract included a clause for indemnification against loss caused by
fire or other accidents.
(c) X cannot recover damages from Y because the clause for indemnification is too broad and includes losses not caused
by Y.
(d) X can recover damages from Y only if Y was directly responsible for causing the fire.

15. ABC Pvt. Ltd. entered into a contract with XYZ Pvt. Ltd. for the supply of goods worth Rs. 5,00,000. As per the
contract, ABC Pvt. Ltd. agreed to indemnify XYZ Pvt. Ltd. against any loss arising from the supply of goods. However,
during the transportation of goods, the truck carrying the goods met with an accident and the goods worth Rs. 2,00,000
were damaged. XYZ Pvt. Ltd. claimed compensation from ABC Pvt. Ltd. for the loss suffered. Which of the following
options is correct?
(a) ABC Pvt. Ltd. is not liable to indemnify XYZ Pvt. Ltd. since the loss was caused due to an accident.
(b) ABC Pvt. Ltd. is liable to indemnify XYZ Pvt. Ltd. only if the contract specifies that loss caused by accidents is covered.
(c) ABC Pvt. Ltd. Is not liable to indemnify XYZ Pvt. Ltd. since the loss was caused during the transportation of goods.
(d) ABC Pvt. Ltd. is liable to indemnify XYZ Pvt. Ltd. only if it is proven that the loss was caused due to the conduct of
the promisor or any other person.

Passage 4
Giving the meaning of a void agreement, the Indian Contract Act says in Section 2{g): An agreement not enforceable
by law is said to be void. The following types of agreement are declared to be void: (1) Agreements of which
consideration and objects are unlawful in part, [S. 24] (2) Agreements without consideration, [S. 25] (3) Agreements
in restraint of marriage, [S. 26] (4) Agreements in restraint of trade, [S. 27] (5) Agreements in restraint of legal
proceedings, [S. 28] (6) Unmeaning agreements, [S. 29] (7) Wagering agreements, [S. 30] and (8) Agreements to do
Legal Reasoning Practice Sheet 6
CLAT POINT

impossible acts. [S. 56] Agreements in which a part of consideration or object is unlawful S.24 of Indian Contract Act,
1872 states the definition of Agreements which are void, if considerations and objects unlawful in part.—"If any part
of a single consideration for one or more objects, or any one or any part of any one of several considerations for a
single object, is unlawful, the agreement is void.”

The working philosophy behind the provision was stated by Willes J in these words: Where you cannot sever the illegal
from the legal part of a covenant the contract is altogether void, but where you can sever them whether the illegality
be created by statute or by common law, you may reject the bad part and retain the good. The section comes into
play when a part of the consideration for an object or more than one objects of an agreement is unlawful. The whole
of the agreement would be void unless unlawful portion can be severed without damaging the lawful portion. For
example- A promises to pay a fixed sum of money on monthly basis to a married woman for living in adultery with the
promisor, which is unlawful, and for keeping his house, which is lawful, the whole agreement was held to be void
because it was impossible to apportion the single lump sum between the lawful object and the unlawful one Where
the legal part of an agreement is severable from the illegal, the former would be enforced.

16. A, a renowned scientist, promised to work for B, a pharmaceutical company, to research and develop a new
medicine for cancer treatment, which is legal, and also promised to engage in the illicit practice of testing the medicine
on humans without their informed consent. In return, B promised to pay A, a salary of 10 crore rupees per year. Choose
the correct option:
(a) The agreement is voidable at the option of A because the legal and illegal parts cannot be separated.
(b) The agreement is valid because A is a renowned scientist and can be trusted to conduct the testing ethically.
(c) The agreement is void because it involves illegal activities, which cannot be separated from the legal ones.
(d) The agreement is voidable at the option of B because B can choose to exclude the illegal part of the agreement and
pay A only for the legal part of the work.

17. A licence was granted to a person (Mr X) for sale of opium and ganja with this restriction that he would not take
any partner in the ganja business without the permission of the collector. It is to be noted that sale of opium is legal
but not of ganja. Without such permission he admitted a partner into the whole business on receiving from him a fixed
sum as his share of capital. Differences arose between them. The new entrant filed a case for dissolution and refund
of his money. Can he succeed?
(a) He will succeed since the object of his contract is not illegal.
(b) He will not succeed because the contract between the new entrant and Mr. X is void due to illegal object
(c) He will not succeed since illegal object make the contract void.
(d) Both b and c.

18. Under which circumstances would an agreement in restraint of legal proceedings be considered void under the
Indian Contract Act?
(a) If the restraint is partial and does not impede the ability to take legal action in certain cases.
(b) If the agreement is entered into with the purpose of preventing a party from exercising their legal rights or
remedies.
(c) If the agreement contains a valid and lawful object but is found to be illegal due to the restraint of legal proceedings.
(d) If the restraint of legal proceedings is only temporary and does not exceed a period of one year.

19. Which one of the following agreements is considered void under the Indian Contract Act?
(a) A promises to sell his car to B for Rs. 5 lakh, subject to B's approval of the car's condition after a test drive.
(b) A agrees to pay B Rs. 10,000 if B refrains from filing a lawsuit against A for a car accident.
(c) A promises to gift B a diamond necklace for her engagement.
(d) A agrees to deliver goods to B at a price that is less than the cost incurred by A to acquire them.

20. Assuming all other elements of a contract are valid, which of the following scenarios would render an agreement
void under Section 27 of the Indian Contract Act?
(a) A non-compete agreement between two software companies in the same city for a duration of two years
(b) An agreement between two artists not to exhibit their work in a particular gallery for a period of one year
Legal Reasoning Practice Sheet 7
CLAT POINT

(c) A contract between a restaurant and a chef to work exclusively for the restaurant for a period of six months
(d) An agreement between a doctor and a patient not to disclose the patient's medical information to anyone outside
of the clinic

Passage 5
In English law a person of unsound mind is competent to contract, although he may avoid his contract if he satisfies
the court that he was incapable of understanding the contract and the other party knew it. The contract is voidable at
his option. It becomes binding on him only if he affirms it." In this case Lord Esher said that a lunatic (now such a
person is known as mentally disordered) can only set aside a contract entered into with a person of sound mind in the
following circumstances: "When a person enters into a contract and afterwards alleges that he was so insane at the
time that he did not know what he was doing, and proves the allegation, the contract is as binding on him in every
respect. whether it is executory or executed, as if he had been sane when he made it, unless he can prove further that
the person with whom he contracted knew him to be so insane as not to be capable of understanding what it was
about." The position of a drunken person is also the same. If he makes a contract while drunk, he may, when sober,
elect to avoid the contract or to affirm it.

In India, on the other hand, the agreement of a person of unsound mind is, like that of minor, absolutely void."
According to Section 12 of the Indian Contract Act, 1872 "a person is said to be of sound mind for the purpose of
making a contract if, at the time when he makes it, he is capable of understanding it and of forming a rational judgment
as to its effects upon his interest." However, a person who is usually of unsound mind may make a contract when he
is of sound mind. But a person who is usually of sound mind may not make a contract when he is of unsound mind. S.
12. What is a sound mind for the purposes of contracting.—A person is said to be of sound mind for the purpose of
making a contract If, at the time when he makes it, he is capable of understanding it and of forming a rational judgment
as to its effect upon his interests. A person who Is usually of unsound mind, but occasionally of sound mind, may make
a contract when he is of sound mind. A person who is usually of sound mind, but occasionally of unsound mind, may
not make a contract when he Is of unsound mind. Illustration (b) appended to Section 12 shows that a drunken person
is in the same category as a person of unsound mind.

21. A mortgagee Mr Johnson sought a decree directing repayment and foreclosure in default of such repayment. There
is evidence that at the time the mortgage was executed the mortgagor, Mr Tull was a lunatic. This case was filed in a
court in England. Choose the most suitable option.
(a) The contract is voidable at the option of Mr Johnson only if Mr Tull was not completely incapable of understanding
the contract at the time it was signed.
(b) The contract is valid if Mr Johnson can prove that Mr Tull understood the contract at the time of signing.
(c) The contract is void if Mr Tull's incapacity was obvious and Mr Johnson should have been aware of it at the time of
signing.
(d) The contract is voidable at the option of Mr Tull only if he can prove that Mr Johnson took advantage of his
incapacity at the time of signing.

22. A sale deed of property was executed between two parties, one of whom was suffering from alcoholic psychosis
at the time of execution, which was proved by a medical certificate. The sale was for a very valuable land but was sold
for a paltry amount. A suit was filed in the Bombay High Court in India for setting aside the sale deed. Which of the
following is the most accurate statement regarding the validity of the sale?
(a) The sale can be set aside because it was a very valuable land which was sold for paltry amount.
(b) The sale can be set aside only if it can be proven that the other party knew about the transferor's condition.
(c) The sale can be set aside only if the transferor had no capacity to understand the terms of the sale.
(d) The sale can be set aside because it is void ab initio.

23. Mr. Jacobs, a person suffering from schizophrenia, enters into a contract with Mr. Alex, a businessman in Delhi,
India. During the execution of the contract, Mr. Jacobs exhibits some symptoms of his condition, but later confirms
the contract during a period of lucidity. Which of the following statements is true?
(a) The contract is void because Mr. Jacobs has a history of schizophrenia.
(b) The contract is valid because Mr. Jacobs confirmed it during a period of lucidity.

Legal Reasoning Practice Sheet 8


CLAT POINT

(c) The contract is voidable at the option of Mr. Jacobs because he exhibited symptoms of his condition during its
execution.
(d) The contract is voidable at the option of Mr. Alex because he was aware of Mr. Jacobs' condition.

24. John suffers from a mental disorder but is generally of sound mind. He enters into a contract with Jane when he is
of sound mind. Later, John has a relapse and claims that he was not capable of understanding the contract at the time
of its execution. Can John avoid the contract under English law?
(a) Yes, John can avoid the contract as he has a mental disorder.
(b) No, John cannot avoid the contract as he was of sound mind when he entered into the contract.
(c) Yes, John can avoid the contract if he can prove that Jane knew about his mental disorder.
(d) None of the above.

25. Mr. Raghav, a person of unsound mind, entered into a contract with Mr. Arun to smuggle drugs into the country.
Later, Mr. Raghav sued Mr. Arun for breach of contract claiming that Mr. Arun failed to deliver the drugs. What will be
the outcome of the case under the Indian Contract Act, 1872?
(a) Mr. Raghav can enforce the contract as he was competent to contract at the time of entering into it.
(b) The contract is void ab initio as it is illegal and Mr. Raghav's unsound mind cannot validate it.
(c) The contract is voidable at Mr. Raghav's option as he was of unsound mind.
(d) Mr. Arun will have to deliver the drugs or pay damages to Mr. Raghav as per the contract.

Passage 6
When a tort is committed by several persons, all the persons involved in it become joint tortfeasor. In addition to this,
all persons will be responsible for the same tort and will be deemed to be joint wrongdoers in the eyes of law.

Now, at this point, it becomes important for one to understand that in order to establish the wrong committed by
joint tortfeasors, there must be some connection between the act of one alleged tortfeasor to that of the other.

The liability of joint tortfeasors is joint and several. The plaintiff has a choice to sue anyone of them, some of them or
all of them, in an action. Each one of them can be made to pay the full amount of compensation. Thus, for the wrong
done by the agent, both the principal and the agent are jointly and severally liable. Even though the actual wrongdoer
is the agent, if the plaintiff so elects, he may sue the principal for the whole of the damage. As against the aggrieved
party, the principal cannot take the defence that the actual wrongdoer was the agent, although after making good the
loss, the principal may hold the agent responsible to the extent of his (agent's) fault. Similarly, for the wrongful act
done by the servant, the master is liable along with the servant as a joint tortfeasor and for the wrongful act of a
partner, the firm is liable therefor to the same extent as the guilty partner. Where the plaintiff elects to bring an action
against all of them jointly, judgment obtained against all of them may be executed in full against any of them. In the
event of liability of joint tortfeasors, it is no concern of the tribunal to apportion the damages between them.

26. Mr. X and Mr. Y are joint tortfeasors. They both commit a tortious act against Mr. Z, causing him damage. If Mr. Z
decides to sue both of them, what are the available options to him for recovering compensation, and what are the
possible outcomes?
(a) Mr. Z can sue only one of the joint tortfeasors, and if he wins, that tortfeasor will have to pay the full amount of
compensation to Mr. Z.
(b) Mr. Z can sue both the joint tortfeasors separately, and if he wins both cases, he can recover compensation from
both of them.
(c) Mr. Z can sue both the joint tortfeasors jointly in one action, and if he wins, he can recover the full amount of
compensation from either of them or both of them jointly.
(d) Mr. Z cannot sue both the joint tortfeasors jointly in one action, and he will have to file separate actions against
each of them, even though they both committed the same tortious act.
Answer: C) Mr. Z can sue both the joint tortfeasors jointly in one action, and if he wins, he can recover the full amount
of compensation from either of them or both of them jointly.

27. In a scenario where A, wearing an ear plug, was reversing his vehicle for parking it while B, who was on a phone
call, drove his car in a hurry to occupy the same parking space, both A and B ended up driving into an adjacent vehicle
Legal Reasoning Practice Sheet 9
CLAT POINT

causing injury to C. In light of the above situation, which of the following statements would be deemed legally
accurate?
(a) A is completely liable to pay damages to both B and C.
(b) B is completely liable to pay damages to both A and C.
(c) Both A and B share joint liability and are required to pay damages to C.
(d) It is at the discretion of C to determine the proportion of damages to be paid by A and B.

28. A and B conjointly destroyed the machinery which belonged to C, eventually, C brought an action against both and
obtained a claim of 2 lakhs by way of the judgment delivered in his favour. The whole amount was levied on A, who
again sought contribution from B to the extent of loss done by him. Decide
(a) A will succeed as he can hold B liable up to the extent of B’s fault
(b) A will not succeed as both A and B were wrong doer in the respective case.
(c) B does not have to pay as he did not ask A to pay on his behalf.
(d) If B does not pay A, C has to return the amount of B to A.

29. Jogindar, an experienced driver, was driving his truck recklessly, while his friend Surindar, sitting beside him,
continuously urged him to slow down. Meanwhile, Manindar had parked his truck improperly on the road, which any
careful driver would have noticed. Jogindar collided with Manindar's truck, and Surindar suffered injuries. Determine
liability for compensation.
(a) Jogindar alone is liable, as he was driving the truck recklessly.
(b) Manindar alone is liable, as he parked his truck improperly.
(c) Both Jogindar and Manindar are liable for Surindar's injuries, as their actions contributed to the accident.
(d) Surindar cannot claim compensation as he was aware of Jogindar's reckless driving and did not take any action to
prevent the accident.

30. Which of the following scenarios best exemplifies joint and severable liability among joint tortfeasors?
(a) Two friends, Alex and Ben, drive their cars recklessly and crash into each other causing damage to both vehicles.
(b) A and B both throw stones at C’s car, breaking the windshield.
(c) X and Y, two construction companies, together build a faulty bridge that collapses causing harm to Z.
(d) John and Max, two robbers, steal a valuable diamond together and then sell it, splitting the proceeds equally.

Passage 7
Indian law, does not recognise any such exception to the doctrine that agreement without consideration are void. But
Section 25 of the Contract Act lays down a few exceptions. S. 25. An agreement made without consideration is void,
unless.— (1) it is in writing and registered.—It is expressed in writing and registered under the law for the time being
in force for registration of [documents], and is made on account of natural love and affection between parties standing
in a near relation to each other; or unless (2) or is a promise to compensate for something done. —It is a promise to
compensate, wholly or in part, a. person who has already voluntarily done something for the promisor, or something
which the promisor was legally compellable to do; unless (3) or is a promise to pay a debt barred by limitation law.—
It is a promise, made in writing and signed by the person to be charged therewith, or by his agent generally or specially
authorised in that behalf, to pay wholly or in part a debt of which the creditor might have enforced payment but for
the law for the limitation of suits. In any of these cases, such an agreement is a contract.

Under the section 25(2), a promise to compensate wholly or in part, a person who has already voluntarily done
something for the promisor, is enforceable. In other words, a promise to pay for a past voluntary service is binding. It
is necessary to attract this exception that the service should have been rendered voluntarily and also for the promisor.

Lastly, a promise to pay a time-barred debt is enforceable. The promise should be in writing. It should also be signed
by the promisor or "by his agent generally or specially authorised in that behalf. The promise may be to pay the whole
or any part of the debt. The debt must be such "of which the creditor might have enforced payment but for the law
for the limitation of suits". Is it necessary that the promise should be given by the person who was liable for the original
time-barred debt? The Bombay High Court has held that "a promise made by a person who is under no obligation to
pay the debts of another... does not fall within the clause." But in the opinion of the Madras High Court "the words 'by
the person to be charged therewith' in Section 25(3) are wide enough to include the case of a person who agrees to
Legal Reasoning Practice Sheet 10
CLAT POINT

become liable for the payment of a debt due by another and need not be limited to the person who was indebted
from the beginning."

31. Mr. X is the legal heir of a deceased debtor Mr. Y, who had a time-barred debt. During a court proceeding, Mr. X
admitted in writing that he is willing to pay the principal time-barred amount. Can this admission be considered a
promise under section 25(3) of the Indian Contract Act, 1872, even though Mr. X was not personally liable for the debt,
and the admission was not signed by him but by his lawyer who was not specially authorized to do so?
(a) This promise is covered under section 25(3) of the Indian Contract Act, 1872, as Mr. X is the legal heir and successor
of the deceased debtor, and the admission was made in writing during a court proceeding.
(b) This promise is not covered under section 25(3) of the Indian Contract Act, 1872, as Mr. X was not personally liable
for the debt and the admission was not signed by him but by his lawyer who was not specially authorized to do so.
(c) Indian Contract Act, 1872 takes cognizance of time-barred debts only if the debtor was alive at the time the debt
became time-barred, and the promise is made in writing and signed by the debtor or his authorized agent.
(d) None of the above.

32. Mr Tucker promised, in writing to execute a transfer of a car as a payment for the past services which were
rendered by his butler Mr Chris. Is this promise valid, given that there is no consideration?
(a) This is not covered by section 25(2) of the Indian Contract Act, 1872 and is not valid.
(b) This is voidable at the option of Mr Tucker as there is no consideration.
(c) This is a valid promise and covered by section 25(2) as it is for services rendered in the past.
(d) None of the above.

33. Mr Z, a tenant, in a letter to the landlord Mr L, referred to the arrears of rent from 5 years ago and said: "I shall
send by the end of the Vysakh month, a portion of the amount I owe you as an acknowledgement of my obligation."
What kind of promise is this as per the Indian Contract Act, 1872?
(a) This promise is voidable at the option of Mr L since it is not a promise to pay the entire debt.
(b) This promise is voidable at the option of Mr Z since the acknowledgement of obligation is not valid consideration.
(c) This promise is void as it is not a promise to pay the entire debt and the acknowledgement of obligation is not valid
consideration.
(d) This promise is valid as it is a written promise to pay a time-barred debt and is covered under section 25(3) of the
Indian Contract Act, 1872.

34. Mr. X borrowed Rs. 1,00,000 from Mr. Y and executed a promissory note in favour of Mr. Y. Later, when the due
date for repayment came, Mr. X told Mr. Y that he was unable to repay the amount as he had suffered huge losses in
his business. Mr. Y agreed to extend the time for repayment and executed a written agreement to that effect.
However, when the extended due date came, Mr. X again failed to repay the amount. Mr. Y filed a suit for recovery of
the amount based on the promissory note. Which of the following is true?
(a) Mr. X is not liable to repay the amount as the original promissory note became void due to the subsequent written
agreement to extend the time for repayment.
(b) Mr. Y is not entitled to recover the amount as the subsequent written agreement was without consideration and
hence void.
(c) Mr. Y is entitled to recover the amount as the subsequent written agreement is a promise to pay a time-barred
debt, which is enforceable under Section 25(3) of the Indian Contract Act, 1872.
(d) Mr. X is liable to repay the amount as the subsequent written agreement is a new contract that replaces the original
promissory note and Mr. Y can enforce it.

35. According to the passage, which of the following statements are true regarding the enforceability of agreements
made without consideration in Indian Contract Law?
(a) An agreement without consideration is void unless it is in writing and registered under the law for registration of
documents and made on account of natural love and affection between parties standing in a near relation to each
other.
(b) A promise to compensate for something done is enforceable, wholly or in part, if the promisor has already
voluntarily done something for the promisee or something which the promisor was legally compellable to do.

Legal Reasoning Practice Sheet 11


CLAT POINT

(c) A promise to pay a debt barred by limitation law is enforceable if it is made in writing, signed by the promisor or by
his agent authorized for that purpose, and the debt is one which the creditor might have enforced payment for but
for the law of limitation of suits.
(d) All of the above are true.

Passage 8
Marriage is a sacred institution; it is the very foundation of a stable family and civilised society. There are, however,
certain prerequisites and conditions for a valid marriage. All personal laws lay down some conditions which need to
be complied with to enter into or solemnise a legal marriage; but before the conditions the question is, does everyone
have an absolute right to marry?

The right to marry is a component of the right to life under Art. 21 of the Constitution of India which says, ‘No person
shall be deprived of his life or personal liberty except according to procedure established by law.’ This right has been
recognised even under the Universal Declaration of Human Rights, 1948. Article 16 of the same states: 1. Men and
women of full age, without any limitation due to race, nationality or religion, have the right to marry and to found a
family. They are entitled to equal rights as to marriage, during marriage and at its dissolution 2. Marriage shall be
entered into only with the free and full consent of the intending spouses. 3. The family is the natural and fundamental
group unit of society and is entitled to protection by society and the state.

Under the Hindu Marriage Act, 1955, the following are the conditions for a marriage -
S. 5. Conditions for a Hindu marriage.— A marriage may be solemnized between any two Hindus, if the following
conditions are fulfilled, namely: (i) neither party has a spouse living at the time of the marriage; (ii)at the time of the
marriage, neither party— (a) is incapable of giving a valid consent to it in consequence of unsoundness of mind; or (b)
though capable of giving a valid consent, has been suffering from mental disorder of such a kind or to such an extent
as to be unfit for marriage and the procreation of children; or (c) has been subject to recurrent attacks of insanity (iii)
the bridegroom has completed the age of [twenty-one years] and the bride, the age of [eighteen years] at the time of
the marriage; (iv) the parties are not within the degrees of prohibited relationship unless the custom or usage
governing each of them permits of a marriage between the two; (v) the parties are not sapindas of each other, unless
the custom or usage governing each of them permits of marriage between the two.

36. Mr X, a Christian converted to become Hindu on 27th Dec 2021. He had married a Ms Y, a Hindu on 24th Dec 2021.
Is this marriage valid under section 5 of the Hindu Marriage Act, 1955?
(a) The marriage is valid he had converted to become Hindu.
(b) The marriage is not valid because he had converted after the marriage had taken place.
(c) The marriage is not valid because a Christian cannot marry a Hindu under the Hindu Marriage Act, 1955.
(d) Both b and c

37. Rahul, a 20-year-old Hindu man, falls in love with Pooja, a 19-year-old Hindu woman. However, Pooja's parents are
against their marriage because they belong to the same gotra, which is considered a prohibited relationship under
Hindu customs. Rahul and Pooja decide to elope and get married in a court. However, during the registration process,
the registrar discovers that Rahul's father had remarried without legally obtaining a divorce from his first wife. As a
result, Rahul has a step-sibling from his father's second marriage. Can Rahul and Pooja still get legally married?
(a) Yes, as neither party has a living spouse, both parties have given valid consent, and they have fulfilled the age
requirement for marriage under the Hindu Marriage Act.
(b) No, because Rahul has a step-sibling from his father's second marriage, and the parties are within the degrees of
prohibited relationship.
(c) Yes, because the registrar cannot refuse to solemnize the marriage under the Special Marriage Act, 1954, if there
is a pre-existing marriage of either party.
(d) All of the above.

38. Sneha, a 23-year-old Hindu woman, is in love with Rohan, a 25-year-old Muslim man. They want to get married,
but Sneha's parents are against it because of Rohan's religion. Sneha and Rohan decide to get married under the
Special Marriage Act, which allows individuals of different religions to marry. However, during the registration process,

Legal Reasoning Practice Sheet 12


CLAT POINT

the registrar discovers that Rohan has been diagnosed with a mental disorder that makes him incapable of giving valid
consent to the marriage. Can Sneha and Rohan still get legally married?
(a) Yes, because the Special Marriage Act allows individuals of different religions to marry, and Sneha and Rohan
fulfill the age and other requirements.
(b) No, because Rohan is incapable of giving valid consent to the marriage due to his mental disorder, which is a
condition mentioned in the act.
(c) Yes, because Art. 21 of the Constitution of India guarantees the right to life, which includes the right to marry, and
any restrictions on this right must be in accordance with the procedure established by law.
(d) None of the above.

39. Meera, a 20-year-old Hindu woman, wants to marry her long-time partner Ravi, who is a 25-year-old Hindu man.
However, Meera's parents are against the marriage as they belong to a different caste, which is considered socially
unacceptable under Hindu customs. Meera and Ravi decide to get married in a court. However, during the registration
process, the registrar discovers that Ravi was previously married and is now seeking a divorce from his first wife. Can
Meera and Ravi still get legally married?
(a) No, because Ravi is still legally married, and the Hindu Marriage Act prohibits the solemnization of a second
marriage unless the first marriage has been legally dissolved.
(b) Yes, as Meera and Ravi fulfill the age requirement for marriage, and Meera's parents' objection to their marriage
based on caste is not a legal ground for refusal of marriage registration.
(c) No, because Meera and Ravi belong to different castes, which is considered a prohibited relationship under Hindu
customs, and hence their marriage would be void under the Hindu Marriage Act.
(d) All of the above.

40. X, a Hindu man, wishes to marry Y, a Hindu woman. However, Y's parents do not consent to the marriage as they
belong to a different caste than X. Which of the following options is legally valid?
(a) X and Y can marry under the Hindu Marriage Act, 1955, as the Act permits marriages between Hindus of different
castes.
(b) X and Y cannot marry as Y's parents' consent is required for the marriage to be valid under Hindu law.
(c) If X and Y cannot marry under Indian law then they can still marry under Article 16 of the Universal Declaration of
Human Rights, which grants the right to marry without any limitation based on race, nationality or religion.
(d) X and Y cannot marry as it violates the fundamental group unit of society as stated in Article 16 of the Universal
Declaration of Human Rights.

Passage 9

Cruelty is a ground for matrimonial relief under all the matrimonial law statutes in India. The same has, however, not
been defined and rightly so, for human nature and conduct are infinitely diverse—what is considered as cruelty today
was not so construed a few decades back, and acts which may not constitute cruelty today might be so regarded after
a few years. As aptly remarked by the apex court in Ravi Kumar v. Julmi Devi, cruelty has no definition; in fact such
definition is not possible. Cruelty in matrimonial cases can be of infinite variety. It defies any definition and its
categories can never be closed. In other words, the concept of cruelty is very subjective—varying with time, place and
persons.

Under the Hindu Marriage Act, 1955 as amended by the Marriage Laws (Amendment) Act, 1976, cruelty is a ground
for divorce as well as for judicial separation. Section 13(1) (a) states that a marriage may be dissolved on the ground
that the other party has ‘after the solemnisation of the marriage, treated the petitioner with cruelty’. Prior to 1976,
cruelty was only a ground for judicial separation.

Another significant change brought about by the 1976 Amendment is that the concept of cruelty has been enlarged.
Earlier, it was confined to ‘such cruelty so as to cause reasonable apprehension in the mind of the petitioner that it
will be harmful or injurious for the petitioner to live with the other party’. However, now the petitioner has simply to

Legal Reasoning Practice Sheet 13


CLAT POINT

establish that the respondent has ‘treated the petitioner with cruelty’. There are no conditions as regards the nature
or fear of injury or harm.

Apart from the personal laws, a new dimension has been added to the concept of cruelty by adding s. 498A to the
Indian Penal Code, 1860 vide the Criminal Laws (Second Amendment) Act, 1983. This section provides for punishment
to a husband or his relatives who subject a woman to cruelty. Cruelty under this section means any wilful conduct
which is of such nature as is likely to drive a woman to suicide or to cause grave injury or danger to life, limb or health
(whether mental or physical). Harassment of the woman, where such harassment is with a view to coercing her or any
person related to her to meet any unlawful demand for any property or valuable security, would also constitute cruelty

41. Mr. X has been married to Mrs. Y for ten years. During this time, Mr. X has been verbally abusive towards Mrs. Y,
calling her names and belittling her in front of others. He has also been physically violent towards her on a few
occasions. Mrs. Y has decided to file for divorce on the grounds of cruelty. Which of the following options is correct?
(a) Mrs. Y cannot file for divorce on the grounds of cruelty under the Hindu Marriage Act, 1955 because the act of
cruelty needs to be continuous and repetitive.
(b) Mrs. Y can file for divorce on the grounds of cruelty under the Indian Penal Code, 1860.
(c) Mrs. Y cannot file for divorce on the grounds of cruelty because the Hindu Marriage Act, 1955 only recognizes
physical violence as an act of cruelty.
(d) Mrs. Y can file for divorce on the grounds of cruelty under the Hindu Marriage Act, 1955 provided she can prove
that the acts of cruelty have caused her mental or physical trauma.

42. Mr. X, a Hindu, has been suffering from harassment, abusive language and threats of false criminal cases from his
wife, Mrs. Z, since the beginning of their marriage. She has also mercilessly beaten their children. One day, while
pregnant with their fourth child, Mrs. Z pushed their three children into a well and jumped after them. While she
survived, the children died. Mr. X has filed for divorce on the grounds of cruelty. Which of the following options is
correct?
(a) Mr. X cannot be granted divorce because his wife's conduct cannot amount to cruelty since she was mentally
unstable at the time of the incident.
(b) Mr. X can be granted divorce because his wife's actions constituted cruelty under the Hindu Marriage Act.
(c) Mr. X cannot be granted divorce because the incident happened while Mrs. Z was pregnant and therefore, she was
not in control of her actions.
(d) Mr. X can be granted divorce only if he can prove that he had no role to play in the incident.

43. Mrs. E has been married to Mr. F for three years, but Mr. F has been emotionally abusive towards her, constantly
belittling her and making her feel inadequate. Mrs. E wants to file for divorce on the grounds of cruelty under the
Hindu Marriage Act, 1955. Which of the following legal principles is relevant to this situation?
(a) The act of cruelty must be continuous and repetitive for a divorce to be granted on the grounds of cruelty.
(b) Mental cruelty is not a recognized ground for divorce under the Hindu Marriage Act, 1955.
(c) The petitioner must establish that the respondent has treated the petitioner with cruelty to obtain a divorce on the
grounds of cruelty.
(d) The Hindu Marriage Act, 1955 only recognizes physical violence as an act of cruelty.

44. Which of the following scenarios constitutes 'cruelty' under the personal laws and the Indian Penal Code, as
mentioned in the passage?
(a) A husband constantly belittles his wife, calling her names and making derogatory comments about her appearance.
(b) A wife refuses to have sexual relations with her husband, claiming that she is not interested in him physically.
(c) A couple argues frequently over trivial matters, such as household chores and errands, but do not engage in physical
or verbal abuse.
(d) A husband and his family members harass the wife and demand dowry, leading to her suicide.

Legal Reasoning Practice Sheet 14


CLAT POINT

45. Which of the following scenarios would be considered an act of cruelty under section 498A of the Indian Penal
Code, 1860?
(a) A husband forgets his wife's birthday and does not buy her a gift, causing her to feel sad and upset.
(b) A husband and his family members repeatedly threaten to harm and physically abuse the wife if she does not bring
additional dowry money from her parents.
(c) A husband and his wife have an argument and he call her a derogatory name, causing her to feel emotionally hurt.
(d) A husband and his wife have different preferences for food and he cooks only his preferred dishes, causing her to
feel dissatisfied with the meals.

Passage 10
In Bipinchandra Jai Singhbai Shah v. Prabhavati, which was a case under the Bombay Hindu Divorce Act, 1947, the
court explained the concept of desertion. It held that “if a spouse abandons the other in a state of temporary passion,
for example, anger or disgust without intending permanently to cease cohabitation, it will not amount to desertion.
For the offence of desertion, so far as the deserting spouse is concerned, two essential conditions must be there,
namely i) the factum of desertion and ii) the intention to bring cohabitation permanently to an end (animus decidendi).
Similarly, two elements are essential so far as the deserted spouse is concerned: (i) the absence of consent; and (ii)
absence of conduct giving reasonable cause to the spouse leaving the matrimonial home to form the necessary
intention aforesaid. The petitioner for divorce bears the burden of proving those elements in the two spouses
respectively. Desertion is a matter of inference to be drawn from the facts and circumstances of each case.”

The inference may be drawn from certain facts which may not in another case be capable of leading to the same
inference; If, in fact, there has been a separation, the essential question always is whether that act could be
attributable to an animus deserendi. The offence of desertion commences when the fact of separation and the animus
deserendi coexist. But it is not necessary that they should commence at the same time. The de facto separation may
have commenced without the necessary animus or it may be that the separation and the animus deserendi coincide
in point of time; for example, when the separating spouse abandons the marital home with the intention, express or
implied, of bringing cohabitation permanently to a close if a deserting spouse takes advantage of the locus
poenitentiae and decide to come back to the deserted spouse by a bona fide offer of resuming the matrimonial home
with all implications of marital life, before the statutory period is out or even after the lapse of that period, unless
proceedings for divorce have been commenced, desertion comes to an end and if the deserted spouse unreasonably
refuses the offer, the latter may be in desertion and not the former. Hence it is necessary that during all the period
that there has been a desertion, the deserted spouse must affirm the marriage and be ready and willing to resume
married life on such conditions as may be reasonable. It is also well settled that in proceedings for divorce the plaintiff
must prove the offence of desertion like any other matrimonial offence, beyond all reasonable doubt. Hence, though
corroboration is not required as an absolute rule of law the courts insist upon corroborative evidence, unless its
absence is accounted for to the satisfaction of the court. Desertion is a ground for judicial separation and also for
divorce under the various matrimonial laws.

46. Raj and Reena got married in 2010 and have been living together since then. In 2022, Raj left the matrimonial
home after a heated argument with Reena. He did not return for a period of six months, during which time Reena
made several attempts to contact him and reconcile, but he refused. In this scenario, would Raj's behavior be
considered an act of desertion under the Bombay Hindu Divorce Act, 1947?
(a) Yes, because Raj left the matrimonial home without the intention of returning and Reena did not consent to his
absence.
(b) No, because Raj's absence was temporary and he may have left in a state of temporary passion.
(c) No, because Reena made several attempts to contact him and reconcile, but he refused.
(d) Yes, because Raj's behavior caused Reena mental and emotional anguish and he did not make any attempts to
reconcile.
Legal Reasoning Practice Sheet 15
CLAT POINT

47. In the case of Mrs. X and Mr. Y, can Mrs. X be held liable for desertion if she was compelled to leave the matrimonial
home due to Mr. Y's mental and physical torture, but the desertion was not permanent and she expressed a willingness
to resume cohabitation under certain conditions?
(a) Yes, because Mrs. X's intention was to end cohabitation with Mr. Y, regardless of the reason for leaving.
(b) Yes, because Mrs. X deserted her husband, regardless of the reason for leaving.
(c) No, Mrs. X is not liable for desertion because Mr. Y's conduct was giving reasonable cause to Mrs. X to form the
necessary intention of deserting him, and her absence was not permanent.
(d) It depends on whether Mrs. X made a bona fide offer to resume cohabitation before the statutory period for
desertion had ended.

48. A and B are married for five years. One day, A leaves the house in a state of anger after a heated argument and
does not return for several days. During this period, B tries to contact A but is unsuccessful. A comes back after two
months and apologizes to B for leaving but B refuses to accept the apology and files for divorce on the grounds of
desertion.
(a) B's claim for desertion will be successful as A left without B's consent and did not intend to return permanently.
(b) B's claim for desertion will not be successful as A left due to temporary passion and not with the intention to end
cohabitation permanently.
(c) B's claim for desertion will not be successful as A came back within the statutory period and offered to resume
matrimonial life.
(d) B's claim for desertion will be successful as A did not offer any explanation for leaving and failed to contact B during
the period of separation.

49. What is the importance of corroboration in cases of desertion?


(a) It is not a requirement in law, but courts insist on corroborative evidence unless its absence is accounted for to the
satisfaction of the court.
(b) It is a mandatory requirement in all cases of desertion for the plaintiff to prove the offense beyond all reasonable
doubt.
(c) It is required only if the deserting spouse denies the act of desertion in court.
(d) It is required only if the deserting spouse has been charged with other matrimonial offenses.

50. Which of the following acts cannot be classified as desertion under the matrimonial laws of India, as expounded in
the landmark case of Bipinchandra Jai Singhbai Shah v. Prabhavati, and based on the essential elements required for
such a classification, which include the factum of desertion and animus deserendi, absence of consent, and absence
of conduct giving reasonable cause to the spouse leaving the matrimonial home to form the necessary intention
aforesaid?
(a) The wife leaving her husband after he had subjected her to persistent physical and mental cruelty, including
harassment for dowry, leading to a breakdown in their marital relationship and a genuine fear for her safety and well-
being.
(b) The wife voluntarily leaving her husband to go overseas and marry someone else, thereby exhibiting a clear
intention to permanently end her matrimonial ties with her husband and move on with her life, without any
reasonable cause or justification for such an action.
(c) The wife choosing to live separately from her husband, with a clear intention to end their marriage, due to
irreconcilable differences, disagreements and dissatisfaction with her husband’s behavior, without any attempt to
resolve their issues or any reasonable cause for such a separation.
(d) None of the above options can be excluded from the ambit of desertion, as each of them can potentially meet the
requirements of the essential elements of desertion, as established by the aforementioned case law.

Passage 11

Legal Reasoning Practice Sheet 16


CLAT POINT

The doctrine of res judicata requires that a party should not be allowed to file the same matter repeatedly against the
other party either in the same court or in other competent court and that the decision given by one court should be
accepted as final subject to any appeal, revision or review. Res Judicata is only applicable in suits between the same
parties. The doctrine is founded on the principle that it is in the interest of the public at large that a finality should be
attached to the binding decisions pronounced by courts of competent jurisdiction, and it is also in the public interest
that individuals should not be vexed twice over with the same kind of litigation.

This apart, the object of the doctrine is to ensure that ultimately there should be an end to litigation. Doctrine of res
judicata is embodied in Section 11 of the Code of Civil Procedure, 1908 which governs the procedure to be followed
in civil matters. Section 11 is inapplicable to writ jurisdictions. The Supreme Court has observed that though the rule
is technical in nature yet the general doctrine of res judicata is based on public policy and therefore, it cannot be
treated as irrelevant or inadmissible even in dealing with fundamental rights in petitions filed under Article 32 of the
Constitution of India. The court observed that if a writ petition filed by a party under Article 226 of the Constitution of
India is considered on merits as a contested matter and is dismissed, the decision thus pronounced would continue to
bind the parties unless it is otherwise modified or reversed in appeal or other appropriate proceedings permissible
under the Constitution of India. It would not be open to a party to ignore the judgment of the High Court and move
Supreme Court under Article 32 by an original petition made on the same facts and for obtaining the same or similar
orders or writs.

If the petition filed in the High Court under Article 226 is dismissed but not on the merits, then the dismissal of the
writ petition would not constitute a bar to a subsequent petition under Article 32, however if the petition is dismissed
without passing a speaking order, then such dismissal cannot be treated as creating a bar of res judicata.

51. In a legal battle between Mr Mahesh and Mr Shivam over possession and title of temple property, Mr Mahesh,
who claimed to be the rightful heir of the chief-priest Mr Jayant and the owner and manager of the temple, failed to
produce valid succession documents to support his claim. Consequently, the court dismissed the case, and Mr Shivam
was able to prove his own succession to Mr Jayant. After a month, Mr Mahesh files another suit against Mr Shivam for
the same cause. Is the subsequent suit barred by legal principles?
(a) The doctrine of res judicata bars the subsequent suit, as the earlier case was dismissed on merits, and the parties
and cause of action are the same.
(b) The doctrine of res judicata does not bar the subsequent suit, as the earlier case was dismissed for lack of valid
succession documents, and there may be new evidence to support Mr Mahesh's claim in the subsequent suit.
(c) The principle of res subjudice bars the subsequent suit, as the earlier case is still pending before another court.
(d) None of the above.

52. In a dispute between X and Y over the payment of rent for the use of land, Y raises the contention that Z is the
rightful owner of the land, not X. X fails to provide sufficient evidence of his title and the case is dismissed.
Subsequently, X files a new suit against both Y and Z seeking a declaration of his title to the land. Would this new suit
be considered barred by the doctrine of Res Judicata?
(a) The new suit is not barred as the parties involved are not identical in both cases.
(b) The new suit is barred as the relief sought is substantially similar to the previous case.
(c) The new suit is barred as the parties involved in both cases are the same.
(d) None of the above.

53. Mr. A filed a writ petition under Article 226 of the Constitution of India in the High Court, seeking a direction to
the state government to provide him with a job. The High Court dismissed the petition for lack of jurisdiction, without
passing a speaking order. Can Mr. A file a subsequent petition under Article 32 of the Constitution of India in the
Supreme Court on the same facts and for obtaining the same or similar orders or writs?
(a) Yes, because the dismissal of the writ petition by the High Court for lack of jurisdiction does not create a bar of res
judicata.
Legal Reasoning Practice Sheet 17
CLAT POINT

(b) No, because the general doctrine of res judicata is based on public policy and applies to all legal matters, regardless
of the court's jurisdiction.
(c) Yes, but only if Mr. A can demonstrate that the dismissal of the writ petition by the High Court was not on the
merits.
(d) No, because Mr. A's petition under Article 226 of the Constitution of India was already dismissed.

54. Mr. X filed a suit against Mr. Y for breach of contract in the District Court. The District Court dismissed the suit on
the ground of limitation. Mr. X filed an appeal before the High Court, which also dismissed the appeal on the same
ground. Mr. X then filed a writ petition under Article 226 of the Constitution of India in the High Court, seeking a
direction to the District Court to entertain his suit on the ground that the District Court's decision was erroneous. The
High Court dismissed the writ petition on the ground of res judicata. Can Mr. X file a subsequent petition under Article
32 of the Constitution of India in the Supreme Court on the same facts and for obtaining the same or similar orders or
writs?
(a) Yes, because the dismissal of the writ petition by the High Court on the ground of res judicata does not create a
bar to a subsequent petition under Article 32 of the Constitution of India.
(b) No, because Mr. X's petition under Article 226 of the Constitution of India was already dismissed on the ground of
res judicata.
(c) Yes, but only if Mr. X can demonstrate that the dismissal of the writ petition by the High Court was not on the
merits.
(d) No, because the doctrine of res judicata applies to all legal matters, regardless of the court's jurisdiction.

55. A man, Mr. Smith, was driving his car when he collided with a pedestrian, Ms. Jones. The police filed a case against
Mr. Smith for reckless driving causing injury to Ms. Jones. During the trial, Ms. Jones testified that she was crossing
the road at a pedestrian crossing when Mr. Smith's car hit her. Mr. Smith's lawyer argued that Ms. Jones was not using
the pedestrian crossing and was jaywalking at the time of the accident. The court accepted Mr. Smith's argument and
acquitted him of all charges. However, Ms. Jones subsequently filed a civil suit against Mr. Smith for damages. Will Ms.
Jones be successful in her civil suit against Mr. Smith?
(a) Ms. Jones will be successful in her civil suit against Mr. Smith because the standard of proof in civil cases is lower
than that in criminal cases.
(b) Ms. Jones will not be successful in her civil suit against Mr. Smith because the court has already acquitted him of
all charges in the criminal case.
(c) Ms. Jones will be successful in her civil suit against Mr. Smith because the court in the criminal case only decided
the criminal liability, and not the civil liability.
(d) None of the above.

Passage 12
Recently, we have seen a surfeit of criminal cases involving alleged obscenity in public. Despite the fundamental right
to freedom of speech and expression enshrined in article 19(1) (a) of the constitution of India, noted artists and actors
have been charged under section 292 of the Indian Penal Code. Indian law on obscenity is often misused in the pursuit
of moral interests.

An exception to the fundamental right of free speech and expression guaranteed under article 19(1)(a) of the
constitution are laws that impose reasonable restrictions in the interest of decency and morality. These vague and
changeable notions differ between societies and depend on the cultural values and moral standards that shape the
history and society of a country.

In India “obscenity” offences are listed under section 292 of the Indian Penal Code. “Obscenity” is defined as that
which is “lascivious or appeals to prurient interest” or which has the tendency to “deprave” and “corrupt” those who
are likely to be exposed to it.

Legal Reasoning Practice Sheet 18


CLAT POINT

The courts have chosen to adopt the old English Hicklin’s test: “whether the tendency of the matter charged as
obscenity is to deprave and corrupt those whose minds are open to such immoral influences, and in whose hands a
publication of this sort may fall”. This test of obscenity was laid down in the Hicklin case in 1869, but has not been
used in England since the enactment of the Obscene Publications Act 1959.

Interestingly in 1965 the Supreme Court of India chose to adopt it in the case of Ranjit D Udeshi v State of Maharashtra,
when the definition of obscenity came up for consideration for the first time. The court felt that the Hicklin’s test
should be used as the test makes the court a judge of obscenity and emphasizes the potential of the impugned object
to deprave and corrupt by immoral influences.

In that case Udeshi was prosecuted along with the other partners of a bookstall who were in possession (for the
purposes of sale) of the unexpurgated edition of the book Lady Chatterley’s Lover. Focusing solely on the impugned
passages, the Supreme Court said that where art and obscenity are mixed, what must be seen is whether the artistic,
literary or social merit of the work in question outweighs its obscene content. The element of art must overshadow
the obscenity or make it so trivial and inconsequential that it can be ignored.
Amendments such as that of section 69A of the Information Technology Act, 2000, which came into effect on 27
October 2009, have raised the bar for the government to block obscenities on websites. The government can still block
such websites, but only if they create a public order problem. A minor but critical difference between section 69A and
the earlier version of section 69 means that websites can be barred only on five specific grounds – sovereignty and
integrity of India, defence of India, security of the state, friendly relations with foreign states and public order. Earlier,
the government could ban websites for the purpose of “preventing incitement to the commission of any cognizable
offence” including obscenity.

56. Mr. X, a noted artist, has created a painting that depicts nudity. The painting is displayed in an art exhibition and
is open to the public. Can Mr. X be charged under section 292 of the Indian Penal Code for obscenity?
(a) Yes, because the painting depicts nudity, which may be considered lascivious or appeal to prurient interest.
(b) No, because the element of art in the painting may overshadow any obscenity or make it so trivial and
inconsequential that it can be ignored.
(c) Yes, but only if the painting has the tendency to deprave and corrupt those who are likely to be exposed to it.
(d) No, because the government can only block websites on specific grounds, and the display of a painting in an art
exhibition does not create a public order problem.

57. Mr. S, the accused, was found to be listing video-clips of women engaging in explicit sexual activities for sale. He
was also inviting people, including children, to purchase and view these video-clips. In light of these circumstances,
can Mr. S be charged under section 292 of the Indian Penal Code?
(a) Yes, Mr. S can be charged under section 292 of the Indian Penal Code, as the video-clips listed have the tendency
to "deprave" and "corrupt" those who are likely to be exposed to it.
(b) No, Mr. S cannot be charged under section 292 of the Indian Penal Code, as the sale of video-clips depicting sexual
activities is protected under the fundamental right to freedom of speech and expression.
(c) No, Mr. S cannot be charged under section 292 of the Indian Penal Code, as it is important for children to receive
sex education, and the video-clips listed may serve as a means of education.
(d) Both b and c are wrong, as the sale of video-clips depicting sexual activities to children is not protected under the
fundamental right to freedom of speech and expression, and the video-clips listed cannot be considered as a means
of sex education.

58. The accused was found selling a packet of playing cards that portrayed luridly obscene naked pictures of men and
women in pornographic sexual postures on the reverse. Upon conducting a raid of his shop, two more similar packets
were recovered from him. In light of these circumstances, which of the following sections of the Indian Penal Code
(IPC) or Information Technology (IT) Act can be used to punish the accused?

Legal Reasoning Practice Sheet 19


CLAT POINT

(a) Section 69A of the IT Act only, as it deals with the power to issue directions for blocking for public access of any
information through any computer resource.
(b) Section 292 of the IPC only, as it deals with obscenity, which is defined as that which is lascivious or appeals to
prurient interest or which has the tendency to deprave and corrupt those who are likely to be exposed to it.
(c) Both a and b, as the accused was found to be selling obscene content and the provisions of the IT Act can be applied
to block access to such content.
(d) None of the above, as the accused's actions do not come under the purview of either section 69A of the IT Act or
section 292 of the IPC.

59. The accused was found selling a packet of playing cards that portrayed luridly obscene naked pictures of men and
women in pornographic sexual postures on the reverse. Upon conducting a raid of his shop, two more similar packets
were recovered from him. Further investigation revealed that the accused had a website on his computer, which he
used to sell the cards online to various customers. It was also found that over 500 orders of such cards had been
already delivered by him to various customers. In light of these circumstances, which of the following sections of the
Indian Penal Code (IPC) or Information Technology (IT) Act can be used to punish the accused?
(a) Section 69A of the IT Act only, as it deals with the power to issue directions for blocking public access of any
information through any computer resource.
(b) Section 292 of the IPC only, as it deals with obscenity, which is defined as that which is lascivious or appeals to
prurient interest or which has the tendency to deprave and corrupt those who are likely to be exposed to it.
(c) Both a and b, as the accused was found to be selling obscene content and had used a website to commit the offence,
which created a public order problem.
(d) None of the above, as the accused's actions do not come under the purview of either section 69A of the IT Act or
section 292 of the IPC.

60. A group of street performers put on a show in a public park that involves nudity and simulated sexual acts. A
passer-by complains to the police, who arrive and arrest the performers for obscenity. Can the performers be charged
under section 292 of the Indian Penal Code?
(a) Yes, the performers can be charged under section 292 of the Indian Penal Code.
(b) No, the performers cannot be charged under section 292 of the Indian Penal Code.
(c) The performers can be charged under section 292 of the Indian Penal Code, but only if they failed to consider the
impact of their performance on public decency.
(d) It depends on the specific details of the case.

Passage 13
The Indian legal system does indeed have some teeth to tackle instances of fake news or rumours, be they in the real
world or online. The problem is that such regulation is largely focused on hate speech and defamation. In reality, as
we know, fake news goes much beyond hate speech. "It's high time that we have some direct provisions to govern
fake news and rumours. At the time when the Indian Penal Code (IPC) was enacted, these problems did not exist;
hence the current laws are inadequate," advocate and cyber law expert Karnika Seth remarked.

So, Section 124A of the IPC punishes any form of speech as an act of sedition when it is aimed against the State, while
Section 153A of the IPC prohibits and punishes promotion of enmity between different groups on grounds of religion,
etc. Instances of circulation of any statement or rumour causing public mischief and hatred between classes as well as
deliberate and malicious acts, intended to outrage religious feelings of any class by insulting its religion are also
punishable under the IPC.

However, how does the law deal with seemingly benign rumours and morphed videos and messages? Indian cyber
laws have no direct provision governing rumours on social and electronic media. The Information Technology (IT) Act,
however, imposes a limited liability on intermediaries such as search engine giant Google for providing a platform to
any objectionable content. While exempting intermediaries from liability for any third party content, Section 79 of the
Legal Reasoning Practice Sheet 20
CLAT POINT

IT Act imposes an obligation on them to remove any such content pursuant to takedown notices by law enforcement
agencies. In the absence of proper laws, the government and law enforcement agencies, as well as the district
magistrate(s), are increasingly using the immense powers they have under the Code of Criminal Procedure, 1973 to
prevent individuals from committing "a breach of the peace" or "disturbing the public tranquillity". That's why India is
increasingly resorting to internet shutdowns. According to think-tank ICRIER, there have been 16,315 hours of Internet
shutdowns in India during the period 2012 to 2017.

Similarly, wary of the impact that fake news can have on voting during the Karnataka state elections, the Election
Commission in May sought help from the technology firms in the region to identify instances of fake news on social
media. “People need to be aware of WhatsApp messages that can create havoc. People circulate the messages blindly.
Law needs one to be mindful", warned Seth. Acknowledging that law-making is a long process, she recommended that
the government must come up with advisories till the time there are no specific provisions of law to tackle the issue.

Any legislative endeavour to curb the epidemic of "fake news" is akin to walking through a minefield as the
consequential repercussions of a stringent law against fake content include its possible impact on free speech and
freedom of the press.

61. Mr. Z, a Kashmiri youth, was arrested in Hyderabad on the charge of sedition. The only evidence against him was
that he was spreading news about the commission of atrocities by the Indian army against Kashmiri Muslims. The
charge framed against him did not contain any allegations of him acting against the Government of India or the State
government. In this scenario, can a case be made against
Mr. Z under section 124A of the Indian Penal Code (IPC)?
(a) Yes, a case can be made against Mr. Z under section 124A of the IPC since he acted against the Government of India
or any State government.
(b) No, a case cannot be made against Mr. Z under section 124A of the IPC since he did not act against the Government
of India or any State government.
(c) The facts provided are insufficient to make a decision.
(d) None of the above options is correct.

62. In the state of Maharashtra, a social media influencer, Ms X, was arrested on charges of promoting enmity between
different religious groups on grounds of religion, through a video she posted on her social media page. The video
allegedly had inflammatory content that was derogatory towards a particular religious group. The charge against her
was filed under Section 153A of the IPC. Can Ms X be held guilty under Section 153A of the IPC?
(a) Yes, Ms X can be held guilty under Section 153A of the IPC as the video posted by her had content that promoted
enmity between different religious groups on grounds of religion.
(b) No, Ms X cannot be held guilty under Section 153A of the IPC as freedom of expression allows an individual to
express their views and opinions, even if they may be derogatory towards a particular religious group.
(c) The charge under Section 153A of the IPC can only be upheld if Ms X had the intention of promoting enmity between
different religious groups on grounds of religion.
(d) It depends on whether the video posted by Ms X had a real likelihood of causing public disorder or violence.

63. Mr X, a resident of India, published messages and slogans on his website that incited violence against people of
different religions. Moreover, his messages also endorsed the idea of waging a war against the Government of India
and Indian Army to separate his native state from the territory of India. The authorities arrested Mr X on charges of
sedition and promoting enmity between different groups on grounds of religion under Sections 124A and 153A of the
Indian Penal Code, respectively. Can Mr X be prosecuted under both sections?
(a) Yes, Mr X can be prosecuted under both Sections 124A and 153A of the Indian Penal Code as he has committed
offences under both sections.
(b) No, Mr X cannot be prosecuted under both sections as the offences committed by him fall under only one section.
(c) Mr X can be prosecuted under Section 124A of the Indian Penal Code, but not under Section 153A.
Legal Reasoning Practice Sheet 21
CLAT POINT

(d) Mr X can be prosecuted under Section 153A of the Indian Penal Code, but not under Section 124A.

64. Mr. X, a politician, while giving a speech on one of his rallies remarked as follows - "The current government is
filled with bootlickers and dogs who do not care about the security and welfare of this country. Therefore, I request
all of my supporters to dethrone this corrupt government in the upcoming elections." The authorities arrested Mr. X
on charges of sedition under Section 124A of the Indian Penal Code. Can Mr. X be charged with sedition?
(a) This is not sedition as Mr. X is exercising his freedom of speech and expression and is only requesting his supporters
to vote against the current government in the upcoming elections.
(b) This is sedition as Mr. X is making derogatory remarks against the government and inciting his supporters to
overthrow the government by use of force or violence.
(c) The charge of sedition can only be upheld if Mr. X had the intention of inciting violence or rebellion against the
government and if his words had a real likelihood of causing public disorder or violence.
(d) It depends on whether Mr. X had the intention of inciting violence or rebellion against the government and if his
words had a real likelihood of causing public disorder or violence.

65. Mr. Z, a social media user, posted a message on his Facebook page claiming that a popular food chain was using
dog meat in their burgers. The message went viral, leading to protests and boycotts of the food chain. However, it was
later found that the message was false and baseless. The food chain filed a complaint against Mr. Z for spreading fake
news and causing damage to their brand reputation. Which of the following legal provisions can be used to prosecute
Mr. Z?
(a) Section 295A of the Indian Penal Code for deliberately and maliciously outraging religious feelings of any class by
insulting its religion or religious beliefs.
(b) Section 153A of the Indian Penal Code for promoting enmity between different groups on grounds of religion, race,
place of birth, residence, language, etc.
(c) Section 79 of the Information Technology Act for failing to remove objectionable content pursuant to takedown
notices by law enforcement agencies.
(d) None of the above.

Passage 14
Section 375 of the Indian Penal Code defines rape as "sexual intercourse with a woman against her will, without her
consent, by coercion, misrepresentation or fraud or at a time when she has been intoxicated or duped, or is of unsound
mental health and in any case if she is under 18 years of age."
It's rape if it falls under following categories:
a. Against her will.
b. Without her consent.
c. With her consent, when her consent has been obtained by putting her or any person in whom she is interested in
fear of death or of hurt.
d. With her consent, when the man knows that he is not her husband, and that her consent is given because she
believes that he is another man to whom she is or believes herself to be lawfully married.
e. With her consent, when, at the time of giving such a consent, by reason of unsoundness of mind or intoxication or
the administration by him personally or through another of any stupefying or unwholesome substance, she is unable
to understand the nature and consequences of that to which she gives consent.
f. With or without her consent, when she is under sixteen years of age. Explanation: Penetration is sufficient to
constitute the sexual intercourse necessary to the offence of rape.

Consent under Section 375


Consent is defined as clear, voluntary communication that the woman gives for a certain sexual act. Marital rape is an
exception to giving consent as it is not a crime under the Indian Penal Code, as long as the woman is above 18 years
of age.

Exceptions to Section 375


Sexual intercourse by a man with his own wife who is above the age of 18, is not sexual assault.
Legal Reasoning Practice Sheet 22
CLAT POINT

Amendment to Section 375 of IPC


The Criminal Law (Amendment) Act, 2013 or the Nirbhaya Act, was passed in Parliament to amend Section 375. To
remove ambiguity in the earlier law and provide for strict punishment in cases of rarest cases of sexual violence, the
legislation was expanded to define acts like penetration of penis into vagina, urethra, anus or mouth, or any object or
any part of body to any extent into the aforesaid woman body parts (or making another person do so), as constituting
an offence of sexual assault. Applying mouth or touching private parts were also classified as offences of sexual assault.
Punishment

Except in certain aggravated situations, the punishment will be imprisonment of not less than seven years but it may
extend to imprisonment for life, and shall also be liable to fine. In aggravated situations, punishment will be rigorous
imprisonment for a term which shall not be less than 10 years but which may extend to imprisonment for life, and
shall also be liable to fine.

66. Mr. X, a wealthy businessman, invites Ms. Y, a young woman, to his luxurious apartment for dinner. After dinner,
Mr. X offers Ms. Y a drink, which she accepts. Soon after, Ms. Y begins to feel dizzy and disoriented. She realizes that
Mr. X has spiked her drink with a sedative. Mr. X proceeds to sexually assault Ms. Y. Which of the following legal
provisions can be used to prosecute Mr. X?
(a) Section 375(d) of the Indian Penal Code for having sexual intercourse with Ms. Y with her consent, when Mr. X
knew that he is not her husband and that her consent was given because she believed that he is another man to whom
she is or believes herself to be lawfully married.
(b) Section 375(e) of the Indian Penal Code for having sexual intercourse with Ms. Y with her consent, when at the
time of giving such consent, by reason of unsoundness of mind or intoxication, she was unable to understand the
nature and consequences of the act to which she gave consent.
(c) Section 375(f) of the Indian Penal Code for having sexual intercourse with Ms. Y, who is under 16 years of age.
(d) None of the above.

67. Mr. X, a violin teacher, told Ms. Y, one of his female students, that he had a unique method for improving her violin
skills. He informed her that if she agreed, he would perform the method on her. Ms. Y, trusting her teacher's expertise,
agreed to the proposal. However, the method that Mr. X performed on Ms. Y involved sexual intercourse. Mr. X argues
that the act was consensual and that Ms. Y had agreed to it for the purpose of improving her violin skills. Which of the
following legal provisions can be used to determine if Mr. X is guilty of rape?
(a) Section 375(a) of the Indian Penal Code for having sexual intercourse with a woman against her will.
(b) Section 375(b) of the Indian Penal Code for having sexual intercourse with a woman without her consent.
(c) Section 375(c) of the Indian Penal Code for having sexual intercourse with a woman with her consent, when her
consent has been obtained by putting her or any person in whom she is interested in fear of death or of hurt.
(d) Section 375(e) of the Indian Penal Code for having sexual intercourse with a woman with her consent, when at the
time of giving such consent, by reason of unsoundness of mind or intoxication, she was unable to understand the
nature and consequences of the act to which she gave consent.

68. Mr. X had forceful sexual intercourse with his wife, who was 19 years of age. Is Mr. X guilty of committing the
offence of rape under Section 375 of the Indian Penal Code?
(a) Yes, Mr. X is guilty of committing the offence of rape under Section 375 of the Indian Penal Code, even if the victim
was his wife.
(b) No, Mr. X is not guilty of committing the offence of rape under Section 375 of the Indian Penal Code, as the victim
was his wife and above the age of 18.
(c) Yes, Mr. X is guilty of committing rape because the sexual intercourse was forceful and against the wife's will.
(d) None of the above options are correct.

69. Ms. A is a renowned artist who is invited to perform at a private party hosted by Mr. B, a wealthy businessman.
After the performance, Mr. B offers Ms. A a drink, which she accepts. Soon after, Ms. A begins to feel dizzy and
Legal Reasoning Practice Sheet 23
CLAT POINT

disoriented. She realizes that Mr. B has spiked her drink with a sedative. Mr. B proceeds to sexually assault Ms. A.
Which of the following legal provisions can be used to prosecute Mr. B?
(a) Section 375(d) of the Indian Penal Code for having sexual intercourse with Ms. A with her consent, when Mr. B
knew that he is not her husband and that her consent was given because she believed that he is another man to whom
she is or believes herself to be lawfully married.
(b) Section 375(e) of the Indian Penal Code for having sexual intercourse with Ms. A with her consent, when at the
time of giving such consent, by reason of unsoundness of mind or intoxication, she was unable to understand the
nature and consequences of the act to which she gave consent.
(c) Section 376(2) of the Indian Penal Code for having sexual intercourse with a woman when she is in a state of
intoxication or under the influence of drugs and unable to give her consent.
(d) None of the above.

70. Ms. X is a 25-year-old woman who is invited to a party hosted by Mr. Y, a wealthy businessman. During the party,
Mr. Y offers Ms. X a drink, which she accepts. Soon after, Ms. X begins to feel dizzy and disoriented. She realizes that
Mr. Y has spiked her drink with a sedative. Mr. Y then proceeds to have sexual intercourse with Ms. X without her
consent. Which of the following legal provisions can be used to prosecute Mr. Y?
(a) Section 375(b) of the Indian Penal Code for having sexual intercourse with a woman without her consent.
(b) Section 375(c) of the Indian Penal Code for having sexual intercourse with a woman with her consent, when her
consent has been obtained by putting her or any person in whom she is interested in fear of death or of hurt.
(c) Section 376(2) of the Indian Penal Code for having sexual intercourse with a woman when she is in a state of
intoxication or under the influence of drugs and unable to give her consent.
(d) None of the above.

Passage 15
The prime objective of the Environmental laws is to protect the environment and form rules for the people on how to
use natural resources. It exists at many levels and partly constituted by conventions, declarations and treaties. It is
encompasses in the provisions of enactments laid down by the legislative bodies charged by the government with
protection of the environment. These laws are also based on certain important doctrines and legal theories which will
help us understand the intent of the maker.

Public Trust Doctrine


The public trust doctrine primarily rests on the principle that certain resources like air, sea, water and the forest have
such a great importance to the people as a whole and it is unjustified to make these resources subject to private
ownership.

The said resources are gift of the nature and there should be available free for all. The doctrine enjoins upon the
government to protect the resources for the enjoyment of the general public rather than to permit their use for private
ownership or commercial purposes.

Various public properties including rivers, seashore and the air are held by the government in trusteeship for the
uninterrupted use of the public so the government cannot transfer these properties to any private party who may
interfere with the interest of public at large.

Precautionary Principle
The principle of precaution involves to prevent the environmental harm and taking some measures to avoid it.
Environmental protection should not only aim at protecting health, property and economic interest but also protect
the environment for its own sake.

Legal Reasoning Practice Sheet 24


CLAT POINT

This principle suggest that where there is an identifiable risk of serious harm e.g. wide spread toxic pollution, extinction
of species and major threat to essential ecological process. It is also the duty of the Supreme Court to render justice
by taking all such aspects into consideration.

Polluter pays principle


It is a principle in the international environmental law where the polluter pays for the damage done to natural
environment. It is also known as the extended polluter responsibility, whosoever is responsible for the damage to the
environment should bear the cost associated to it. Its purpose is to shift the responsibility from the government to the
public. It is a simple extension of the principle of fairness and justice.

To enhance the economic efficiency that is to protect the environment without sacrificing the efficiency of a free
market economic system. The additional revenues are used to cut income, pay role and corporate taxes. Revenues
collected can help to achieve other social goals.

Some drawbacks of the doctrine are that ambiguity still exists in determining who the polluter is. It can cane difficult
to measure how much pollution is produces. a large number of poor house hold informal sector forms cannot bear
any additional charge for energy or for waste disposal. Pollution can be shifted to countries with weak legislation.

71. Mr. X is the owner of a chemical factory that produces toxic waste. The factory is located near a river that
supplies water to a nearby village. The villagers have complained of health problems due to the pollution caused by
the factory. Which of the following legal principles can be invoked against Mr. X?
(a) Public Trust Doctrine, as the river is a public resource that should be protected for the enjoyment of the general
public.
(b) Precautionary Principle, as the identifiable risk of serious harm to the villagers due to the toxic waste should be
prevented.
(c) Polluter Pays Principle, as Mr. X is responsible for the damage caused to the environment and should bear the
cost associated with it.
(d) None of the above.

72. Astra Pvt Ltd is a pharmaceutical company producing vaccines and medicines to combat the Covid-19 pandemic.
One of their factories, located in Varanasi beside the River Ganges, has been disposing of residual waste in the river,
causing pollution and serious health problems for local residents. Mr. X, who is a resident of the area, has filed a case
against the company seeking damages for the harm caused by the factory. Which of the following legal principles
should the court rely on to determine the compensation to be awarded to Mr. X?
(a) The Precautionary Principle, which emphasizes the importance of preventive measures to minimize environmental
damage and prevent harm to public health.
(b) The Polluters Pay Principle, which requires the polluter to pay for the damage caused to the natural environment.
This principle aims to ensure that the costs of pollution are borne by those who cause it, rather than the public at
large.
(c) The Public Trust Principle, which holds that certain natural resources belong to the public and should be protected
for the enjoyment of the general public. This principle prohibits the transfer of public resources, such as rivers and
seashores, to private parties who may interfere with the interest of the public at large.
(d) None of the above.

73. The Government of India recently enacted the "Protection of Wildlife and Waterbodies Act, 2021," which aimed
to punish individuals who caused harm to wildlife or waterbodies for their own private gains. One of the primary
features of this Act was that it empowered the Courts to impose fines on offenders based on the extent of damage
inflicted on wildlife or waterbodies. Which of the following legal principles was primarily relied upon by the
Government of India in enacting this law?
(a) The Precautionary Principle, which suggests that environmental protection should aim at preventing harm and
taking measures to avoid it, rather than assessing liability for damage already inflicted. This principle emphasizes the
importance of preventive measures to minimize environmental damage.

Legal Reasoning Practice Sheet 25


CLAT POINT

(b) The Polluters Pay Principle, which requires the polluter to pay for the damage caused to the natural environment.
This principle aims to ensure that the costs of pollution are borne by those who cause it, rather than the public at
large.
(c) The Public Trust Principle, which rests on the principle that certain natural resources belong to the public and should
be protected for the enjoyment of the general public. This principle prohibits the transfer of public resources, such as
rivers and seashores, to private parties who may interfere with the interest of the public at large.
(d) The Principle of Sustainable Development, which emphasizes the need to balance economic development, social
development, and environmental protection. This principle recognizes that economic growth and environmental
protection can go hand in hand if sustainable development practices are adopted.

74. The government of a small island nation has recently enacted a law called the "Environmental Protection Act"
aimed at protecting the marine environment from pollution. The Act contains provisions that prohibit the discharge
of pollutants into the ocean and empower the government to impose fines on offenders based on the extent of
damage inflicted on the marine environment. Which of the following legal principles was primarily relied upon by the
government in enacting this law?
(a) The Precautionary Principle, which suggests that environmental protection should aim at preventing harm and
taking measures to avoid it, rather than assessing liability for damage already inflicted.
(b) The Polluters Pay Principle, which requires the polluter to pay for the damage caused to the natural environment.
This principle aims to ensure that the costs of pollution are borne by those who cause it, rather than the public at
large.
(c) The Public Trust Principle, which rests on the principle that certain natural resources belong to the public and should
be protected for the enjoyment of the general public. This principle prohibits the transfer of public resources, such as
rivers and seashores, to private parties who may interfere with the interest of the public at large.
(d) The Principle of Inter-generational Equity, which emphasizes the need to balance the needs of the present
generation with the needs of future generations. This principle recognizes that the actions of the present generation
can impact the well-being of future generations.

75. A company located in a small town has been dumping hazardous waste in a nearby lake for several years. In recent
months, the water quality of the lake has deteriorated and several residents of the town have reported health
problems. A group of residents has filed a lawsuit against the company seeking compensation for the harm caused by
the dumping of hazardous waste. Which of the following legal principles should the court rely on to determine the
compensation to be awarded to the residents?
(a) The Precautionary Principle, which emphasizes the importance of taking preventive measures to minimize
environmental damage and prevent harm to public health.
(b) The Polluters Pay Principle, which requires the polluter to pay for the damage caused to the natural environment.
This principle aims to ensure that the costs of pollution are borne by those who cause it, rather than the public at
large.
(c) The Public Trust Principle, which holds that certain natural resources belong to the public and should be protected
for the enjoyment of the general public. This principle prohibits the transfer of public resources, such as lakes and
rivers, to private parties who may interfere with the interest of the public at large.
(d) The Principle of Inter-generational Equity, which emphasizes the need to balance the needs of the present
generation with the needs of future generations. This principle recognizes that the actions of the present generation
can impact the well-being of future generations.

Passage 16
When it comes to legally binding agreements, certain people are always considered to lack the legal ability (or
"capacity") to contract. As a legal matter, basically they are presumed not to know what they're doing. These people-
-legal minors and the mentally ill, for example--are placed into a special category. If they enter into a contract, the
agreement is considered "voidable" by them (as the person who lacked capacity to enter the agreement in the first
place). Voidable means that the person who lacked capacity to enter the contact can either end the contract or permit
it to go ahead as agreed on. This protects the party who lacks capacity from being forced to go through with a deal
that takes advantage of his or her lack of savvy.

Legal Reasoning Practice Sheet 26


CLAT POINT

Minors Have No Capacity to Contract Minors (those under the age of 18) lack the capacity to make a contract. So a
minor who signs a contract can either honor the deal or void the contract. There are a few exceptions, however. For
example, a minor cannot void a contract for necessities like food, clothing, lodging etc. In other words, the
incapacitated person will have to pay for the necessities from his/her assets. Also, a minor can void a contract for lack
of capacity only while still under the age of minority.

Mental Incapacity
A person who lacks mental capacity can void, or have a guardian void, most contracts (except contracts for necessities).
Some courts determine mental capacity based on whether the party understood the meaning and effect of the words
comprising the contract or transaction. This is called the "cognitive" test. Some courts use what's called the "affective"
test: a contract can be voided if one party is unable to act in a reasonable manner and the other party has reason to
know of the condition. And some courts use a third measure, called the "motivational" test. Courts, while applying
this, measure capacity of the person through his mental condition at the time he entered into the agreement. These
tests may produce varying results when applied to mental conditions.

Alcohol and Drugs People who are intoxicated by drugs or alcohol are usually not considered to lack capacity to
contract. Courts generally rule that those who are voluntarily intoxicated shouldn't be allowed to avoid their
contractual obligations, but should instead have to take responsibility for the results of their self-induced altered state
of mind. However, if a party is so far gone as to be unable to understand even the nature and consequences of the
agreement, and the other (sober) party takes advantage of the person's condition, then the contract may be voidable
by the inebriated party.

76. A person with a history of mental health issues and substance abuse enters into a contract to purchase a car from
a dealership. The person is heavily intoxicated at the time of signing the contract. The car turns out to be defective
and the person seeks to void the contract. Which of the following statements is true regarding the person's ability to
void the contract?
(a) The person can void the contract based on the cognitive test of mental capacity.
(b) The person cannot void the contract because the person was voluntarily intoxicated at the time of signing the
contract.
(c) The person can void the contract based on the affective test of mental capacity.
(d) The person cannot void the contract because the dealership did not take advantage of the person's condition.

77. Mr. T, who had a history of alcohol abuse, entered into a contract with Mr. P for the sale of six horses at the market
price. At the time of making the contract, Mr. T had consumed some alcohol but was still able to negotiate with Mr. P
efficiently. However, Mr. T's behavior raised concerns about his level of intoxication. Which of the following options
best reflects the validity of the contract?
(a) The contract is valid because Mr. T understood the consequences of the contract despite his level of intoxication.
(b) The contract is void because Mr. T lacked the capacity to contract due to his history of alcohol abuse.
(c) The contract is voidable because Mr. T's level of intoxication at the time of making the contract rendered him
incapable of understanding the consequences of the contract.
(d) The contract is unenforceable because Mr. P had reason to believe that Mr. T was intoxicated at the time of making
the contract.

78. Mrs. P was coerced by her husband, Mr. S, into consuming large quantities of alcohol before being asked to sign
certain contract documents that transferred the title of her paternal house to Mr. S in exchange for a small sum of
money. Which of the following options best reflects the validity of the contract?
(a) The contract is void because Mrs. P lacked the capacity to contract due to her intoxication.
(b) The contract is valid because Mrs. P signed the documents willingly, even if under duress.
(c) The contract is voidable at the option of Mrs. P because she was coerced into consuming alcohol and could not
understand the meaning of the documents.
(d) The contract is voidable at the option of Mr. S because he obtained the title of the house in exchange for money.
Ans C

Legal Reasoning Practice Sheet 27


CLAT POINT

79. A 17-year-old high school student signed a contract to purchase a car from a dealership. The student later realized
that the car was not in good condition and sought to void the contract. Which of the following statements is true
regarding the student's ability to void the contract?
(a) The student can void the contract because minors lack the capacity to contract.
(b) The student cannot void the contract because the car was a necessity.
(c) The student cannot void the contract because the student was close to turning 18.
(d) The student can void the contract only if the student's mental capacity at the time of signing the contract was
impaired.

80. Mr. X, who has a history of mental illness, enters into a contract to sell his antique watch to Mr. Y. After the contract
is signed, Mr. X's mental condition worsens, and he is unable to understand the consequences of the contract. Which
of the following options best reflects the validity of the contract?
(a) The contract is valid because Mr. X understood the consequences of the contract at the time of signing it.
(b) The contract is void because Mr. X lacked the capacity to contract due to his history of mental illness.
(c) The contract is voidable at the option of Mr. Y because Mr. X's mental condition worsened after the contract was
signed.
(d) The contract is voidable at the option of Mr. X because he was not mentally competent to enter into the contract
at the time of signing it.

Passage 17

The Constitution of India guarantees the fundamental right of freedom of speech and expression, as well as the right
to carry on any trade, business or occupation over the medium of the internet, under Article 19(1)(a) and Article
19(1)(g) respectively. However, these fundamental rights are not absolute and can be subject to reasonable
restrictions as provided under Article 19(2) and (6) of the Constitution.

In recent years, there have been instances where internet services have been suspended or restricted by the
government authorities on the grounds of maintaining law and order, national security, or public safety. The Supreme
Court of India has repeatedly emphasized that the power to restrict or suspend internet services should be used
sparingly and only in exceptional circumstances, and that any such restrictions must be in consonance with the
constitutional principles of necessity, proportionality, and the need to first look for alternate means.

In a recent case, the Supreme Court ordered the authorities in Jammu & Kashmir to immediately restore internet
services in all institutions providing essential services, including hospitals and banks. The apex court also held that the
power under Section 144 of the Code of Criminal Procedure (CrPC) cannot be used to suppress legitimate expression
of opinion or grievance or exercise of any democratic rights, and that any restrictions imposed under Section 144 must
be used as a last resort to maintain law and order.

The Supreme Court further elaborated that any restriction on the fundamental rights of citizens must be based on
objective facts and cannot be done through an arbitrary exercise of power. The magistrate's power to issue orders
under Section 144 should be based on the type of exigency, extent of territoriality, nature of restriction, and duration
of the same. The court also emphasized that in situations where fundamental rights of citizens are being curtailed, the
same cannot be done through a blanket or indefinite internet shutdown order, and that the degree of restriction and
the scope of the same must stand in relation to what is actually necessary to combat an emergent situation.

The Supreme Court's ruling on internet shutdowns and Section 144 orders has far-reaching implications for the
protection of fundamental rights in India. It signals a shift towards a more progressive and rights-based approach to
governance, where restrictions on fundamental rights are subject to rigorous scrutiny and accountability. It also
highlights the importance of striking a balance between public safety and individual freedoms, and the need to ensure
that any restrictions on fundamental rights are proportionate, necessary, and based on objective facts.

Legal Reasoning Practice Sheet 28


CLAT POINT

81. In the city of Sitapur, six individuals, X, Y, Z, A, B, and C, conducted various meetings while planning to attack and
loot police stations and banks in the city. They further intended and planned to attack and kill the Chief Minister of
the state after mobilizing people of Sitapur. Can such a meeting be disallowed under section 144 of the Criminal
Procedure Code (CrPC)?
(a) Yes, because the meeting is unlawful in nature and poses a threat to public order and safety.
(b) No, because the right to free speech and expression and freedom to assemble peacefully are fundamental rights
protected under the Constitution.
(c) Yes, because the individuals intended to use criminal force against the State government by killing the Chief
Minister, which poses a threat to public order and safety.
(d) Both a and c are correct.

82. In May 2021, the government of X State imposed a year-long internet shutdown in response to rising protests
against the State government's policies. However, there was no evidence to suggest that the protests were causing a
public order problem in the State. As a result, the speech and expression of the people of X State were severely
affected. Can such a measure be allowed under Constitutional provisions?
(a) Such a measure cannot be allowed because freedom of speech and expression is a fundamental right protected
under the Constitution, and any restriction on this right must be proportionate and justified under the Constitution.
(b) Such a measure cannot be allowed because the internet shutdown was disproportionate to the situation, and there
was no clear evidence to suggest that the protests were causing a law and order problem in the State.
(c) Such a measure can be allowed because the government has the power to take any step it deems fit to maintain
law and order in the State.
(d) Both a and b are correct.

83. In May 2021, the Government of X State imposed a 1-year long internet shutdown in response to the rising protests
against the policies of the State government. However, there was no evidence to suggest that the protests were
causing a public order problem in the State. As a result, the speech and expression of people in State X were greatly
affected. Can such a measure be allowed under Constitutional provisions?
(a) No, such a measure cannot be allowed because Freedom of Speech and Expression is a fundamental right protected
by the Constitution, and any restriction on this right must be based on objective facts, necessity, and proportionality.
(b) No, such a measure cannot be allowed because the internet shutdown was disproportionate and arbitrary, and
there was no law-and-order problem due to the protests.
(c) Yes, such a measure can be allowed because the government has the power to take any step it deems necessary
to maintain public order and safety.
(d) Both A and B are correct, as they accurately explain that such a measure cannot be allowed under Constitutional
provisions due to the protection of fundamental rights and the requirement of necessity and proportionality.

84. In the state of ABC, the government authorities have imposed an internet shutdown for an indefinite period, citing
national security concerns. However, there is no evidence to suggest that the shutdown is necessary or proportionate
to the threat posed to national security. Can the government authorities impose such an internet shutdown under the
Constitution of India?
(a) No, the government authorities cannot impose such an indefinite internet shutdown because the Supreme Court
has emphasized that any restrictions on fundamental rights must be proportionate, necessary, and based on objective
facts.
(b) Yes, the government authorities can impose such an indefinite internet shutdown because they have the power to
take any step they deem fit to maintain national security.
(c) No, the government authorities cannot impose such an indefinite internet shutdown because the fundamental
right of freedom of speech and expression is protected by the Constitution, and any restriction on this right must be
based on objective facts, necessity, and proportionality.
(d) Yes, the government authorities can impose such an indefinite internet shutdown because national security takes
precedence over individual rights and freedoms.

85. In the state of PQR, there is a widespread protest against a new law that has been passed by the government. The
authorities have imposed Section 144 of the CrPC in the area, prohibiting the assembly of more than four people and

Legal Reasoning Practice Sheet 29


CLAT POINT

the use of any form of communication. Which of the following options represents the correct implementation of
Section 144 of the CrPC and Article 19 of the Constitution in this scenario?
(a) The authorities should allow the protesters to assemble and express their opinions freely, as guaranteed by Article
19 of the Constitution.
(b) The authorities should use Section 144 of the CrPC to suppress the legitimate expression of opinion or grievance,
as this is necessary to maintain law and order.
(c) The authorities should use Section 144 of the CrPC as a last resort to maintain law and order, but the restrictions
imposed must be based on objective facts, necessity, and proportionality, as per the recent Supreme Court ruling.
(d) The authorities should impose a blanket ban on all forms of communication and assembly in the area, as this is
necessary to prevent any potential threat to public safety and order.

Passage 18

It is already known that the Indian law of torts is based on the English common law. Thus, the law relating to negligence
is adopted and modified by the courts of India on the principles of justice, equity and good conscience. The term
Negligence is derived from the Latin word negligentia, which means ‘failing to pick up’. In the general sense, the term
negligence means the act of being careless and in the legal sense, it signifies the failure to exercise a standard of care
which the doer as a reasonable man should have exercised in a particular situation. Negligence in English law emerged
as an independent cause of action only in the 18th century. Similarly in Indian law, the IPC, 1860 contained no provision
for causing the death of a person by negligence which was subsequently amended in the year 1870 by inserting section
304A.

In Blyth v. Birmingham Water Works Co, Negligence was defined as the omission to do something which a reasonable
man would do or doing something which a prudent or reasonable man would not do.

The plaintiff needs to prove that the defendant owed him a duty of care and made a breach of the same. The nature
of duty for negligence is purely legal and not moral or religious. ‘Duty’ can be seen as an obligation to be careful
towards others. When the first condition is established, the second step is to establish that there was a breach of duty.
The defendant is supposed to carry out his duty like a reasonable man. The deciding test is to see whether the
defendant has taken reasonable care or not. As a result of the breach of duty by the defendant, the plaintiff must
suffer some damage.

Contributory negligence is the ignorance of due care on the part of the plaintiff to avoid the consequences of the
defendant’s negligence. This concept is loosely based on the maxim- “Volenti non fit injuria” (injury sustained
voluntarily). It means If a person is not taking due diligence in order to avoid consequences resulting out from the
negligence of the defendant the liability of negligence will be on both of them.
In Medical Negligence, a person is expected to possess the required skill and understanding of his duties when he is in
a particular profession. Especially in the medical profession where the stakes are very high, a huge onus lies on the
practitioner to take care while choosing and administering a particular treatment for his patient. A breach of this duty
amounts to medical negligence.

86. Stark, while driving his car, ignored a red light and attempted to cross a rail track. Unfortunately, his engine stalled,
and he was stuck on the track. Steve, who was the driver of the oncoming train, saw Stark's car and would have been
able to stop the train had he not been waving to the driver of another train. The train hit the car, and Stark was injured.
Subsequently, Stark files a suit against Steve for negligence. In this situation, which of the following options is the most
accurate regarding Stark's likelihood of success in the lawsuit?
(a) Stark will succeed because Steve was negligent driving the train and is therefore liable for Stark's loss.
(b) Stark will not succeed because it was his fault that he ignored the red light and attempted to cross the road.
(c) Stark has contributed to the injury, and therefore his claim will be reduced to the extent of his own negligence.
(d) None of these options is accurate.

87. Banner went to Avengers Stores and asked the shopkeeper to give him good biscuits. The shopkeeper suggested
DIAMOND biscuits as a popular choice among customers. Banner purchased a packet of DIAMOND biscuits, and while

Legal Reasoning Practice Sheet 30


CLAT POINT

eating one, he bit into a stone and broke his tooth. In such a situation, who should Banner seek compensation for
negligence from, the shopkeeper or the DIAMOND Biscuit Company?
(a) Banner should only seek compensation from the shopkeeper as he suggested DIAMOND biscuits and therefore is
liable for any harm caused.
(b) Banner should only seek compensation from the DIAMOND Biscuit Company as they are responsible for ensuring
the quality of their products and owe a duty of care to consumers like Banner.
(c) Banner should seek compensation from both the shopkeeper and the DIAMOND Biscuit Company as they both
played a role in his injury.
(d) Banner will not be compensated as the injury caused by biting into a stone in a biscuit is trivial and insignificant.

88. Peter was walking down the street, and as he was crossing MJ's house, a bag of flour fell from the window of MJ
on Peter, causing him harm. In such a situation, Peter filed a case of negligence against MJ. Based on the legal principles
of negligence, which of the following options accurately reflects the outcome of the case?
(a) The burden of proof is on MJ to prove that she is not negligent in the incident.
(b) The burden of proof is on Peter to prove that MJ is negligent in the incident.
(c) MJ is negligent and liable for the harm caused to Peter.
(d) MJ is not negligent and not liable for the harm caused to Peter.

89. A public sector undertaking (PSU) is operating buses for its employee across the city. The buses are easily
distinguishable from public buses and carries a board saying, "Only for employees. Mr. P, however, mistakenly gets
onto the bus along with the crowd of employees. A little while later, due to the driver's negligence, there is an accident,
and Mr. P, along with some others, are injured. Can Mr. P claim damages from the PSU?
(a) It is the case of volenti non fit injuria and therefore will not get any damages.
(b) Only the employees of the company who were travelling will get damages.
(c) Everyone who was travelling in the bus will get damages.
(d) Only Mr. P will get damages.

90. 'X' carelessly left an iron pole protruding across a public road. 300 meters from that spot was a traffic signal
indicating the speed limit to be at 40 kmph. Y, riding a scooter at 50 kmph, noticed the protrusion from a distance, but
still could not avoid it, collided with the pole, and was injured. Will Y succeed in an action against X?
(a) Y will not succeed, as it was his own negligence.
(b) Y will succeed as negligence was on the part of X.
(c) Y will not succeed, as there was a contributory negligence.
(d) None of these.

Passage 19
While matrimonial law cannot take notice of every kind of physical abnormality as affording a ground for relief, there
are certain serious diseases, which it would take into account in its provisions relating to matrimonial reliefs. In Indian
law, the two diseases which figure are leprosy and venereal disease.

Under the Hindu Marriage Act, 1955, as amended in 1976, divorce is available if the other party is suffering from
‘virulent and incurable form of leprosy.’ Venereal disease in a communicable form is also a ground for relief. Under
the Special Marriage Act, 1954 also leprosy and venereal disease are grounds for divorce with a difference that in case
of leprosy, the disease need not be ‘virulent or incurable’(unlike the requirement under the Hindu Marriage Act, 1955)
though it should not have been contracted from the petitioner. The venereal disease must be communicable. The Parsi
Marriage and Divorce Act, 1936 is silent in regard to leprosy but where the defendant has infected the plaintiff with
venereal disease, the latter may file a suit for divorce. Such suit has to be filed not later than two years after knowledge
of the infection.

The Indian Divorce Act, 1869 as amended by Act 51 of 2001, also provides for virulent and incurable form of leprosy
as a ground for dissolution of marriage, provided that the respondent must have been suffering from the disease for
not less than two years immediately preceding the presentation of the petition. Venereal disease in a communicable
form is also a ground for dissolution. In this case also, the respondent must have been suffering from the disease for
not less than two years immediately preceding the petition.
Legal Reasoning Practice Sheet 31
CLAT POINT

So far as the Muslim Law is concerned, under the Dissolution of Muslim Marriages Act, 1939, a wife is entitled to a
decree of dissolution of the marriage on the ground, inter alia, that the husband is suffering from leprosy or virulent
venereal disease. Thus, we find that there is no uniformity in regard to the nature and duration of the disease, so as
to afford a ground for relief. Under the Hindu Law, the provision requires that in case of leprosy, it has to be virulent
and incurable. As regards venereal disease, the same must be shown to be of a kind, which is communicable. Under
the Special Marriage Act, 1954, the provision just refers to leprosy, without defining the nature and extent of the
disease. The Christian Law, like the Hindu Law, provides that leprosy should be virulent and incurable and venereal
disease should be communicable in order to entitle the plaintiff to relief. The Parsi Law has no reference to leprosy
and in case of venereal disease, the plaintiff should have been infected with the disease by the respondent in order to
obtain relief. A Muslim wife has simply to establish leprosy or virulent venereal diseases of the husband.

91. Mr. X, the husband of Mrs. Y, filed a petition for divorce on the grounds of his wife's alleged mental disorder and
incurable leprosy. During the proceedings, when her hands were exposed, no traces of any skin disorder could be seen.
Furthermore, she intelligently answered all queries, which disproved Mr. X's charge of mental disorder. In such a
situation, can the court grant the divorce under the Hindu Marriage Act, 1955, which is applicable to the parties?
Consider the following options:
(a) Yes, because Mrs. Y was mentally ill, and this renders her unfit for marriage.
(b) No, because there is no evidence that Mrs. Y was suffering from either mental illness or leprosy.
(c) Yes, because Mrs. Y was not fit for marriage, even if she did not have leprosy or a mental disorder.
(d) None of the above options is accurate.

92. Mr. X and Mrs. Z were a Muslim couple who got married. However, just two weeks after their wedding, Mr. Z
discovered that Mr. X was suffering from syphilis, a venereal disease. Based on the legal principles mentioned in the
passage, which of the following options is correct?
(a) There cannot be a dissolution of marriage because syphilis is not a communicable disease.
(b) There can be a dissolution of marriage because the husband is suffering from a venereal disease.
(c) The facts are insufficient to determine whether there can be a dissolution of marriage.
(d) None of the above options is accurate.

93. Mr. X, a Hindu, and Mrs. Y, a Christian, entered into a marriage contract as per the provisions of the Special
Marriage Act, 1954. In case one of them contracts a venereal disease, what are the legal options available to the other
party? Consider the following elaborated options:
(a) Divorce: The affected party can seek a divorce on the grounds of communicable venereal disease as per the
provisions of the Special Marriage Act, 1954.
(b) Maintenance: The unaffected party can seek maintenance from the affected party due to the illness contracted by
them.
(c) Restitution of conjugal rights: The unaffected party can file a petition seeking restitution of conjugal rights as a
remedy to avoid the dissolution of marriage.
(d) None of the above options is available: The affected party cannot seek any legal remedy, and the unaffected party
has no recourse to any legal remedy either.

94. Mr. and Mrs. Gupta have been married for 5 years, but Mrs. Gupta has been diagnosed with an incurable form of
leprosy. Mr. Gupta wants to seek a divorce. Can he file for divorce under the Indian Divorce Act, 1869?
(a) No, Mr. Gupta cannot file for divorce under the Indian Divorce Act, 1869 as leprosy is not a ground for dissolution
of marriage.
(b) Yes, Mr. Gupta can file for divorce under the Indian Divorce Act, 1869 as leprosy is a ground for dissolution of
marriage, provided that the respondent must have been suffering from the disease for not less than two years
immediately preceding the presentation of the petition.
(c) No, Mr. Gupta cannot file for divorce under the Indian Divorce Act, 1869 as the respondent must have been
suffering from the disease for not less than three years immediately preceding the presentation of the petition.
(d) Yes, Mr. Gupta can file for divorce under the Indian Divorce Act, 1869 as venereal disease in a communicable form
is also a ground for dissolution of marriage.

Legal Reasoning Practice Sheet 32


CLAT POINT

95. Jack and Jill got married in a church without getting their marriage registered with the government. Later, they
found out that their marriage was invalid as they did not fulfill the necessary legal requirements for marriage in their
jurisdiction. Can Jack still seek a divorce on the grounds of virulent and incurable form of leprosy or venereal disease
as provided under the Indian Divorce Act, 1869?
(a) Yes, Jack can still seek a divorce on the grounds of leprosy or venereal disease under the Indian Divorce Act, 1869.
(b) No, Jack cannot seek a divorce on the grounds of leprosy or venereal disease under the Indian Divorce Act, 1869.
(c) Jack cannot only seek a divorce on the grounds of invalidity of his marriage and not on any other grounds.
(d) The Indian Divorce Act, 1869 is applicable in this case.

Passage 20
Fault or ground-based matrimonial litigation is time-consuming and expensive. It also involves a lot of mud-slinging
thereby further embittering the relationships and thwarting prospects of amicable resolution of ancillary issues like
maintenance, child-custody/visitation and, so on.

Prior to 1976, the only Indian statute which had a provision of divorce by mutual consent was the Special Marriage
Act, 1954. Persons married or registered under the Act could get their marriage dissolved by mutual consent. Legal
Position under Special Marriage Act, 1954 and Hindu Marriage Act, 1955-

Special Marriage Act, 1954


Section 28 of the Act reads as under: Section 34(1)(c) further provides that when divorce is sought on the ground of
mutual consent, the court has to satisfy itself that such consent has not been obtained by force, fraud or undue
influence. In order to get relief under the provisions of this Act, it is mandatory that the marriage should either have
been performed under the provisions of this Act, or registered thereunder. Thus, in Reynold Rajamani v. Union of
India, where the parties were married under the Indian Christian Marriage Act, 1872, and their petition for divorce by
mutual consent under s. Section 28 of the Special Marriage Act, 1954 was dismissed, the court held that the ground
could not be availed of by the parties, as they were governed by the Indian Divorce Act, 1869 which did not provide
for divorce by mutual consent.

Hindu Marriage Act, 1955


The ground of divorce by mutual consent was inserted in the Hindu Marriage Act, 1955 by an Amendment in 1976, by
adding s. 13B. The section states: (i) The requirements which have to be complied with under this provision thus are:
the parties have been living separately for a period of at least one year; they have not been able to live together; and
they have mutually agreed to have the marriage dissolved. At the outset it may be pointed out that a petition under
this section can be filed only by the spouses. When both parties are well educated and mature, the father of either
party has no locus standi or right to be impleaded as a party in such matrimonial petitions unless the spouses or any
of them is a minor or of unsound mind.

96. Mr. X and Mrs. Y, a couple whose marriage was governed by the provisions of the Special Marriage Act, 1954,
approached the court for a divorce by mutual consent. However, during the proceedings, it came to light that Mrs. Y
was being coerced by Mr. X to agree to the divorce. In light of this, can the court grant the divorce to the couple?
(a) Yes, the court can grant the divorce as long as both parties have mutually agreed to it.
(b) No, the court cannot grant the divorce as Mrs. Y's consent was obtained by force.
(c) The facts are insufficient to make a decision on whether the court can grant the divorce or not.
(d) None of the above.

97. Mr. Chris and Mrs. Emily, a Christian couple, have decided to seek a divorce by mutual consent after being married
for 10 years. However, their marriage was governed by the Indian Divorce Act, 1869. In light of this, can they be granted
a divorce by the court?
(a) No, the Indian Divorce Act, 1869 does not provide for divorce by mutual consent.
(b) Yes, the court can grant the divorce as long as both parties have mutually agreed to it.
(c) The facts are insufficient to make a decision on whether the court can grant the divorce or not.
(d) None of the above.

Legal Reasoning Practice Sheet 33


CLAT POINT

98. A Hindu couple, married for 6 months, have decided to seek a divorce. Can they be granted a divorce under the
Hindu Marriage Act, 1955?
(a) No, because the parties must be living separately for a period of at least one year.
(b) No, because the Hindu Marriage Act does not provide for divorce by mutual consent.
(c) Yes, because they have completed all the requisites for a divorce by mutual consent.
(d) None of the above.

99. Can a couple married under the Indian Divorce Act, 1869, seek a divorce by mutual consent under the Special
Marriage Act, 1954?
(a) No, they cannot seek a divorce by mutual consent under the Special Marriage Act, 1954 since their marriage was
not performed under the provisions of that Act.
(b) Yes, they can seek a divorce by mutual consent under the Special Marriage Act, 1954 even if their marriage was
not performed under the provisions of that Act.
(c) The court can only grant a divorce by mutual consent if the parties were married under the Hindu Marriage Act,
1955.
(d) The court can only grant a divorce by mutual consent if the parties were married under the Indian Christian
Marriage Act, 1872.

100. Mr. A and Mrs. B were married under the Hindu Marriage Act, 1955, and have been living separately for the past
6 months. They wish to seek a divorce by mutual consent under the Special Marriage Act, 1954. Can the court grant
them a divorce?
(a) Yes, the court can grant them a divorce by mutual consent under the Special Marriage Act, 1954 since it provides
for such a provision.
(b) No, the court cannot grant them a divorce by mutual consent under the Special Marriage Act, 1954 since they were
married under the Hindu Marriage Act, 1955 and it does not provide for divorce by mutual consent.
(c) The court can grant them a divorce by mutual consent under the Special Marriage Act, 1954 but only if they were
married under the Act or registered thereunder.
(d) The court cannot grant them a divorce by mutual consent under the Special Marriage Act, 1954 since the provision
for divorce by mutual consent was inserted only in 1976 and cannot have a retrospective effect.

Passage 21
Section 2 (b) of the Indian Contract Act, 1872 defines acceptance as follows: When the person to whom the proposal
is made signifies his assent thereto, the proposal is said to be accepted. A proposal, when accepted, becomes a
promise. Thus "acceptance" is the assent given to a proposal, and it has the effect of converting the proposal into a
promise. The definition clearly requires that the assent should be signified. It may be signified or expressed by an act
or omission by which the party accepting intends to communicate his assent or which has the effect of communicating
it. [S. 3] A very common instance of an act amounting to acceptance is the fall of the hammer in the case of an auction
sale. The principle is that there should be some external manifestation [overt act) of acceptance. A mere mental
determination to accept unaccompanied by any external indication will not be sufficient. Such manifestation may be
in the form of express words, written or spoken or may be signified through conduct. A common example of
acceptance by conduct is an action in terms of the offer. All cases of general offers, which are a kind of unilateral
contract, demand some act in return for the promise to pay. In express recognition of this principle Section 8 provides
that "performance of the conditions of a proposal, of the acceptance of any consideration for a reciprocal promise
which may be offered with a proposal, is an acceptance of the proposal". Such proposals demand acceptance by
performance. Acceptance of money after prior information that higher rates of interest would be charged and
retention of goods sent on approval amount to acceptance of the consideration offered along with a proposal. A
Canadian court has gone as far as to hold that, where an offer was made for the sale of a part of a land and lease of
the other part subject to the condition that the offeree obtained planning permission and the offeree obtained
permission within nine months, a valid contract arose, though no communication of acceptance had been made during
the period of nine months. Further, acceptance must be communicated to the offeror himself. A communication to
any other person is as ineffectual as if no communication has been made. The natural corollary of this principle is that
the communication of acceptance should be from a person who has the authority to accept. Information received
from an unauthorized person is ineffective.

Legal Reasoning Practice Sheet 34


CLAT POINT

101. B had been supplying coal to a railway company without any formal agreement. B suggested that a formal
agreement should be drawn up. The agents of both the parties met and drew up a draft agreement. It had some blanks
when it was sent to B for his approval. He filled up the blanks including the name of an arbitrator and then returned it
to the company. The agent of the company put the draft in his drawer and it remained there without final approval
having been signified. B kept up his supply of coals but on the new terms and also received payment on the new terms.
A dispute having arisen B refused to be bound by the agreement.
(a) There was no external manifestation of acceptance on the part of agents as there was no reply given by them.
(b) B is bound by the agreement because there was acceptance by conduct is an action in terms of the offer by B.
(c) There is no agreement between the parties as there was no acceptance on the part of B.
(d) Both a) and c)

102. The plaintiff offered by means of a letter to purchase his nephew's mare. The letter said: "If I hear no more about
the horse, I consider the horse mine at £20." To this letter no reply was sent. But the nephew told the defendant, his
auctioneer, not to sell the mare as it was already sold to his uncle. The auctioneer by mistake put up the mare for
auction and sold it. The plaintiff sued the auctioneer on the ground that under the contract the mare had become his
property and, therefore, defendant's unauthorized sale amounted to conversion.
(a) There was conversion as the auctioneer had wrongfully sold the property of the plaintiff.
(b) There was conversion because there was a legally binding contract between the parties.
(c) There was conversion because the nephew had already conveyed his acceptance of his offer to his uncle by not
replying to the letter.
(d) There was no conversion as there was no communication of acceptance from nephew to uncle.

103. In a case, the plaintiff applied for the position of a professor in a university and the managers passed a resolution
appointing him. However, the decision was not communicated to him officially. One of the members of the board
informed him about the decision in his individual capacity. The managers later cancelled their resolution and the
plaintiff sued for breach of contract. Can it be considered as a breach of contract on the part of the managers?
(a) Yes, since the member of the board communicated the decision of acceptance, there was a breach of contract on
the part of the managers of the resolution.
(b) No, since the member of the board communicated the decision of acceptance only in his individual capacity and
not as a formal communication, there was no valid acceptance by the managers.
(c) There was a formal communication of acceptance on the part of the managers, but it was later cancelled due to
some reason.
(d) Both A) and C)

104. According to the passage, which of the following is untrue?


(a) Any person can communicate acceptance of an offer as long as the acceptance is communicated to the other party
or the offeror.
(b) Acceptance must be communicated to the offeror himself for it to be valid.
(c) Only person who is authorized to communicate acceptance can do so legally
(d) All of the option are true according to the passage.

105. John, the manager of a company, sent an offer to a potential vendor to supply raw materials for their production.
The vendor's assistant received the offer and immediately responded with acceptance via email. Later, John
discovered that the assistant did not have the authority to accept the offer. Is the acceptance valid?
(a) Yes, the acceptance is valid because it was communicated through email.
(b) No, the acceptance is not valid because it was not communicated to the offeror himself.
(c) No, the acceptance is not valid because it was communicated by an unauthorized person.
(d) Yes, the acceptance is valid because the offer was sent to the vendor's company, and the assistant acted on behalf
of the company.

Passage 22
Generally, a person is liable for his own wrongful acts and one does not incur any liability for the acts done by others.
In certain cases, however, vicarious liability, that is the liability of one person for the act done by another person, may
arise. In order that the liability of A for the act done by B can arise, it is necessary that there should be a certain kind
Legal Reasoning Practice Sheet 35
CLAT POINT

of relationship between A and B, and the wrongful act should be, in a certain way, connected with that relationship.
The common examples of such vicarious liability are : (1) Liability of the principal for the tort of his agent; (2) Liability
of partners of each other's tort; (3) Liability of the master for the tort of his servant. When an agent commits a tort in
the course of performance of his duty as an agent, the liability of the principal arises for such a wrongful act. The agent
is liable because he has done the wrongful act. The principal is liable vicariously because of the principal-agent
relationship between the two. Both can be made liable for the same wrongful act. They are considered to be joint
tortfeasors and their liability is joint and several. In such a case, the plaintiff has a choice either to sue the principal, or
the agent, or both of them.

Similarly, when the wrongful act is done by one partner in the ordinary course of the business of the firm, all the other
partners are vicariously liable for the same. All the partners of the firm, i.e., the guilty partner and the others are
considered to be joint tortfeasors. The same rule applies in the case of master-servant relationship. The master is
vicariously liable for the wrongful act .done by his servant in the course of employment. The liability of the master, of
course, is in addition to that of the servant. Where one person authorizes another to commit a tort, the liability for
that will be not only of that person who has committed it but also of that who authorized it. It is based on the general
principle "Qui facit per alium facit per se" which means that "the act of an agent is the act of the principal." For any
act authorized by the principal and done by the agent, both of them are liable. Their liability is joint and several. The
authority to do the act may be express or implied. The principal generally does not expressly ask his agent to do the
wrongful act, but when the agent acts in the ordinary course of the performance of his duties as an agent, the principal
becomes liable for the same.

106. Mrs X, who owned two apartments but was not satisfied with the income therefrom, approached the office of
XYZ and Partners a firm of solicitors, to consult them about the matter of her property. The principal clerk of the
company attended her and advised her to mortgage the two apartments and invest the money in gold. She was asked
to sign two documents, which were supposed to be mortgage deeds. In fact, the documents got signed were gift deeds
in the name of the principal clerk himself. He then disposed of the property and misappropriated the proceeds. He
had acted solely for his personal benefit and without the knowledge of his principal. Can XYZ Partners be held for liable
for fraud as principals?
(a) Since there was no official communication from the XYZ Partners asking her to mortgage the land, there is no
vicariously liability in this case.
(b) The principal clerk had acted in his personal capacity in this case.
(c) Since the principal clerk acted in ostensible authority and as the agent of XYZ Partners, there hare liable for fraud.
(d) Both a) and b)

107. The plaintiff's husband gave some amount and cheques to his friend, who was an employee in the defendant
bank, for being deposited in the plaintiff's account. No proper receipt for the deposits was obtained. The bank
employee misappropriated the amount. Can the bank be held liable vicariously?
(a) The bank can be held vicariously liable since the employee had acted as the agent of the bank.
(b) The bank cannot be held vicariously liable because the employee had acted in his personal capacity.
(c) The bank is liable because the employee had been given money to deposit it. Hence the employee was deemed to
be an agent of the bank.
(d) Both a) and c)

108. Mr. Z, the owner of the motor cycle was outside the country, his younger brother Y took the motor cycle without
his knowledge or permission and caused the accident. Can Mr. Z be held vicariously liable for the accident?
(a) Mr Z can be held vicariously liable for the accident because it was his bike that caused it.
(b) Mr Z cannot be held vicariously liable for the accident because Y could not be deemed to be his agent because he
never asked for his permission.
(c) Mr Z can be held liable for the accident because he had given permission to Y to ride his motorcycle.
(d) None of the above.

109. Mr. A, a businessman, hires Mr. B as his agent to sell his goods. While selling the goods, Mr. B makes a false
representation about the quality of the goods to Mr. C, a customer. Who is liable for Mr. B's false representation?
(a) Mr. B is solely liable for his false representation since he made it.
Legal Reasoning Practice Sheet 36
CLAT POINT

(b) Mr. A is solely liable for Mr. B's false representation since Mr. B was acting as his agent.
(c) Both Mr. A and Mr. B are jointly and severally liable for Mr. B's false representation.
(d) Neither Mr. A nor Mr. B is liable for Mr. B's false representation since it was made to a third party.

110. Mr. A and Mr. B are partners in a law firm. While Mr. A was representing a client in court, he made a false
statement to the judge, which resulted in the client losing the case. Can Mr. B be held liable for Mr. A's false statement?
(a) No, since Mr. A made the false statement in the course of his individual capacity as a lawyer, Mr. B cannot be held
liable for it.
(b) Yes, since Mr. A and Mr. B are partners in the law firm, they can be held liable for each other's torts.
(c) Only Mr. A can be held liable for his false statement since he made it in the course of representing the client.
(d) Both Mr. A and Mr. B can be held liable for the false statement if it can be proved that Mr. B was aware of the false
statement made by Mr. A.

Passage 23
When the plaintiff brings an action against the defendant for a particular tort, providing the existence of all the
essentials of that tort, the defendant would be liable for the same. The defendant may, however, even in such a case,
avoid his liability by taking the plea of some defence. There are some specific defences, which are peculiar to some
particular wrongs, for example, in an action for defamation, the defences of privilege, fair comment or justification
are available. There are some general defences which may be taken against action for number of wrongs. When a
person consents to the infliction of some harm upon himself, he has no remedy for that in tort.1 In case, the plaintiff
voluntarily agrees to suffer some harm, he is not allowed to complain for that and his consent serves as a good defence
against him. No man can enforce a right which he has voluntarily waived or abandoned. Consent to suffer the harm
may be express or implied. Many a time, the consent may be implied or inferred from the conduct of the parties. For
example, a player in the games of cricket or football is deemed to be agreeing to any hurt which may be likely in the
normal course of the game. Similarly, a person going on a highway is presumed to consent to the risk of pure accidents.
In the same way, a spectator at a cricket match or a motor race cannot recover if he is hit by the ball or injured by a
car coming on the track. For the defence of consent to be available, the act causing the harm must not go beyond the
limit of what has been consented. Also, it is necessary to show that the plaintiff's consent to the act done by the
defendant was free. If the consent of the plaintiff has been obtained by fraud or under compulsion or under some
mistaken impression, such consent does not serve as a good defence.

111. The plaintiff was a spectator at a motor car race being held on a track owned by the defendant company. During
the race, there was a collision between two cars, one of which was thrown among the spectators, thereby injuring the
plaintiff. Decide.
(a) Since the collision between the cars was a danger inherent in the car race, the plaintiff took the implied risk. Hence
the defendants can take the defence of volenti non fit injuria.
(b) Since the collision between the cars was a danger unusual for a car race, the plaintiff nevr took the implied risk of
a collision. Hence the defendants cannot take the defence of volenti non fit injuria.
(c) The defendants are liable because they should have ensured no one should be injured even in case there is a
collision.
(d) None of the above.

112. Mr. A was taking the jeep for filling petrol in the tank, two strangers, Mr X and Mr Y took lift in the jeep. Suddenly
one of the bolts fixing the left back wheel to the axle gave way toppling the jeep. The two strangers were thrown out
and sustained injuries, and one of them, Mr Y died as a consequence of the same. Can Mr A take the defence of volenti
non fit injuria?
(a) Mr A cannot take the defence of volenti non fit injuria because X and Y never expected the bolt in the front axle to
give way and cause the jeep to be toppled.
(b) Mr A cannot take the defence of volenti non fit injuria as the accident was caused by his negligence in maintaining
the jeep.
(c) Mr A can take the defence of volenti non fit injuria because they got into the jeep into the jeep voluntarily and it
was a case of sheer accident.
(d) Both a) and b)
Legal Reasoning Practice Sheet 37
CLAT POINT

113. The plaintiff, who was a photographer, was taking photographs at a horse show while he was standing at the
boundary of the arena. One of the horses, belonging to the defendant, rounded the bend too fast. As the horse
galloped furiously, the plaintiff was frightened and he fell into the horses' course and there he was seriously injured
by the galloping horse. The horse in question won the competition.
(a) There was no fault of defendants, hence they are not liable.
(b) The plaintiff assumed the risk of standing too close to the boundary of arena. Hence the defendants can take the
defence of volenti non fit injuria.
(c) The defendants are liable because the horse caused the photographer to be frightened and fall onto the tracks.
(d) Both a) and b)

114. Plaintiff, an employee in the defendant's brewery, was trying to remove a lid from a boiling vat. The lid was stuck
and by the plaintiff's extra pull to it, it came off suddenly and the plaintiff fell back into the cooling vat which contained
scalding liquid. The plaintiff was severely injured.
(a) The defendant can take the defence of volenti non fit injuria because the plaintiff had taken the risk associated
with the task.
(b) The defendant is liable because the lid came off unexpectedly and injured the plaintiff.
(c) The defendant is liable because the plaintiff did not know the lid was stuck.
(d) None of the above.

115. The plaintiff, a married woman, aged 40 years, noticed development of a painful lump in her breast. The lump
had no effect on her uterus, but during surgery, her uterus was removed without any justification. The defendant
hospital claimed the defence of volenti non fit injuria.
(a) The defendant hospital can claim the defence of volenti non fit injuria because the plaintiff had given the
permission for surgery.
(b) The defendant hospital cannot the defence of volenti non fit injuria because the plaintiff did not consent to removal
of her uterus.
(c) The defendant hospital can claim the defence of volenti non fit injuria because the uterus had to be removed
according to the procedure.
(d) None of the above.

Passage 24
It is often seen that one wants to protect one’s body and property, whether movable or immovable. People usually
seem anxious as to their person or property being vulnerable to negative elements, who are willing to misappropriate
and exploit their wealth with a Mala Fide intention. This apprehension of large sections is what is required to be dealt
with the iron hands of the law.
The law takes care of the acts which are to be recuperated with compensation or punishment. These acts can lie from
a mere intentional touch to one’s person with an evil intention of intrusion into one’s property without assigning any
reason for the action.

The law is often applied and has evolved to strike a delicate balance between the private rights to the exclusion of
others and the socially valuable public and private interests that are sometimes served by permitting unauthorized
instances of access. Therefore, it becomes extremely necessary to identify the precise problem and its solution.

Public interest often trumps the private interest and is widely recognized by law to be the distinctive exception to the
owner’s “Right to exclude.” Trespass is a varied topic involving both civil and criminal elements. What distinguishes
criminal trespass from civil trespass is that in the former, the entry should be with intent to commit an offence or to
intimidate, insult or annoy the person in possession of the property. Trespass is both a civil and criminal wrong because
it can cause injury, i.e., violation of legal rights as well as damage to one’s person and property substantially if a physical
attack takes place. Trespass law is commonly presented as a relatively straightforward doctrine that protects
landowners against intrusions by opportunistic trespassers

Legal Reasoning Practice Sheet 38


CLAT POINT

116. Peter and Sam had an argument, after which Peter pulled out a revolver and pointed it at Sam. He said, "I will
shoot you" while Sam stood there calmly, thinking that the revolver had no bullets. Later, it was found that the revolver
did have bullets. Can Sam succeed in a claim for damages flowing from the tort of assault?
(a) Yes, Sam can succeed because Peter pointed a revolver at him, which constitutes assault regardless of whether
Sam was scared or not.
(b) No, Sam cannot succeed because he did not have any apprehension of use of force in his mind, even if it was due
to a mistaken understanding.
(c) Sam may succeed because Peter's actions could still be seen as threatening and could have caused emotional
distress to Sam.
(d) None of these

117. A, the fiancée of B, entered into a room and had a heated argument with B. B's mother, who always disapproved
of A, filed a case of battery against A. The mother claimed that A trespassed on B's personal space by making physical
contact during the argument. Will she succeed in her case?
(a) No, she will not succeed as battery requires trespass to the person, and there is no indication that A made physical
contact with B's mother.
(b) Yes, she will succeed as A made physical contact with B during the argument, thereby committing trespass to
person against B's mother's wishes.
(c) No, she will not succeed as being B's mother does not give her the right to file a case for battery against A.
(d) Yes, she will succeed as trespass to person is borne out of the facts of the case, and A made physical contact with
B during the argument, which constitutes battery.

118. Jack saw Jone's new car and couldn't resist getting inside to take a closer look, even though he did not have Jone's
permission. Jone filed a lawsuit against Jack for trespass. Will Jone succeed in his lawsuit?
(a) Yes, Jone will succeed, as Jack entered Jone's personal property without his permission.
(b) No, Jone will not succeed, as trespass only applies to immovable property, and a car is a movable property.
(c) Yes, Jone will succeed, as Jack had no valid reason for entering Jone's car without permission.
(d) Yes, Jone will succeed, as Jack clearly committed a wrong by entering someone else's property without permission.

119. A was a prisoner who fell ill while sleeping inside the jail. He was taken to the hospital for treatment but was
chained to a window throughout his stay in the hospital. Is this considered trespass to person?
(a) Yes, it is considered trespass to person as A's physical freedom was restrained without his consent.
(b) No, it is not considered trespass to person as there is an exception in the law that allows for public interest to
trump private interest, especially in cases involving prisoners.
(c) It depends on whether A wants to take action against the jail authorities for restricting his physical freedom while
he was receiving medical treatment.
(d) None of these options provide a clear answer as the situation is complex and requires further legal analysis.

120. A group of environmentalists entered a privately owned forest to protest against deforestation activities. The
owner of the forest filed a case of trespass against the environmentalists. Will the owner succeed in his case?
(a) Yes, the owner will succeed as the environmentalists trespassed onto the owner's private property without
permission.
(b) No, the owner will not succeed as public interest in protecting the environment trumps the owner's private interest
in excluding others from his property.
(c) Yes, the owner will succeed as the environmentalists violated the owner's "Right to exclude" as the owner has the
right to control access to his property.
(d) No, the owner will not succeed as the environmentalists had a valid reason to enter the forest and protest against
deforestation activities.

Passage 25
According to S. 16 of the Indian Contract Act, 1872, "Undue influence" defined.—(1) A contract is said to be induced
by "undue influence" where the relations subsisting between the parties are such that one of the parties is in a position
to dominate the will of the other and uses that position to obtain an unfair advantage over the other.
Legal Reasoning Practice Sheet 39
CLAT POINT

(2) In particular and without prejudice to the generality of the foregoing principle, a person is deemed to be in a
position to dominate the will of another— [a] where he holds a real or apparent authority over the other or where he
stands in a fiduciary relation to the other; or [b]where he makes a contract with a person whose mental capacity is
temporarily or permanently affected by reason of age, illness, or mental or bodily distress.

(3)Where a person who is in a position to dominate the will of another, enters into a contract with him, and the
transaction appears, on the face of it or on evidence adduced, to be unconscionable, the burden of proving that such
contract was not induced by undue influence shall lie upon the person in a position to dominate the will of the other.

Sometimes the parties to an agreement are so related to each other that one of them is able to dominate the will of
the other. The person who occupies the superior position may prevail upon the other to obtain his consent to an
agreement to which he, but for the influence so exerted, would not have consented. The relationship between the
parties so as to enable one of them to dominate the will of the other is a sina qua non for undue influence to come
into play. Undue influence is said to be a subtle species of fraud whereby mastery is obtained over the mind of the
victim, by insidious approaches and seductive artifices.

Sometimes the result is brought about by fear, coercion, importunity or other domination, calculated to prevent
expression of the victim's true mind. It is a constraint undermining free agency, overcoming the powers of resistance,
bringing about a submission of the other

[Source: Extracted with edits and revisions from Elaboration On Undue influence Under Indian Contract Act, 1872,
https://blog.ipleaders.in/undue-influence/]

121. Mr. X, a spiritual adviser (guru), has several followers who attend his meditation retreats, including celebrities
and politicians. He induces one of his devotees, Mr. Y, to gift him the whole of his property, claiming that this act will
secure spiritual benefits for Mr. Y's soul in the next world. Mr. Y, who is an educated businessman and has experienced
some life-changing events after attending Mr. X's retreats, genuinely believes in the spiritual benefits promised by Mr.
X and gifts him his entire property. Under these circumstances, can such inducement be categorized as undue
influence?

(a) This is not a case of undue influence because Mr. Y is a well-educated businessman who has experienced
positive life changes after attending Mr. X's retreats, suggesting that Mr. Y was not coerced into giving away his
property.
(b) Since Mr. Y is the devotee of Mr. X and trusts him as his spiritual guide, Mr. X is in a position to dominate the
will of Mr. Y to his advantage, and therefore, it constitutes undue influence.
(c) Since believing in spiritual benefits is based on personal faith and is not necessarily reasonable in the eyes of
the law, this is not a case of undue influence.
(d) This case does not qualify as undue influence as Mr. Y and Mr. X have a contractual agreement that includes a
clause allowing Mr. Y to reclaim his property under specific circumstances.

122. A, a wealthy entrepreneur, having advanced money to his son, B, during his minority to support his education
and hobbies, sees B grow up to be an accomplished young man. Upon B's coming of age, A decides to host a lavish
party to celebrate. During the party, A takes B into his study and, by misusing his parental influence and emotional
connection, obtains a bond from B for a greater amount than the sum due in respect of the advance. B, feeling obliged
and emotionally influenced, agrees to sign the bond. Has A exercised undue influence?

(a) This is a case of undue influence since A was the parent of B and used their close emotional relationship and
the celebratory context to manipulate B into signing the bond for a greater amount.
(b) This is not a case of undue influence because once B came of age, he was no longer under the legal authority
of his father and therefore should have been able to resist his father's influence.
(c) A has not exercised undue influence because B, as an accomplished young man, should have been capable of
understanding the consequences of signing the bond and making an informed decision.
Legal Reasoning Practice Sheet 40
CLAT POINT

(d) A's actions may be considered morally wrong, but they do not fall under the category of undue influence, as B
was not forced into signing the bond and did so voluntarily.

123. A, a successful small business owner, finds himself facing unexpected expenses due to unforeseen circumstances,
and thus applies to a local banker for a loan at a time when there is stringency in the money market. The banker, who
is well-acquainted with A's business and the local economy, declines to make the loan except at an unusually high rate
of interest, citing the unfavorable market conditions and A's pressing need for the funds. A, after carefully considering
other financing options and weighing the pros and cons, reluctantly accepts the loan on these terms. Is the contract
between A and the banker induced by undue influence?

(a) Since this is a transaction in the ordinary course of business and both parties had the opportunity to evaluate
the terms, the contract is not induced by undue influence.
(b) The banker took advantage of A's urgent need for funds and the tight money market to impose an unusually
high rate of interest, which constitutes undue influence.
(c) A's pressing need for the funds and the absence of viable alternatives for financing puts A in a vulnerable
position, making this a case of undue influence.
(d) The local banker's familiarity with A's business and the local economy allowed the banker to manipulate the
terms in their favor, which constitutes undue influence.

124. A, a small farmer in a remote village, finds himself struggling with debt due to a prolonged drought. In
desperation, he approaches B, the village moneylender who is known for his high interest rates and strict enforcement
of repayment terms. A contracts a fresh loan from B to settle his existing debt, agreeing to terms that appear to be
unconscionable, including extremely high interest rates and the forfeiture of his ancestral land if he fails to repay the
loan within a short period. In light of these circumstances, which of the following statements is true in respect of the
legal principle in the given paragraph?

(a) A must prove that the loan terms were unconscionable by providing evidence of the market rates and
conditions.
(b) A has to establish that B was in a position to exercise undue influence on him by demonstrating the power
dynamics in the village.
(c) B has the burden of proving that he was not in a position to exercise undue influence on A, and that the contract
was freely and fairly entered into.
(d) None of the above, as A willingly entered into the contract and must bear the consequences of his decisions.

125. Mr. Y, a highly talented artist, is going through financial difficulties due to his inability to sell his artwork in the
local market. Mr. Z, a well-known art dealer in town, offers to purchase Mr. Y's entire collection at a price lower than
its true worth. Although Mr. Y is aware of the lower offer and that Mr. Z could potentially profit more from selling the
artwork, he chooses to accept the deal as it would immediately alleviate his financial troubles. Later, Mr. Y finds out
that Mr. Z sold the art collection at a significantly higher price and feels that he was unfairly treated. Mr. Y decides to
challenge the contract in court, claiming undue influence. Based on the legal principles mentioned in the passage,
which of the following is the most likely outcome of the case?

(a) The court will consider the contract void, as Mr. Z took advantage of Mr. Y's financial difficulties to make a
profit.
(b) The court will require Mr. Y to prove that Mr. Z exercised undue influence by demonstrating that Mr. Z
dominated his will during the negotiation.
(c) The court will place the burden on Mr. Z to prove that the contract was not induced by undue influence, given
the apparent disparity in the transaction.
(d) The court will consider the contract valid, as there is no indication that Mr. Z was in a position to dominate the
will of Mr. Y or used his position to obtain an unfair advantage over him.

Legal Reasoning Practice Sheet 41


CLAT POINT

Passage 26
The provisions of the Contract Act relevant to a study of the effect of "mistake" upon a contract that the parties have
purported to make may be noted first. Mistake may operate upon a contract in two ways. It may, firstly, defeat the
consent altogether that the parties are supposed to have given, that is to say, the consent is unreal. Secondly, the
mistake may mislead the parties as to the purpose which they contemplated. The cases in which the consent is
defeated or is rendered unreal fall under Section 13 which defines "consent" as follows: Two or more persons are said
to consent when they agree upon the same thing in the same sense. An agreement upon the same thing in the same
sense is known as true consent or consensus ad idem, and is at the root of every contract.

This seems to have been picked up from the judgment of Lord Hannen in Smith v Hughes. "It is essential to the creation
of a contract that both parties should agree to the same thing in the same sense. Thus, if two persons enter into an
apparent contract concerning a particular person or ship, and it turns out that each of them, misled by a similarity of
name, had a different person or ship in mind, no contract would exist between them: Raffles v Wichelhaus". Where
the mistake does not defeat consent, but only misleads the parties. Section 20 of the Indian Contract Act shall apply.
It provides: Where both the parties to an agreement are under a mistake as to a matter of fact essential to the
agreement, the agreement is void.

Speaking broadly, certain facts are essential to every agreement. They are: (1) The identity of the parties; (2) the
identity and nature of the subject-matter of the contract; and (3) the nature and content of the promise itself.

Mistake as to identity occurs where one of the parties represents himself to be some person other than he really is. In
the case of Boulton v Jones it was held that “ the rule of law is clear, that if you propose to make a contract with A,
then B cannot substitute himself for A without your consent and to your disadvantage, securing to himself all the
benefit of the contract." Similarly, Bramwell B said: "When anyone makes a contract in which the personality, so to
speak, of the particular party contracted with is important, for any reason, whether because it is to write a book or
paint a picture or do any work of personal skill, or whether because there is a set-off due from that party, no one else
is at liberty to step in and maintain that he is the party contracted with."

[Source: Extracted with edits and revision from Validity of a contract, voidable contract and void...

https://www.scconline.com/blog/post/2021/01/30/validity-of-a-contract-voidable-contract-and-void-agreement-as-
given-under-indian-contract-act-1872/]

126. A, an art collector, agrees to purchase a painting from B, an art dealer, which both parties believe to be an original
piece by a famous artist. The painting was in the possession of C, a friend of B, who lent the painting for an exhibition.
A and B discuss the artwork's provenance, and A is shown a certificate of authenticity that B claims was issued by an
art expert. However, after the contract is made and A takes possession of the painting, it is discovered that the painting
is actually a skillful forgery. Neither A nor B were aware of this at the time of the agreement. In light of the facts above,
which of the following options is most suitable?

(a) There is a mistake as to the nature of the subject-matter of the contract.


(b) The agreement is not void because both parties were mistaken, and A relied on the certificate of authenticity.
(c) There is no mistake involved in this contract, as the certificate of authenticity was issued by an art expert.
(d) The mistake is not essential to the agreement, as both parties had the same understanding of the painting's
authenticity.

Legal Reasoning Practice Sheet 42


CLAT POINT

127. The plaintiffs, intending to purchase high-quality textiles from a reputable company, Stellar Fabrics Inc., visited
their office and placed an order with a person who claimed to be a partner in the firm. However, the individual was
neither a member nor an agent of the firm. He informed the plaintiffs that the goods should be billed under the name
Regal Textiles Corp. After receiving the goods, he transported them to another location and sold them to the
defendant, a buyer in good faith. The plaintiffs sued the defendant to recover their goods. In light of the facts above,
which of the following options is most suitable?

(a) The contract was valid, as there was no mistake in the identity of the parties.
(b) There was no valid contract, as there was a mistake of identity in the contract.
(c) The contract is voidable at the option of the plaintiffs since there was a mistake of identity.
(d) The plaintiffs have no legal remedy, as the defendant was a bona fide buyer.

128. The plaintiffs, a textile company, received an order in writing for a large consignment of handkerchiefs from a
fraudulent individual named James. The order included a printed heading that read: "James & Co, 37 Wood Street."
Coincidentally, there was a reputable firm named Jamie & Co located on the same street. Believing that the order
came from Jamie & Co, the plaintiffs shipped the handkerchiefs to the address specified. James received the goods,
sold them to the defendants, who acted in good faith. In this situation, was there a contract between the plaintiffs and
James?

(a) The contract was void due to a mistake as to the identity, as the plaintiffs intended to contract with Jamie &
Co.
(b) There was a valid contract as the plaintiffs should have verified the intended recipient before shipping the
handkerchiefs; thus, there was no mistake as to the identity.
(c) The contract was voidable at the option of James.
(d) The contract was valid, and the plaintiffs can sue James for breach of contract.

129. An artist, Ms. A, is commissioned by Mr. B to paint a portrait of Mr. B's deceased father. However, due to a mix-
up in communication, Ms. A paints a portrait of Mr. B's uncle, who has a similar name to Mr. B's father. Upon receiving
the portrait, Mr. B realizes the mistake and refuses to pay Ms. A. Can Mr. B claim that the contract is void due to a
mistake in the subject-matter?

(a) The contract is void because there was a mistake in the subject-matter, which is essential to the agreement.
(b) The contract is valid because both parties agreed upon the terms of the contract.
(c) The contract is void because Ms. A failed to deliver the expected work.
(d) The contract is valid because the mistake in the subject-matter was not significant enough to void the contract.

130. An author, Mr. X, approaches a publishing company, "Creative Publishers," to get his book published. The
representative from the publishing company, Mr. Y, finalizes the contract with Mr. X but uses the name "Innovative
Publishers" by mistake. Mr. X also doesn't notice the error and signs the contract. Later, Mr. X realizes that the
publishing company he signed the contract with was not the one he intended to work with. Can Mr. X claim that the
contract is void due to a mistake in identity?

(a) The contract is void because there was a mistake in the identity of the publishing company.
(b) The contract is valid because the mistake was not essential to the agreement.
(c) The contract is void because Mr. Y used the wrong name by mistake.
(d) The contract is valid because both parties agreed upon the terms of the contract.

Passage 27
In order to be successful in an action for tort, the plaintiff has to prove that there has been legal damage caused to
him. In other words, it has to be proved that there was a wrongful act—an act or omission— causing breach of a legal
duty or the violation of a legal right vested in the plaintiff. Unless there has been violation of a legal right, there can
Legal Reasoning Practice Sheet 43
CLAT POINT

be no action under law of torts. If there has been violation of a legal right, the same is actionable whether, as a
consequence thereof, the plaintiff has suffered any loss or not.

This is expressed by the maxim "Injuria sine damno." Injuria means infringement of a right conferred by law on the
plaintiff or an unauthorized interference, howsoever trivial, with the plaintiff's right. Damnum means substantial harm,
loss or damage in respect of money, comfort, health or the like. Thus, when there has been injuria or the violation of
a legal right and the same has not been coupled with a damnum or harm to the plaintiff, the plaintiff can still go to the
court of law because no Violation of a legal right should go unredressed.

Since what is actionable is the violation of a legal right, it therefore follows that when there is no violation of a legal
right, no action can lie in a court of law even though the defendant's act has caused some loss or harm or damage to
the plaintiff. This is expressed by the maxim 'Damnum sine injuria'. It means that a damage without the violation of a
legal right is not actionable in a court of law. The reason for the same is that if the interference in the rights of another
person is not unlawful or unauthorized but a necessary consequence of the exercise of his own lawful rights by the
defendant, no action should lie. Thus, the test to know whether the defendant should or should not be liable is not
whether the plaintiff has suffered any loss or not but the real test is whether any lawful right vested in the plaintiff,
has been violated or not.

[Source: Conceptual Study of Law of Torts and General Defences, https://www.legalserviceindia.com/legal/article-


4469-conceptual-study-of-law-of-torts-and-general-defences.html

131. Mr. E, an esteemed baker, is known for his unique bread recipe that attracts customers from all over the city. Mr.
F, who recently started his own bakery nearby, discovers a new bread-making technique that greatly reduces the cost
of production while maintaining the quality. This results in Mr. F selling the bread at a lower price, leading to a decline
in sales for Mr. E. Frustrated, Mr. E files a suit against Mr. F, seeking compensation for the losses suffered. Will Mr. E
succeed in his suit?

(a) Mr. E will succeed in the suit because Mr. F's innovation has caused substantial harm to his business.
(b) Mr. E will not succeed in the suit because, although Mr. F's innovation has caused harm to Mr. E's business,
there was no violation of a legal right.
(c) Mr. E will succeed in the suit because Mr. F's lower prices create an unfair competitive advantage over Mr. E's
established business.
(d) Mr. E will not succeed in the suit because it is the responsibility of Mr. E to adapt his business to changing
market conditions.

132. Ms. A, a resident of the town of Greenfield, sought to create a beautifully landscaped garden in her front yard.
However, the town recently passed a new zoning law prohibiting gardens of this type in the specific area where Ms.
A's property is located. Despite this, Ms. A went ahead and created the garden. Her neighbor, Mr. B, who always
disliked her, reported the violation to the town authorities. Ms. A was subsequently required to remove the garden.
Distraught, Ms. A sues the town for damages. Can Ms. A succeed in her suit?

(a) Ms. A will succeed in the suit because the garden improved the overall appearance of the neighborhood, thus
benefiting the community.
(b) Ms. A will succeed in the suit because her right to use her property as she wishes was unlawfully restricted by
the new zoning law.
(c) Ms. A will not succeed in the suit because, although the removal of the garden caused her distress, there was
no violation of a legal right.
(d) Ms. A will not succeed in the suit because the town authorities were simply enforcing the zoning laws enacted
to regulate land use in Greenfield.
Legal Reasoning Practice Sheet 44
CLAT POINT

133. ‘A’ a Member of Legislative Assembly (MLA) of a certain State of India, was arrested by the police of the
respective state. A was not produced before the Magistrate within requisite period. A was later released and brought
a suit against the state. Will he succeed or not.

(a) Will succeed for violation of his legal rights were violated.
(b) Will not succeed because police of every state have a special power of arresting anyone, they find suspicious
under the domain of National Security.
(c) Will succeed because MLA enjoys special exceptional power under Constitution of India where they cannot be
arrested for any crime unless proven guilty.
(d) Will not succeed because politicians in India has a dark history and court will give decision in the favour of
police as they are doing national job unlike politicians.

134. Mr. C is an amateur painter who enjoys creating beautiful and vibrant artwork. He lives in a small town that
attracts tourists due to its scenic beauty. Mr. D is a local art gallery owner who has been struggling to stay afloat in the
increasingly competitive art market. Mr. C, in an attempt to contribute to the local community and showcase his
artwork, decides to set up a free outdoor exhibition along the town's main street. As a result, Mr. D's art gallery
experiences a significant drop in foot traffic and revenue. Mr. D files a suit against Mr. C, claiming compensation for
the losses suffered. Will Mr. D succeed in his suit?

(a) Mr. D will succeed in the suit because Mr. C's actions were in direct competition with his business, leading to
substantial losses.
(b) Mr. D will not succeed in the suit because, although Mr. C's exhibition caused harm to Mr. D's business, there
was no violation of a legal right.
(c) Mr. D will succeed in the suit because the outdoor exhibition constituted an unfair business practice that gave
Mr. C an undue advantage.
(d) Mr. D will not succeed in the suit because it is the responsibility of Mr. D to adapt his business to changing
market conditions.

135. Mrs. Hopper lives in a society and has a lawn in her bungalow. She mows her lawn every morning with the help
of a lawnmower. Her one of the neighbour Mr. Whooper files a tort claim against her stating that noise of the
lawnmower disturbs him. Will his claim succeed?

(a) His claim will not succeed because Mrs. Hopper has every right of using her land in any way she wants. And if
Mr. Whooper has any problem, he should change his locality.
(b) His claim will succeed because Mrs. Hopper is creating public nuisance.
(c) His claim will not succeed because there is no legal damage to him.
(d) His claim will succeed because Mrs Hopper has other ways of mowing the lawn but she deliberately uses a
lawnmower to annoy him.

Passage 28
This Article advances the view that the reasonable person is best understood as a standard for evaluating conduct, not
as a psychological construct for evaluating people. That is not to deny that some cases and commentators have
described the reasonable person in terms that are not only evaluative but also psychological. The point is that these
descriptions are misleading and ultimately unhelpful in understanding the reasonable person concept. The reasonable
person is not the average person, the normal person, or the typical person. Rather, the reasonable person is an
idealized standard that embodies the idea of what a person should do or not do in a given situation. The reasonable
person standard is an important part of negligence law because it allows courts to compare the defendant's conduct
to that of an idealized person in order to determine if the defendant has breached a duty of care.

Legal Reasoning Practice Sheet 45


CLAT POINT

The reasonable person standard is not meant to be a strict liability standard, which would impose liability on a
defendant even if the defendant took reasonable precautions to avoid causing harm. Instead, the reasonable person
standard is a negligence standard that focuses on the reasonableness of the defendant's conduct. This distinction is
crucial because it allows the law to strike a balance between the competing interests of protecting individuals from
harm and allowing actors to engage in activities that may have some risk of harm.

The reasonable person standard is also an objective standard, which means that it does not take into account the
subjective beliefs or intentions of the defendant. Instead, the standard focuses on what a reasonable person would
do in the defendant's situation, taking into account the objective circumstances of the case. This objective approach
has several advantages. First, it allows courts to apply a consistent standard across different cases, which promotes
fairness and predictability in the law. Second, it avoids the difficulties of trying to determine the defendant's subjective
state of mind, which can be difficult to prove and can lead to inconsistent results. Finally, the objective approach
encourages actors to conform their conduct to a socially acceptable standard, rather than relying on their own
subjective beliefs about what is reasonable.

In applying the reasonable person standard, courts often consider factors such as the foreseeability of harm, the
magnitude of the risk, the burden of taking precautions, and the social utility of the defendant's conduct. These factors
help courts to determine whether the defendant's conduct was reasonable under the circumstances, and whether the
defendant should be held liable for negligence. In some cases, courts may also consider whether the defendant had
any special skills or knowledge that would affect the reasonableness of their conduct. For example, a professional,
such as a doctor or engineer, may be held to a higher standard of care than an ordinary person in certain situations.

[Source: Extracted from - Negligence and the Test of the Reasonable Person" by Benjamin C. Zipursky, published in
the Yale Law Journal, Volume 130, Issue 6 (2021), pages 1690-1733].

136. A surgeon is sued for negligence after a patient suffers severe complications during a routine surgery. The surgeon
argues that they followed the standard protocol for the surgery and that the complications were unforeseeable. The
plaintiff alleges that the surgeon had an unusually high number of patients that day and was rushing through the
surgeries. Which of the following factors would a court most likely consider when determining if the surgeon's conduct
was reasonable under the reasonable person standard?

(a) The surgeon's subjective belief about the reasonableness of their conduct in relation to the number of patients
they treated that day.
(b) The foreseeability of harm from rushing through surgeries and the potential for complications.
(c) The social utility of the surgeon's conduct in performing life-saving surgeries.
(d) The burden of taking precautions, such as limiting the number of surgeries performed in one day

137. A hypothetical situation involves a licensed electrician who installs electrical wiring in a residential property. The
homeowner later sues the electrician for negligence, claiming that the wiring caused a fire. The electrician asserts that
they followed all applicable electrical codes and guidelines. In determining the electrician's liability under the
reasonable person standard, which aspect would be most relevant?

(a) The electrician's strict adherence to the electrical codes and guidelines.
(b) The electrician's specialized skills and knowledge as a licensed professional.
(c) The homeowner's subjective belief about the reasonableness of the electrician's conduct.
(d) The social utility of the electrician's conduct in providing electrical services.
Legal Reasoning Practice Sheet 46
CLAT POINT

138. A company that provides bungee jumping experiences is sued for negligence after a participant is severely injured
during a jump. The company argues that they followed all safety regulations, used proper equipment, and provided
detailed instructions to the participant. The plaintiff claims that the company failed to properly inspect the bungee
cord, which ultimately snapped during the jump. Under the reasonable person standard, can the company be held
liable for negligence?

(a) Yes, because the company is strictly liable for engaging in inherently risky activities.
(b) Yes, because the company's failure to properly inspect the bungee cord indicates unreasonable conduct given
the foreseeability of harm, the magnitude of the risk, and the burden of taking additional precautions.
(c) No, because the company's subjective belief is that they took all necessary precautions to avoid causing harm.
(d) No, because the law aims to strike a balance between protecting individuals from harm and allowing
companies to engage in activities that may have some risk of harm.

139. A store owner is sued for negligence after a customer slips and falls on a wet floor in the store. The store owner
claims that they were in the process of retrieving a "wet floor" sign when the accident occurred, and they believed
they were taking reasonable precautions. Under the reasonable person standard, can the store owner be held liable
for negligence?

(a) Yes, because the store owner's subjective belief about taking reasonable precautions is considered under the
reasonable person standard.
(b) Yes, because the court would focus on what a reasonable person would do in the store owner's situation,
considering the objective circumstances of the case, such as the risk of harm and the burden of taking
precautions.
(c) No, because the store owner was in the process of retrieving a "wet floor" sign and believed they were taking
reasonable precautions.
(d) No, because the court would consider the store owner's subjective belief that they were taking reasonable
precautions and the intentions behind their actions.

140. Suppose in the above question, the store owner is sued for negligence after a customer slips and falls on a wet
floor in the store. The store owner claims that they had just placed a "wet floor" sign to warn customers, and they
believed they were taking reasonable precautions. However, the sign was not clearly visible due to the placement of
store merchandise. Under the reasonable person standard, can the store owner be held liable for negligence?

(a) Yes, because the store owner's subjective belief about taking reasonable precautions is considered under the
reasonable person standard.
(b) Yes, because the court would focus on what a reasonable person would do in the store owner's situation,
considering the objective circumstances of the case, such as the risk of harm and the burden of taking
precautions, including the proper placement of the "wet floor" sign.
(c) No, because the store owner had placed a "wet floor" sign and believed they were taking reasonable
precautions.
(d) No, because the court would consider the store owner's subjective belief that they were taking reasonable
precautions and the intentions behind their actions.

Passage 29
The Indian Penal Code (IPC) is the main criminal code of India. It was enacted in 1860, during the British colonial period,
and has been amended several times since then. The IPC sets out various criminal offences and their punishments.

Legal Reasoning Practice Sheet 47


CLAT POINT

One offence in the IPC is Section 354, which deals with the crime of assault or criminal force to a woman with intent
to outrage her modesty. This offence is a serious crime and carries a punishment of up to three years imprisonment,
along with a fine.

The section defines "outraging the modesty of a woman" as any act that is intended to or results in the offending
person insulting the modesty of a woman. This can include actions such as making sexual advances, using vulgar
language, or making unwelcome physical contact. The act must be done with the intention of outraging the woman's
modesty or knowing that it is likely to do so.

The section further provides that if the offender uses criminal force against a woman with the intention of outraging
her modesty, the punishment can be increased to up to five years imprisonment, along with a fine. Criminal force
means the use of physical force against the woman, such as touching her inappropriately or restraining her.

The offence under Section 354 is a cognizable offence, which means that the police can arrest the accused without a
warrant. The offence is also non-bailable, which means that the accused cannot be released on bail as a matter of
right.

The section is an important tool for protecting women from sexual harassment and assault. It is often used in cases
where a woman has been groped or molested in a public place, or where a man has made lewd comments or gestures
towards a woman. The offence is gender-specific and can only be committed against a woman.

[Source: Indian Penal Code, 1860, § 354, India Code, available at https://www.indiacode.nic.in/.]

141. In a hypothetical scenario, four individuals - A, B, C, and D - engage in separate incidents involving a woman
named E, with each scenario described in intricate detail. Based on the provided passage, determine which individual's
actions would constitute "outraging the modesty of a woman." Select the correct option from the following:

I. a researcher conducting a study on interpersonal space in public transport, positions himself in close proximity to E
on a crowded train, accidentally brushes against her arm, and records her reaction. He then immediately apologizes
and informs her about the study.

II. an avant-garde performance artist, recites a poem at a public event that E attends. The poem contains explicit
language and vivid imagery, some of which are sexual in nature, but it is not directed at E or any other individual in
the audience.

III. a linguistics professor, writes an academic paper analyzing the use of offensive language in online discourse. To
illustrate her point, she quotes an anonymous social media user who directed a sexually explicit insult at E. The paper
is published in a prestigious journal.

IV. a participant in a virtual reality gaming platform, creates an avatar that closely resembles E without her knowledge
or consent. He then engages in explicit interactions with other players, using E's likeness in the virtual environment.

(a) I and II
(b) III only
(c) IV only
(d) III and IV

Legal Reasoning Practice Sheet 48


CLAT POINT

142. A, a renowned dance instructor, is teaching an advanced dance class in which E is a participant. The dance routine
they are practicing involves several intricate movements that require close physical contact between the partners.
During the class, A engages in an action towards E. Based on the provided passage, determine which of the following
actions by A would most likely constitute the use of criminal force with the intention of outraging the modesty of a
woman, leading to potential increased punishment. Select the correct option from the following:

I. Adjusts E's posture during a dance move, inadvertently touching her in a manner that could be perceived as
inappropriate, but immediately apologizes and ensures not to repeat the action.
II. Uses a metaphorical description involving intimate imagery to explain a dance move to the class, which E finds
offensive, but does not involve any physical contact.
III. Under the guise of assisting E with a dance move, intentionally touches her inappropriately, exploiting the
close physical contact required in the dance routine.
IV. A and E execute a complex lift during the dance routine, but A loses his grip, causing E to fall.
(a) I
(b) II
(c) III
(d) Both III and IV

143. A, a renowned dance instructor, and E, a participant in the advanced dance class, A intentionally touches E
inappropriately while assisting her with a dance move, exploiting the close physical contact required in the dance
routine. A misinterprets E's body language and believes that she has given consent to his actions. Considering the
provided passage, would A likely be held liable for using criminal force with the intention of outraging the modesty of
a woman?

(a) Yes, A would likely be held liable because his actions involve intentional inappropriate touching, regardless of
his misinterpretation of E's body language.
(b) No, A would not likely be held liable because he genuinely believed E had given consent to the contact, negating
the intention to outrage her modesty.
(c) Yes, A would likely be held liable, but only if E explicitly communicates her lack of consent after the incident.
(d) No, A would not likely be held liable because the close physical contact is a necessary part of the advanced
dance class, and any misinterpretation of consent is considered an inherent risk.

144. Suppose in the above question A assists E with a dance move that requires close physical contact. While doing
so, A inadvertently touches E inappropriately. He genuinely believed that the contact was necessary for executing the
dance move. Considering the provided passage, would A likely be held liable for using criminal force with the intention
of outraging the modesty of a woman?

(a) Yes, A would likely be held liable because the physical contact was inappropriate, regardless of his intentions.
(b) No, A would not likely be held liable because he genuinely believed the contact was necessary for executing
the dance move, even though he inadvertently touched E inappropriately.
(c) Yes, A would likely be held liable, but only if E explicitly communicates her lack of consent after the incident.
(d) No, A would not likely be held liable because close physical contact is a necessary part of the advanced dance
class, and any unintentional inappropriate touching is considered an inherent risk.

145. F is attending a professional conference where she encounters G, a successful businesswoman. During a break, F
overhears G making inappropriate comments about another woman, H, who is also attending the conference.
Subsequently, G approaches H and deliberately touches her inappropriately in front of F and other attendees.
Considering the provided passage, whether G can be held liable under 354?

(a) G cannot be held liable under Section 354, as the offense is gender-specific and can only be committed by a
man against a woman.
(b) G can be arrested without a warrant and denied bail as a matter of right, as her actions constitute an offense
under Section 354, as the offense is gender specific only in terms of a victim and not an accused.
Legal Reasoning Practice Sheet 49
CLAT POINT

(c) G can be held liable under Section 354, but only if H explicitly communicates her lack of consent after the
incident and other attendees testify against G.
(d) G's liability under Section 354 depends on whether the jurisdiction in question has expanded the scope of the
offense to include acts committed by women against other women.

Passage 30
Article 19 of the Indian Constitution guarantees the fundamental right to freedom of speech and expression. This right,
however, is not absolute and is subject to certain reasonable restrictions. Article 19(1)(a) states that all citizens shall
have the right to freedom of speech and expression. This right extends to any form of communication, including spoken
words, written text, and various forms of media, such as radio, television, and the internet.

Article 19(2) outlines the reasonable restrictions that can be imposed on this right in the interest of the sovereignty
and integrity of India, the security of the state, friendly relations with foreign states, public order, decency, or morality,
or in relation to contempt of court, defamation, or incitement to an offense. These restrictions are imposed to balance
the individual's right to free speech with the need to maintain public order and protect the rights and reputations of
others.

The Indian courts have played a significant role in interpreting the scope and limits of Article 19(1)(a). In several
landmark judgments, the courts have laid down the principles for determining the constitutionality of restrictions on
free speech. One such principle is the "clear and present danger" test, which requires the restriction to be necessary
to prevent a clear and imminent threat to public order or national security. Another important principle is the
"proportionality" test, which mandates that the restriction should be proportional to the objective it seeks to achieve
and should not impose an excessive burden on the right to free speech.

Over the years, the courts have also recognized various forms of speech that are not protected under Article 19(1)(a).
These include hate speech, obscenity, defamation, and speech that incites violence or undermines national security.
The courts have emphasized that the right to free speech does not extend to the propagation of ideas that are
prejudicial to the public interest or that promote enmity between different groups.

In addition to Article 19, the Indian Constitution also guarantees other fundamental rights related to free speech, such
as the right to assemble peacefully and without arms (Article 19(1)(b)) and the right to form associations or unions
(Article 19(1)(c)). These rights, too, are subject to reasonable restrictions in the interest of public order and national
security.

The freedom of speech and expression enshrined in Article 19 is a cornerstone of India's democratic system. It enables
citizens to express their opinions and ideas, fostering the exchange of diverse perspectives and promoting intellectual
growth. However, it is important to strike a balance between protecting individual liberties and maintaining social
harmony, which is achieved through the imposition of reasonable restrictions on this fundamental right.

[Source: Constitution of India-Freedom of speech and expression, https://www.legalserviceindia.com/legal/article-


572-constitution-of-india-freedom-of-speech-and-expression.html ]

146. Journalist X publishes an article on a news website criticizing the government's economic policies. Subsequently,
the government enforces a new regulation restricting online publications from discussing economic policies without
prior approval. Journalist X challenges the regulation, claiming it violates their fundamental right to freedom of speech
and expression under Article 19(1)(a) of the Indian Constitution. Will X's claim succeed?

(a) No, the regulation is valid as it falls within the scope of reasonable restrictions allowed by Article 19(2) to
protect the sovereignty and integrity of India.
Legal Reasoning Practice Sheet 50
CLAT POINT

(b) Yes, the regulation is invalid, as it infringes upon the fundamental right to freedom of speech and expression
guaranteed by Article 19(1)(a) without a justifiable reason.
(c) No, the regulation is valid as long as it is proportional to the objective it seeks to achieve and does not impose
an excessive burden on Journalist X's right to free speech.
(d) Yes, the regulation is invalid, as it specifically targets online publications, thus violating the principle of equality
before the law.

147. Political Activist Y organizes a protest against the government's policies on a sensitive issue. The protest is
peaceful, but Y delivers a speech that contains inflammatory remarks and encourages civil disobedience. The
government argues that Y's speech should be restricted under Article 19(2) of the Indian Constitution. Y claims that
this would violate their right to freedom of speech and expression. Is the government’s argument valid?

(a) The government's argument is valid because Y's speech poses a threat to public order.
(b) The government's argument is invalid because Y's speech is protected under the right to free speech, regardless
of its content.
(c) The government's argument is valid because Y's speech incites an offense, which falls under the reasonable
restrictions of Article 19(2).
(d) The government's argument is invalid because Y's speech constitutes peaceful protest and does not infringe
upon any rights or reputations of others.

148. Filmmaker Z releases a controversial documentary that investigates alleged corruption within the government.
The documentary receives widespread attention and sparks intense debate. The government seeks to impose a
restriction on the film's distribution, claiming that it poses a threat to public order. Filmmaker Z challenges the
restriction, arguing that it violates their right to freedom of speech and expression under Article 19(1)(a) of the Indian
Constitution. Assess the validity of the government's claim using the "clear and present danger" test established by
Indian courts.

(a) The government's claim is valid because the documentary has the potential to incite protests, which could
disrupt public order.
(b) The government's claim is invalid because the documentary does not present a clear and imminent threat to
public order or national security.
(c) The government's claim is valid because the documentary's content is defamatory and falls within the
reasonable restrictions of Article 19(2).
(d) The government's claim is invalid because the documentary is protected by the right to freedom of speech and
expression, regardless of its controversial nature.

149. isolated incidents of violence between different religious communities. The government decides to ban the book,
claiming that it threatens public order and decency. Author X challenges the ban, arguing that it violates their right to
freedom of speech and expression under Article 19(1)(a) of the Indian Constitution. Whether the government's
decision is valid under the "proportionality" test established by Indian courts.

(a) The government's decision is valid because the ban is proportional to the objective of maintaining public order
and decency.
(b) The government's decision is invalid because the ban is excessive and disproportionately restricts the right to
free speech.
(c) The government's decision is valid because the book's content threatens friendly relations with foreign states.
(d) The government's decision is invalid because the book is protected by the right to freedom of speech and
expression, regardless of its controversial nature.

150. A political activist delivers a speech during a public demonstration in the midst of escalating tensions between
two ethnic groups in a region. The activist's speech consists of various statements, each addressing different aspects

Legal Reasoning Practice Sheet 51


CLAT POINT

of the ongoing conflict. The government decides to prosecute the activist, claiming that the speech violates the right
to freedom of speech. Based on the provided information, which of the following statements, if proven true, would
provide the most valid justification for holding the activist liable?

(a) The activist's speech highlights the historical injustices faced by one ethnic group, leading to a significant
increase in the group's political engagement and a shift in the region's balance of power.
(b) The activist's speech includes a detailed analysis of the socio-economic disparities between the two ethnic
groups, and proposes policies aimed at reducing inequality and fostering integration.
(c) The activist's speech contains explicit language that promotes enmity between the two ethnic groups, resulting
in a series of violent clashes that exacerbate the existing tensions in the region.
(d) The activist's speech criticizes the government's handling of the conflict, alleging that the current
administration is biased in favor of one ethnic group, and calls for a peaceful protest against the government's
policies.

Passage 31
The National Security Act, 1980 has been invoked in the case of Amritpal Singh, the self-styled Sikh preacher and leader
of Waris Punjab De who is currently on the run. This was confirmed by Punjab Advocate General Vinod Ghai during a
hearing at the Punjab and Haryana High Court on a habeas corpus petition filed by the legal advisor of Waris Punjab
De.

The National Security Act, passed by the Indian Parliament in 1980, has been amended several times since then. The
Act allows the state to detain a person without a formal charge and without trial to prevent them from acting in any
manner prejudicial to the security of the state or for the maintenance of public order. It is an administrative order
passed by the Divisional Commissioner or the District Magistrate, and not detention ordered by police based on
specific allegations or for a specific violation of the law.

The NSA can be invoked to prevent a person from acting in any manner prejudicial to the defence of India, relations
of India with foreign powers or the security of India. Among others, it can also be applied to prevent a person from
acting in any manner prejudicial to the maintenance of supply and services essential to the community. An individual
can be detained without a charge for a maximum period of 12 months. The detained person can be held for 10 to 12
days in special circumstances without being told the charges against them.

The Indian Constitution allows both preventive detention and the right of protection against arrest and detention in
certain cases, enshrined under Article 22 of the Constitution. However, Article 22(3) provides that the rights available
to an arrested person will not be applicable in case of preventive detention. One crucial procedural safeguard under
the NSA is granted under Article 22(5), where all the detained persons have the right to make an effective
representation before an independent advisory board, which consists of three members, chaired by a member who
is, or has been, a judge of a high court.

The Act has been criticised by human rights groups who argue that it vitiates Article 22 of the Constitution and various
provisions under the CrPC that safeguard the interest of an arrested person, namely that the arrested person should
be informed regarding the ground of arrest and their right to consult a legal practitioner. Some human rights groups
argue that the Act is often misused by authorities to silence political opponents or those who are critical of the
government. However, there is an opposing view that the Act protects the larger interest of the state.

Legal Reasoning Practice Sheet 52


CLAT POINT

The Supreme Court has previously held that the law of preventive detention has to be strictly construed to prevent
misuse of this potentially dangerous power, and meticulous compliance with the procedural safeguards has to be
ensured.

[Source: Extracted with edits and revision from https://economictimes.indiatimes.com/news/politics-and-nation/nsa-


invoked-against-amritpal-singh-punjab-government-tells-high-court-lawyer/articleshow/98859809.cms?from=mdr ]

151. X is a social activist who has been organizing protests against government policies. Recently, the government has
been facing severe criticism from various sections of the society for these policies. The Divisional Commissioner passes
an order under the National Security Act to detain X without a formal charge or trial. X challenges this order, arguing
that it violates his fundamental rights. Will X succeed in his claim?

(a) Yes, because the detention is not based on specific allegations or for a specific violation of the law.
(b) No, because the Act allows the state to detain a person to prevent them from acting in any manner prejudicial
to the security of the state or for the maintenance of public order.
(c) Yes, because X is a social activist, and his actions do not qualify as prejudicial to the security of the state or for
the maintenance of public order.
(d) No, because X is organizing protests, which can be considered prejudicial to the security of the state or for the
maintenance of public order.

152. X is a social activist who has been organizing peaceful protests against government policies. Recently, the
government has been facing severe criticism from various sections of the society for these policies. The District
Magistrate passes an order under the National Security Act to detain X without a formal charge or trial. However,
there is no evidence to suggest that X's actions have caused or are likely to cause any harm to the security of the state
or the maintenance of public order. X challenges this order, arguing that it violates his fundamental rights. Will X
succeed in his claim?

(a) Yes, because the detention is not based on specific allegations or for a specific violation of the law.
(b) No, because the Act allows the state to detain a person to prevent them from acting in any manner prejudicial
to the security of the state or for the maintenance of public order.
(c) Yes, because X's peaceful protests do not qualify as prejudicial to the security of the state or for the
maintenance of public order, and there is no evidence to suggest otherwise.
(d) No, because X is organizing protests, which can be considered prejudicial to the security of the state or for the
maintenance of public order.
153. X, a political activist, organizes a large-scale protest against a government project. The protest disrupts the supply
chain of essential goods and services to the community, causing shortages of vital resources. As a result, the authorities
decide to detain X under the National Security Act (NSA) to prevent further disruptions. X is held for 11 days without
being informed of the charges against him. X later challenges the detention, claiming that it is illegal and violates his
fundamental rights. Will X be held liable?

(a) Yes, because X's actions disrupted the supply chain of essential goods and services to the community, which
can be considered prejudicial under the NSA.
(b) No, because the detention period of 11 days without informing X of the charges against him is unlawful under
the NSA.
(c) Yes, because X's actions can be considered prejudicial to the security of India, and the NSA allows for detention
without charge for a maximum period of 12 months.
(d) No, because X's actions, though disruptive, do not fall within the scope of the NSA's provisions relating to the
defense of India or relations with foreign powers.

Legal Reasoning Practice Sheet 53


CLAT POINT

154. X, a political activist, organizes a small-scale peaceful protest against a government project. The protest does not
disrupt the supply chain of essential goods and services to the community, nor does it have any impact on the defense
of India, relations of India with foreign powers, or the security of India. Despite this, the authorities decide to detain X
under the National Security Act (NSA) to prevent any potential disruptions. X is held for 11 days without being informed
of the charges against him. X later challenges the detention, claiming that it is illegal and violates his fundamental
rights. Will X be held liable?

(a) No, because X's actions did not disrupt the supply chain of essential goods and services to the community, nor
did they affect the defense of India, relations of India with foreign powers, or the security of India.
(b) Yes, because the NSA allows for detention without charge for a maximum period of 12 months, and X's actions
could potentially disrupt the supply chain of essential goods and services to the community in the future.
(c) No, because the detention is unlawful under the NSA, as X's actions do not fall within the scope of any of the
categories mentioned in the NSA for detention without charge.
(d) Yes, because X's actions, although peaceful and small-scale, can be considered a potential threat to the
maintenance of supply and services essential to the community under the NSA

155. A, a political dissident, is arrested and placed under preventive detention under the National Security Act for
allegedly posing a threat to the country's security. A claims that his detention violates his fundamental rights enshrined
in Article 22 of the Indian Constitution, as he has not been informed of the grounds for his detention and has not been
granted the right to consult and be defended by a legal practitioner of his choice. A files a petition challenging his
detention. Will A succeed in his claim?

(a) Yes, because A's detention violates his fundamental rights under Article 22 of the Indian Constitution, which
provides protection against arrest and detention in certain cases.
(b) No, because A's rights under Article 22 are not applicable in cases of preventive detention, as specified in
Article 22(3) of the Indian Constitution.
(c) Yes, because A has not been given the opportunity to make an effective representation before an independent
advisory board, as mandated by Article 22(5) of the Indian Constitution.
(d) No, because A's right to consult and be defended by a legal practitioner of his choice is not guaranteed under
Article 22(5) in cases of preventive detention.

Passage 32

The Contract Act draws a distinction between an agreement which is only void and the one in which the consideration
or object is also unlawful. "Section 23 points out in what cases the consideration of an agreement is unlawful, and in
such cases the agreement is also void, that is, not enforceable at law.'" Sections 25 to 30 refer to cases in which the
agreement is only void, though the consideration is not necessarily unlawful. An illegal agreement is one which is
actually forbidden by the law; but a void agreement may not be forbidden, "the law may merely say that if it is made,
the courts will not enforce it". Thus, every illegal contract is also void, but a void contract is not necessarily illegal.

However, the distinction is not always clear. Another similarity between an illegal and a void agreement is that in
either case, the main or the primary agreement is unenforceable. Nothing can be recovered under either kind of
agreement and if something has been delivered or some payment made, it cannot be recovered back. It is settled law
that where the parties are not in pari delicto the less guilty may be able to recover money paid, or property transferred,
under the contract. This possibility may arise in three situations: First, the contract may be of a kind made illegal by a
statute in the interest of a particular class of persons of whom the plaintiff is one. Secondly, the plaintiff must have
been induced to enter into the contract by fraud or strong pressure. Thirdly, there is some authority for the view that
a person who is under a fiduciary duty to the plaintiff will not be allowed to retain property, or to refuse to account
for money received, on the grounds of an illegal transaction."

Legal Reasoning Practice Sheet 54


CLAT POINT

One of the exceptions, where the contract is still executory, in the sense that no part of the illegal purpose has been
carried into effect, the money paid or goods delivered under it may be recovered.

[Source: Extracted with revisions and edits from Difference Between Void And Illegal Contract: Everything You Need
to Know https://www.upcounsel.com/difference-between-void-and-illegal-contract]

156. Mr. X paid a large sum of money to Mr. Y, a well-connected political leader, in exchange for a high-ranking
government job. Mr. Y assured Mr. X that the position was all but guaranteed. After months of waiting and seeing no
progress, Mr. X demanded his money back from Mr. Y. To settle the matter, Mr. Y issued a cheque to Mr. X, but upon
attempting to cash the cheque, Mr. X found the payment was disallowed. Given the circumstances, can the contract
between Mr. X and Mr. Y be enforced?

(a) The contract can be enforced between Mr. X and Mr. Y because the parties reached a mutual understanding
and there was valid consideration in the contract.
(b) The contract cannot be enforced because the act of bribery is forbidden by law, rendering the consideration
illegal and the contract void.
(c) The contract cannot be enforced because the Indian Contract Act does not recognize unlawful agreements,
categorizing them as void.
(d) The contract can be enforced because Mr. X is a victim of fraud and is entitled to recover the money paid under
the agreement.

157. Mr. L, an event organizer, entered into a contract with Mr. M, a property owner, to rent a spacious hall for a large
gathering. Mr. L provided all necessary details, including the date, time, and purpose of the event, which appeared
legitimate. However, unbeknownst to Mr. M, the event's true purpose was to promote blasphemous ideas, which are
illegal in India. When Mr. M discovered the real intent behind the event, he immediately refused to hand over the keys
to the hall. Given these circumstances, what is the nature of the contract between Mr. L and Mr. M?

(a) The contract is valid as there is no express provision in the Indian Contract Act, 1872, which voids a contract
with a hidden illegal purpose.
(b) The contract is voidable at the option of Mr. M because he was deceived by Mr. L.
(c) The contract is void due to the illegal nature of its purpose.
(d) The contract is voidable at the option of Mr. L as he might argue that his intentions changed and the event
would no longer be blasphemous.

158. A debtor, Mr X executed a transfer to deceive his creditors, but before any creditor could be deceived, he
repented and sought to recover back the property. Can he be allowed to do so as per the exception to the Section 23
provided in the passage?

(a) He cannot be allowed to recover because illegal purpose has already been carried into effect.
(b) He cannot be allowed to recover as there was an illegal consideration to begin with.
(c) He can be allowed to recover as the contract is still executory and no illegal purpose has been carried into
effect.
(d) None of the above.

159. In a small town, Mr. A, a businessman, entered into a contract with Mr. B, an influential political figure, for the
construction of a bridge across a local river. Unbeknownst to the town's residents, Mr. A and Mr. B agreed to use
substandard materials in order to save on costs and split the savings between themselves. This agreement was illegal,
as it endangered public safety. Upon completion of the bridge, a whistleblower revealed the illegal agreement

Legal Reasoning Practice Sheet 55


CLAT POINT

between Mr. A and Mr. B, and the authorities began an investigation. Which of the following statements is true
regarding the contract between Mr. A and Mr. B?

(a) The contract is valid because both Mr. A and Mr. B had consented to it and provided consideration.
(b) The contract is voidable at the option of Mr. A because he was under pressure from Mr. B, the political figure.
(c) The contract is void due to the illegal nature of the agreement between Mr. A and Mr. B.
(d) The contract is voidable at the option of Mr. B as he can claim that the bridge met the minimum safety
requirements despite using substandard materials.

160. Mr. P, an investor, entered into a contract with Mr. Q, the owner of a pharmaceutical company. The contract
involved Mr. P investing a large sum of money in Mr. Q's business, which would be used to develop and sell a new
drug. Unbeknownst to Mr. P, the drug was not approved by the government for human consumption, making its sale
illegal. When Mr. P discovered the illegal nature of the business, he sought to recover the invested money. Which of
the following statements is true regarding the contract between Mr. P and Mr. Q?

(a) The contract is voidable at the option of Mr. P, as he was not aware of the drug's illegal status at the time of
the contract.
(b) The contract is valid because both Mr. P and Mr. Q had consented to it and provided consideration.
(c) The contract is void due to the illegal object of selling the unapproved drug.
(d) The contract is voidable at the option of Mr. Q, as he can argue that the drug's unapproved status was not
crucial to the contract.

Passage 33

The Hon’ble Supreme Court of India directed the Central Govt to amend the provision of property inheritance under
the Hindu Succession Act so that it is applicable to Scheduled Tribes. The apex court mentioned that when daughters
of a non-tribal household are entitled to an equal share in the father’s property, the same should be applicable to
daughters from tribal communities. Presently, Section 2 (2) of the Hindu Succession Act states that the Hindu
Succession Act is not applicable to Scheduled Tribes (ST).

A Will is a document that clearly lists down all the assets that are to be divided among the heirs in the absence of the
owner of the property. All movable and immovable property including mutual funds, savings, fixed deposits, gold etc.
should be mentioned in the document. The assets should be divided in a clear manner stating who will inherit what
and signed by two witnesses. Indian Succession Act, 1925 governing Wills has not prescribed any particular format or
technical requirement.

Testamentary succession is applicable in cases where the deceased has left a Will. The property is distributed exactly
as per the Will. In the absence of a Will, the property is distributed as per the intestate succession. Intestate succession
laws under the Indian Succession Act are different for different religions. Hindus, Muslims and Christians have different
laws of succession in absence of a Will.

The Indian Succession Act governs the property rights of Christians in India. Before 1925, territories like Cochin,
Travancore, Pondicherry, Goa, Daman and Diu followed a different set of rules but the Indian Succession Act united
them all. Some Christians still follow their customary laws. The Christian law of inheritance and succession treats men
and women equally. Section 31 to 49 of the Indian Succession Act states that in case, a man leaves behind his wife and
children, then his widow will get one-third of his property and the remaining two-thirds will go to his direct lineage. If
there are no children, then the wife gets half of the property, and the other half is given to the kindred(close relatives).
The widow will get the full share if there are no children or kindred. In the absence of the widow, the property will be
Legal Reasoning Practice Sheet 56
CLAT POINT

equally distributed among the children. Children of children (grandchildren) will get a share if their parents are no
more.

Source: Extracted with edits and revisions from, Indian Succession Act 1925 - Indian Inheritance Laws ,
https://www.magicbricks.com/blog/indian-succession-act/129352.html#Indian_Succession_Act,_1925

161. Mr. X, a wealthy individual, passes away leaving behind a Will that he had drafted without any legal assistance.
The Will lists all of his assets, including real estate, savings, and personal possessions. However, the document is not
in a standard legal format and has several ambiguities. Mr. Y, one of Mr. X's relatives, challenges the validity of the
Will, arguing that it is not legally enforceable due to its non-standard format and unclear language. Based on the
information provided in the passage, will Mr. Y succeed in his claim?

(a) Mr. Y will succeed in his claim, as the Indian Succession Act, 1925 requires Wills to adhere to a strict format
and use specific language to be considered valid and enforceable.
(b) Mr. Y will not succeed in his claim, as the Indian Succession Act, 1925 does not prescribe any particular format
or technical requirements for Wills, allowing for variations in drafting and presentation.
(c) Mr. Y will succeed in his claim, as the ambiguities in the Will render it invalid, regardless of the Indian Succession
Act, 1925's lack of specific format and technical requirements.
(d) Mr. Y will not succeed in his claim, as the Indian Succession Act, 1925 only requires the Will to be signed by
two witnesses, and the passage does not indicate that this requirement has not been met.

162. Suppose in the above question, Mr X drafted a Will without any legal assistance. The Will lists all of his assets,
including real estate, savings, and personal possessions. The document is not in a standard legal format, but the
language is clear and unambiguous. Mr. Y, one of Mr. X's relatives, challenges the validity of the Will, arguing that it is
not legally enforceable due to its non-standard format. Based on the information provided in the passage, will Mr. Y
succeed in his claim?

(a) Mr. Y will succeed in his claim, as the Indian Succession Act, 1925 requires Wills to adhere to a strict format
and use specific language to be considered valid and enforceable.
(b) Mr. Y will not succeed in his claim, as the Indian Succession Act, 1925 does not prescribe any particular format
or technical requirements for Wills, allowing for variations in drafting and presentation.
(c) Mr. Y will succeed in his claim, as the non-standard format of the Will renders it invalid, regardless of the Indian
Succession Act, 1925's lack of specific format and technical requirements.
(d) Mr. Y will not succeed in his claim, as the Indian Succession Act, 1925 only requires the Will to be signed by
two witnesses, and the passage does not indicate that this requirement has not been met.

163. Mr. X, a Christian man, passes away intestate, leaving behind his wife, Mrs. Y, and their two children. The property
is to be divided according to the Christian law of inheritance and succession as outlined in the Indian Succession Act.
Mr. Z, a distant relative of Mr. X, claims that he is entitled to a share of the property. Based on the information provided
in the passage, will Mr. Z succeed in his claim?

(a) Mr. Z will succeed in his claim, as the Indian Succession Act mandates a division of property that includes distant
relatives when a person dies intestate.
(b) Mr. Z will not succeed in his claim, as the Indian Succession Act prescribes that in the case of intestate
succession, the widow receives one-third of the property, and the remaining two-thirds are distributed among
the direct lineage.
(c) Mr. Z will succeed in his claim, as the Indian Succession Act requires that distant relatives receive a small portion
of the property in cases of intestate succession.
(d) Mr. Z will not succeed in his claim, as the Indian Succession Act explicitly states that the division of property in
cases of intestate succession is permissible, as long as the widow receives at least one-third of the property
and the remaining is distributed among the direct lineage.
Legal Reasoning Practice Sheet 57
CLAT POINT

164. Mr. X, a Christian man, passes away intestate, leaving behind his wife, Mrs. Y, and no children. Mr. X has two
surviving siblings and a few distant relatives. Mrs. Y claims her right to half of Mr. X's property as per the Christian law
of inheritance and succession. However, Mr. Z, a distant relative of Mr. X, challenges her claim, arguing that he is
entitled to a share of the property as well. Based on the information provided in the passage, will Mrs. Y succeed in
her claim?

(a) Mrs. Y will succeed in her claim, as the Christian law of inheritance and succession entitles the wife to half of
the property when there are no children, with the other half being distributed among the kindred.
(b) Mrs. Y will not succeed in her claim, as the Christian law of inheritance and succession requires that all property
be equally divided among the surviving spouse and all relatives, including distant relatives, in cases of intestate
succession.
(c) Mrs. Y will succeed in her claim, as the Christian law of inheritance and succession mandates that the wife
receives half of the property when there are no children, with the other half being allocated to the siblings of
the deceased.
(d) Mrs. Y will not succeed in her claim, as the Christian law of inheritance and succession provides for equal
distribution of property among the surviving spouse, siblings, and distant relatives when there are no children.

165. Mr. A, a widower, passes away intestate, leaving behind his three adult children, Mr. X, Ms. Y, and Ms. Z, and two
minor grandchildren from his son, Mr. X. The adult children and the grandchildren are now claiming their respective
shares in Mr. A's property. Based on the information provided in the passage, will the grandchildren succeed in their
claim?

(a) The grandchildren will succeed in their claim, as the passage states that in the absence of a widow, the property
will be equally distributed among the children, and the grandchildren will get a share if their parents are no
more.
(b) The grandchildren will succeed in their claim, as the passage states that in the absence of a widow, the property
will be equally distributed among the children, and the grandchildren have the right to claim a portion of their
parent's share.
(c) The grandchildren will not succeed in their claim, as the passage states that only the widow and the children
are entitled to inherit the property, and the grandchildren have no right to claim any share in the absence of
their parent's death.
(d) None of the above

Passage 34

Legitimate Expectation means that a person may have a reasonable expectation of being treated in a certain way by
administrative authorities owing to some consistent practice in the past or an express promise made by the concerned
authority. According to this doctrine, a public authority can be made accountable in lieu of a legitimate expectation.
Thus, the doctrine of Legitimate Expectation pertains to the relationship between an individual and a public authority.

"What is legitimate expectation? Obviously, it is not a legal right. It is an expectation of a benefit, relief or remedy that
may ordinarily flow from a promise or established practice. The term 'established practice' refers to a regular,
consistent predictable and certain conduct, process or activity of the decision-making authority. The expectation
should be legitimate, that is, reasonable, logical and valid. Any expectation which is based on sporadic or casual or
random acts, or which is unreasonable, illogical or invalid cannot be a legitimate expectation. Not being a right, it is
not enforceable as such. It is a concept fashioned by courts, for judicial review of administrative action. It is procedural
in character based on the requirement of a higher degree of fairness in administrative action, as a consequence of the
promise made, or practice established. In short, a person can be said to have a 'legitimate expectation' of a particular
treatment, if any representation or promise is made by an authority, either expressly or impliedly, or if the regular and
consistent past practice of the authority gives room for such expectation in the normal course."
Legal Reasoning Practice Sheet 58
CLAT POINT

Therefore, it can be said that this doctrine is a form of a check on the administrative authority. When a representation
has been made, the doctrine of legitimate expectation imposes, in essence, a duty on public authority to act fairly by
taking into consideration all relevant factors relating to such legitimate expectation. It also adds a duty on the public
authority not to act in a way to defeat the legitimate expectation without having some reason of public policy to justify
its doing so.

The doctrine of legitimate expectation, based on established practice, can be invoked only by someone who has
dealings or transactions or negotiations with an authority, on which such established practice has a bearing, or by
someone who has a recognized legal relationship with the authority.

Whenever the question arises, it is to be determined not according to the claimant's perception but in the larger public
interest wherein other more important considerations may outweigh what would otherwise have been the legitimate
expectation of the claimant. A bona fide decision of the public authority reached in this manner would satisfy the
requirement of non-arbitrariness and withstand judicial scrutiny.

The expectation cannot be the same as anticipation. It is different from a wish, a desire or a hope nor does it amount
to a claim or demand on the ground of a right.

[Source - Extracted with edits and revisions from - Doctrine Of Legitimate Expectation

https://www.mondaq.com/india/constitutional--administrative-law/881956/doctrine-of-legitimate-expectation]

166. In the hypothetical city of Metropolis, a new policy is being proposed that would significantly change the process
of allocating government-funded scholarships for higher education. Citizen X, who meets all the current requirements
for receiving a scholarship, has filed a claim against the proposed policy, arguing that it would jeopardize their
legitimate expectation of receiving a scholarship. Will X succeed in their claim, considering the passage provided?

(a) Yes, X will succeed in their claim, as their legitimate expectation must be protected.
(b) No, X will not succeed in their claim if the new policy is determined to serve the larger public interest, even if
it affects X's legitimate expectation.
(c) Yes, X will succeed in their claim if the new policy does not take into account the unique circumstances of each
applicant.
(d) No, X will not succeed in their claim as long as the public authority makes a bona fide decision.

167. Suppose in the city of Metropolis, a new policy is being proposed to limit the construction of new shopping malls
within a certain radius from the city center. This policy aims to address the issue of congestion and promote local
businesses. Citizen X, who owns a large parcel of land within the affected radius and has secured necessary approvals,
has filed a claim against the proposed policy, arguing that it would jeopardy their legitimate expectation of constructing
a shopping mall on their property. Will X succeed in their claim, considering the passage provided?

(a) Yes, X will succeed in their claim, as their legitimate expectation must be protected.
(b) No, X will not succeed in their claim if the new policy is determined to serve the larger public interest, even if
it affects X's legitimate expectation.
(c) Yes, X will succeed in their claim if the new policy fails to take into account the potential benefits of constructing
a shopping mall.

Legal Reasoning Practice Sheet 59


CLAT POINT

(d) No, X will not succeed in their claim if the new policy is determined to serve the larger public interest, and
regards the potential benefits of constructing a shopping mall.

168. The local government has maintained a consistent practice of providing financial assistance to small businesses
that have been negatively impacted by natural disasters. This practice has been in place for the past 20 years. Citizen
A, who owns a small business, has recently been affected by a flood and expects to receive financial assistance based
on the established practice. However, the local government decides to discontinue the financial assistance program
for small businesses affected by natural disasters. Will A succeed in their claim for financial assistance, considering the
passage provided?

(a) Yes, A will succeed in their claim, as they have a legitimate expectation based on the established practice.
(b) No, A will not succeed in their claim, as legitimate expectation is not enforceable as a legal right.
(c) Yes, A will succeed in their claim, as the discontinuation of the program is unreasonable and illogical.
(d) No, A will not succeed in their claim, as legitimate expectation only applies to promises made by an authority.

169. Suppose in the above question, the local government has maintained a consistent practice of providing financial
assistance to small businesses that have been negatively impacted by natural disasters. This practice has been in place
for the past 20 years. Citizen A, who owns a small business, has recently been affected by a flood and expects to receive
financial assistance based on the established practice. The local government decides to review the financial assistance
program and, without giving any prior notice, discontinues it for small businesses affected by natural disasters. Will A
succeed in their claim for financial assistance, considering the passage provided?

(a) Yes, A will succeed in their claim, as they have a legitimate expectation based on the established practice, and
the government's abrupt discontinuation is procedurally unfair.
(b) No, A will not succeed in their claim, as legitimate expectation is not enforceable as a legal right.
(c) Yes, A will succeed in their claim, as the discontinuation of the program is unreasonable and illogical.
(d) No, A will not succeed in their claim, as legitimate expectation only applies to promises made by an authority.

170. In the hypothetical city of Menapolis, the local government has a well-established practice of offering tax breaks
to businesses that use renewable energy sources for at least 80% of their energy needs. This practice has been in place
for the past 10 years. Citizen A, who plans to start a new business using renewable energy sources, hears about the
tax breaks from a friend and intends to take advantage of the practice. However, the local government decides to
discontinue the tax breaks program without any prior notice. Will A succeed in their claim for tax breaks, considering
the passage provided?

(a) Yes, A will succeed in their claim, as the local government's decision to discontinue the tax breaks program
violates the doctrine of legitimate expectation.
(b) No, A will not succeed in their claim, as they have not yet started their business and have not had any dealings,
transactions, or negotiations with the local government.
(c) Yes, A will succeed in their claim, as the local government did not take into consideration all relevant factors
relating to the legitimate expectation of businesses using renewable energy sources.
(d) No, A will not succeed in their claim, as the local government has the authority to change policies as they see
fit.

Passage 35

The Supreme Court set aside an order passed by the Delhi High Court which held that since the compensation was not
paid to the landowners, the acquisition of the land in question was lapsed in view of Section 24(2) of the Right to Fair
Compensation and Transparency in Land Acquisition, Rehabilitation and Resettlement Act, 2013. While relying upon
the judgment of Supreme Court in Indore Development Authority v. Manoharlal and Others (2020) 8 SCC 129, the

Legal Reasoning Practice Sheet 60


CLAT POINT

division bench of Justice Abhay S. Oka and Justice Rajesh Bindal observed: “It was further submitted that the
compensation could not be paid to the recorded land owners as they never came forward to claim the same.”

The High Court relied upon the judgment of the Apex Court in Pune Municipal Corporation & Anr. v. Misirimal Solanki
& Ors. (2014) 3 SCC 183 and held that since the compensation was not paid to the landowners (the present
respondents), the acquisition in question has lapsed. Hence, the appellant approach the Supreme Court assailing the
impugned order of the High Court.

The counsel appearing for the appellants submitted before the Apex Court that the Constitution Bench judgment of
the Supreme Court in Indore Development Authority v. Manoharlal and Others (2020) 8 SCC 129 whereby earlier
judgment of this Court in Pune Municipal Corporation & Anr.’s case (supra) was overruled.

It was further submitted that the Constitution Bench in Indore Development Authority (supra) opined that the
compliance of either of the two conditions i.e., taking over of possession of the land or payment of compensation, is
good enough to sustain the acquisition. In this context, it was argued that the possession of land in dispute was taken
after the acquisition was complete.

The Supreme Court noted that the possession of the land was taken over by the Land Acquisition Collector and handed
over to Delhi Development Authority and therefore one of the conditions laid down in Indore Development Authority
(supra) has been satisfied.

“If after the process of acquisition is complete and land vest in the State free from all encumbrances with possession,
any person retaining the land or any re-entry made by any person is nothing else but trespass on the State land.”, the
Court observed.

Accordingly, keeping in view the law laid down by the Constitution Bench of the Supreme Court in Indore Development
Authority’s case (supra), the Court set aside the impugned order passed by the High Court. However, the Court held
that the respondents shall be entitled to receive compensation as per their entitlement.

171. Which judgment did the High Court rely upon while making its decision?

(a) Indore Development Authority v. Manoharlal and Others


(b) Pune Municipal Corporation & Anr. v. Misirimal Solanki & Ors.
(c) Delhi Development Authority v. Land Acquisition Collector
(d) None of the above.

172. State of Delhi decides to acquire a 100-acre land in west Delhi to construct a waste dumping plant. It contacts
the owners of the land and after all the paperwork, it is bought for 20 crores and the payment is done to the owners
via bank transfer. Meanwhile the government falls and the state goes into political crisis. 2 years later the government
decides to start the construction of new plant but the owners contend that the possession was not taken for two years
hence the right has been forfeited. Decide:

(a) they are correct as they forfeit their claim as per the land acquisition act of 1894.
Legal Reasoning Practice Sheet 61
CLAT POINT

(b) government can still claim the land.


(c) previous owners should be charged with trespass.
(d) both b and c.

173. State government decided to purchase a land and contracted its owners. The owners agreed and the government
promised to pay the price in the coming six months meanwhile taking its possession. Instalments for the first three
months came on time but then for one year they have not received any payment. So, they decided to reclaim the land
by settling on it. The government registered a case against them for the offence of trespass. Decide:

(a) The owners are right in doing so as they have not received payment thus the contract stands null and void.
(b) The government is right in registering the case against them for trespass.
(c) The land belongs to government as one of the essentials have been fulfilled to sustain the acquisition.
(d) Cannot determine.

174. Suppose in the above case, the government had not taken the possession of the land and only partly fulfilled the
payment obligation, and other facts remaining the same. Does this still constitute as an offence of trespassing?

(a) the government is right in registering the case against them for trespass.
(b) the land belongs to government as one of the essentials have been fulfilled to sustain the acquisition.
(c) the government cannot register the case for trespass against them.
(d) none of the above.

175. What did the Supreme Court note in its verdict?

(a) The possession of land was taken over by Delhi Development Authority
(b) The possession of land was taken over by Land Acquisition Collector and handed over to Delhi Development
Authority
(c) The possession of land was taken over by Land Acquisition Collector and handed over to Pune Municipal
Corporation
(d) The possession of land was taken over by Pune Municipal Corporation.

Passage 36

The Court noted that the possession of the land was taken over by the Land Acquisition Collector and handed over to
Delhi Development Authority. Therefore, the correct answer is option b.

An FIR for rape cannot be quashed under Section 482 of the CrPC on the basis of settlement between the woman and
the man and their subsequent marriage as the same does not waive the offence, the Delhi High Court has ruled. High
Court has no authority to quash FIR related to serious offences ". Act of rape is not an act against individual but an
offence against society," Justice Rajnish Bhatnagar said, while rejecting the plea of a man, an officer in the Customs
here. The plea sought to quash an FIR for rape registered against him as he has now married the victim.

"By simply entering into a compromise, charges cannot be said to have been mitigated or that the allegations levelled
by the respondent no. 2 [woman] regarding the alleged offence lost its gravity by any means. Justice Bhatnagar said.
She also gave history of sexual assault in the form of intercourse without her consent by the accused. She said her
request to the accused to meet her parents was turned down as he "asked for more time to understand each other".
The man, in his plea, had contended that he was holding a government job and his whole career is at stake if the FIR
was not quashed in view of the dispute being amicably settled between him and the woman.

Legal Reasoning Practice Sheet 62


CLAT POINT

Justice Bhatnagar remarked that the man, "being a government servant is expected to maintain high moral rectitude
and decent standard of conduct in his personal and private life and not bring discredit to his service by his
misdemeanours, in fact a government servant has all the more responsibility as far as his conduct is concerned towards
the society. Rape not only destroys the personality of the victim but also scars the mental psyche of the victim, which
remain-embedded on the mind of the victim for years together. The charges of rape are of grave concern and cannot
be treated in a casual manner," the judge said.

When the parties have reached the settlement and on that basis petition for quashing the criminal proceedings is filed.
the guiding factor in such cases would be to secure ends of justice, or to prevent abuse of the process of any court.

176. Amit and varsha were in a live in relationship for 3 years. After they broke up with each other, amit started
blackmailing varsha with their intimate photographs and videos, he also demanded some money and forced her into
sexual intercourse and in return offered to delete the pictures and videos. She gave him the money, after this she filed
an FIR against him under section 66e and 67a under IT act for violating her privacy and for rape. The case was in high
court when they told the court that they had compromised and court quashed the FIR afterwards, is the high court
correct in doing so:

(a) Yes, as HC has no power to quash FIR.


(b) No, as this is a compoundable offence and could be only quashed by Supreme Court.
(c) Yes, as rape is a serious offence.
(d) None of the above.

177. Tola and mola were two best friends who wanted to marry pinky. Pinky decided to marry mola and tola enraged
by this pushed mola and his head fell on a rock which caused him serious injuries. Tola was charged for attempt to
murder and the case has reached high court. Can the FIR be quashed if both the parties have come to a compromise:

(a) Yes, as this is not a case of rape.


(b) No, as this is a serious offence.
(c) No, as this is a crime against society.
(d) Both b and c.

178. Mr. Vyas is an IAS officer who is charged with outraging the modesty of his secretary Ms. Sheila. It was alleged
that he in an inebriated state touched her improperly. The case was in high court when Mr. Vyas offered an apology
along with some compensation to Ms. Sheila and asked then court to quash the FIR keeping in view the dignity of the
post. Decide:

(a) The FIR will be quashed.


(b) The high court may reserve its order and announce it some other day after discussion.
(c) The FIR will not be quashed.
(d) The FIR will be quashed if Ms. Sheila consents to it.

179. Which of the following is not considered as a grave offence?

(a) Murder.
(b) Rape.
(c) Dacoity.
(d) None of the above.

Legal Reasoning Practice Sheet 63


CLAT POINT

180. Ramani yadav, a journalist, has filed a case of rape against mohanlal, who was a very renowned politician. while
she was interviewing him at his farmhouse. While the Court A was hearing this case, mohanlal has filed a case of
criminal defamation against her for tarnishing his public image, which the Court B has accepted subsequently. Before
both the Court could have finalized their ruling, they both entered into a compromise that they will take their
respective cases back. Hence. Ramani filed an application of Section 482 of the CrPC before the High Court to quash
their cases as both entered into a compromise. Decide:

a) The application for quashing her case can be accepted by the Court because the parties have come into a
compromise.
b) The application for quashing her case can be accepted by the Court because the case of rape filed by one of
the parties was frivolous in nature.
c) The application for quashing her case cannot be accepted by the Court because the alleged allegations on the
person was not of private nature.
d) The application for quashing her case cannot be accepted by the Court because rape is a serious offence.

Passage 37

A conversion is any act of wilful interference, without any lawful justification, in a manner which is inconsistent with
the right of another, whereby that other is deprived of the use and possession of the chattel. The expression ‘wilful
interference’ is used for describing the element of intention referring to the intentional commission of the act resulting
in conversion. If a person deals with a chattel in a manner which is of such a nature that is necessarily inconsistent with
the rights of the plaintiff, such dealings will be considered as intentional and will amount to conversion even if he did
not know of the right held by the plaintiff and honestly believed that he was entitled to do so.

For example, an auctioneer is held liable for conversion even if he honestly believed that the goods which are being
auctioned belongs to the seller and not to the plaintiff. Conversion can be committed in various ways but the main link
in every act that constitutes conversion is that it consists of dealings with goods by assertion of rights which is either
inconsistent with the rights of another or unjustifiable denial of the rights of another in them. If the defendant has
never been in physical possession of the goods but his act amounted to an absolute of the plaintiff’s right on them,
then it will be termed as conversion.

If any individual without the proper authority takes possession of another man’s goods with the sole intention of
asserting dominion over them is guilty of conversion. The reason being that the act will be inconsistent in regards to
the general right of dominion which the owner of the chattel, who is entitled to the use of it at all times and in all
places, has in it. An act that is constituted by taking the goods but without any intention of exercising permanent or
temporary dominion can be termed as trespass but not as conversion. If there is a wrongful taking, even though such
an act was done under a mistaken but honest supposition of being lawfully entitled, or with the intention of benefiting
the true owner it does not make any difference.

[Source: Extracted with edits and revisions from Conversion, The Basic Tort, https://www.stimmel-
law.com/en/articles/conversion-basic-tort ]

181. Shrikant, a patient suffering from a stomach ailment, approached Sappolo Hospital for treatment. The doctors
diagnosed him with appendicitis and removed his appendix. Unbeknownst to Shrikant, his appendix contained unique
cells that the hospital used to develop highly valuable drugs. Years later, Shrikant discovered the hospital's commercial
success using his cells and demanded a share in the profits. Which of the following options is correct?

Legal Reasoning Practice Sheet 64


CLAT POINT

(a) Sappolo Hospital must share its profits with Shrikant because they used his unique cells without obtaining
explicit consent.
(b) Sappolo Hospital can choose to share its profits with Shrikant on an ex gratia basis, but it is not legally obligated
to do so as there was no direct harm caused to him.
(c) Sappolo Hospital is not required to share its profits with Shrikant because his appendix was removed lawfully,
and he benefitted from the removal without any deprivation of use or possession.
(d) Sappolo Hospital should establish a trust to benefit Shrikant, as the hospital's actions indirectly resulted in a
conversion of his unique cells.

182. Tobey allowed Andrew to store his motor-car in Tobey's hotel yard without any payment, intending it to be for a
short duration. However, Andrew's car remained in the yard for several years, becoming an obstacle to the hotel's
operations and suffering significant damage due to prolonged exposure to the elements. Despite numerous attempts
to contact Andrew, Tobey received no response. Faced with the dilemma, Tobey decided to repair and renovate the
car, which he then sold at an auction. Andrew discovered this and filed a lawsuit against Tobey for damages due to
conversion and detinue of the car. Decide:

(a) Andrew will not succeed because Tobey's actions were justifiable, given the car's negative impact on hotel
operations and the absence of communication from Andrew.
(b) Andrew will succeed, but Tobey's liability will be limited to the car's value before renovation since the repairs
were initiated by Tobey without Andrew's consent.
(c) Andrew will succeed, but must compensate Tobey for the repair expenses incurred to increase the car's value,
which in turn will be deducted from any awarded damages.
(d) Andrew will not succeed as Tobey's repeated attempts to contact Andrew and subsequent actions in repairing
and selling the car nullified Andrew's claim over the car.

183. Thanos who is the owner of a diamond ring, entrusted it to Thor (the agent) who undertook to try to sell it on his
behalf. Thanos was to receive the price specified by him and if there was any surplus it was to be received by Thor with
the condition that the ring has to be returned to the Thanos if not sold within seven days. Thor sold the ring after the
seven days had elapsed, representing himself as the owner of the ring, and sold it to the Natasha, for a price which
was one-third of the price mentioned by Thanos, who bought it in good faith and re-sold it. Thor was subsequently
convicted of the larceny of the ring as a bailee. Thanos sued Natasha for wrongful conversion of the ring. Decide.

(a) Thanos will not succeed in the suit because Thor is right in selling the ring to Natasha as it was his work and
hence, Natasha is the rightful owner of the ring.
(b) Thanos will succeed in the suit because Thor was not in the position of agent when he sold the ring and hence,
he’s liable for the losses.
(c) Thanos will succeed in the suit because Natasha was negligent as she didn’t confirm the employability
beforehand and hence has to return the ring back to Thanos.
(d) Thanos will not succeed in the suit because he cannot ask for ring back but can get proportional amount of
money as damage from Thor.

184. Jane, an art collector, mistakenly believes that a valuable painting she found in a storage unit she purchased at
auction belongs to her. She decides to sell the painting to recover some costs associated with the storage unit.
However, the painting actually belongs to Robert, who was unaware that his painting had been misplaced in the
storage unit. Jane hires an auctioneer, George, to sell the painting. George honestly believes that Jane has the right to
sell the painting. Robert discovers the auction and sues Jane and George for conversion. Decide:

(a) Jane and George are not liable for conversion, as they both honestly believed that Jane had the right to sell the
painting.
(b) Only Jane is liable for conversion, as she initiated the sale of the painting without confirming her ownership
rights.

Legal Reasoning Practice Sheet 65


CLAT POINT

(c) Both Jane and George are liable for conversion, as their actions interfered with Robert's rights to the painting,
despite their honest belief.
(d) Neither Jane nor George is liable for conversion, as they acted with the intention of benefiting the true owner
by recovering the costs associated with the storage unit.

185. Mary discovers a painting in her attic that she believes is her property. She decides to sell the painting to an
auction house. Unbeknownst to Mary, the painting actually belongs to her neighbor, John. The auction house sells the
painting to an art collector. What is the legal liability of the auction house in this scenario?

(a) The auction house is not liable, as they honestly believed the painting belonged to Mary.
(b) The auction house is liable for conversion, as they dealt with the painting in a manner inconsistent with John's
rights.
(c) The auction house is liable for trespass, as they took possession of the painting without exercising dominion
over it.
(d) The auction house is not liable, as Mary's initial possession of the painting was lawful.

Passage 38

Marriage being essentially a consensual relationship, the consent must be free and intelligent; this pre-supposes a
person with mental competence of a level that enables one to make an intelligent choice. Mental deficiency which
hampers the making of such a choice should therefore constitute a ground for annulment of a marriage entered into
by the person mentally deficient. Apart from this there can be mental abnormality which supervenes after marriage.
Conjugal harmony and happiness would be substantially impaired if either of the parties suffers from such abnormality
which has intervened after marriage. Such abnormality could be a ground for divorce as distinct from annulment.

Mental abnormalities comprise of various forms and degrees and the prevailing matrimonial laws in India provide a
relief either by way of divorce or annulment. While the Hindu Marriage Act, 1955, the Special Marriage Act, 1954 and
the Indian Divorce Act, 1869 as amended in 2001, provide for both the reliefs, under different situations though, the
Parsi Law provides only for annulment.

The statutory phraseology adopted to indicate abnormality in general, or various species thereof, varies such as
‘insanity’, ‘idiocy’, ‘lunacy’, ‘unsound-ness of mind’, ‘mental disorder’, ‘psychopathic disorders’, and so on.

Under s. 5(ii) of the Hindu Marriage Act, 1955, if either party to the marriage is: (a) incapable of giving a valid consent
to it in consequence of unsoundness of mind; or (b) though capable of giving a valid consent, has been suffering from
mental disorder of such a kind or to such an extent as to be unfit for marriage and the procreation of children; or (c)
has been subject to recurrent attacks of insanity, then that marriage shall be voidable and may be annulled by a decree
of nullity. It may be pointed out here, that prior to 1976, the clause was differently worded and the marriage could be
avoided, if the opposite party was an idiot or a lunatic at the time of the marriage. The words ‘idiot’ or ‘lunatic’ were
not defined, and therefore, when required, reference was made to s. 3(5) of the Indian Lunacy Act, 1912. Under that
section, a lunatic means an idiot or a person of unsound mind. The position is now different, and if a person is not fit
to give a valid consent owing to unsoundness of mind, or even if capable of giving a valid consent, is suffering from
such a mental disorder so, as to be unfit for marriage and procreation, the marriage may be annulled. In 1976,
‘recurrent attacks of insanity or epilepsy’ was included.

Legal Reasoning Practice Sheet 66


CLAT POINT

[Source: Extracted with edits and revisions from "NULLITY OF MARRIAGE IN MODERN HINDU LAW",
https://eprints.soas.ac.uk/33926/1/11015651.pdf ]

186. Mr. Y and Mrs. X, both devout Hindus, were married in a lavish ceremony attended by their extended families
and friends. Following their marriage, they moved into a beautiful house and had a son together. Over time, however,
Mrs. X began to exhibit erratic behavior. She would physically abuse Mr. Y and their son, experience extreme mood
swings, cry and scream at night, and even attempt suicide on several occasions. They consulted numerous doctors,
who concluded that Mrs. X was mentally unstable and suffered from recurring epileptic attacks. In this situation, and
considering the provisions of the Hindu Marriage Act, can Mr. Y's marriage be annulled?

(a) Mr. Y's marriage can be annulled under section 5(iii), since Mrs. X's condition is covered by the Hindu Marriage
Act.
(b) Mr. Y's marriage can be annulled solely due to Mrs. X's epilepsy, as it impairs her ability to function in a marital
relationship.
(c) Mr. Y's marriage cannot be annulled, as the law does not permit annulment for mental health issues after
marriage.
(d) Both A and B

187. Mr. A and Mrs. B, both practicing Hindus, had been happily married for five years and were blessed with a lovely
son. After celebrating their fifth wedding anniversary, Mr. A was diagnosed with schizophrenia, which resulted in
severe seizures at random times throughout the day. During these seizures, he would lose control of his emotions and,
unfortunately, physically abuse Mrs. B and their son while screaming and shouting uncontrollably. Mrs. B consulted
several doctors, therapists, and spiritual advisors in an attempt to find a solution to Mr. A's condition, but the situation
only worsened over time. Considering the circumstances and the provisions of the Hindu Marriage Act, what is the
status of their marriage under Section 5(iii)?

(a) Valid, because Mr. A's condition developed after their marriage and has no bearing on their marital status.
(b) Void, as Mr. A's schizophrenia and seizures render their marriage invalid under the Hindu Marriage Act.
(c) Voidable, since Mr. A's condition falls under the provisions of Section 5(iii), allowing for annulment if pursued.
(d) None of the above, as the marriage status cannot be determined without further information.

188. Mr. P, an elderly man, had been suffering from extreme anger and depression issues due to significant losses in
his business. As a result, he sometimes argued with his wife, Mrs. Q, who was a successful entrepreneur herself. Mr. P
decided to consult a renowned psychiatrist, Dr. R, to evaluate his mental health. After a series of comprehensive tests,
Dr. R confirmed that Mr. P was not suffering from any mental health issues and attributed his anger and depression to
situational stressors. The couple, both practicing Hindus, went on a vacation to improve their relationship, but the
arguments persisted. Can Mrs. Q get her marriage annulled under Section 5(iii) of the Hindu Marriage Act?

(a) Yes, because Mr. P's extreme anger and depression issues may still impact their marital life.
(b) No, since Dr. R's diagnosis confirmed that Mr. P was not suffering from any mental health conditions that would
warrant annulment under Section 5(iii) of the Hindu Marriage Act.
(c) Facts insufficient, as there is no information about any pre-existing mental health conditions in either Mr. P or
Mrs. Q.
(d) Yes, because Mr. P's depression issues may qualify as a mental disorder under the Hindu Marriage Act.

189. Priya and Amit were married under the Hindu Marriage Act, 1955. Priya, unbeknownst to Amit, has been
experiencing recurrent attacks of epilepsy. These episodes cause occasional disruption in their married life. Amit learns
about Priya's condition and wishes to seek legal recourse. Which of the following legal actions can Amit take in this
situation?

(a) Amit can seek annulment of the marriage, as Priya's epilepsy qualifies as a mental disorder.
Legal Reasoning Practice Sheet 67
CLAT POINT

(b) Amit can seek divorce, as Priya's epilepsy developed after their marriage and affects their conjugal harmony.
(c) Amit cannot seek any legal relief, as Priya's epilepsy does not render her unfit for marriage.
(d) Amit can seek annulment of the marriage, as Priya has been subject to recurrent attacks of epilepsy and the
marriage is voidable under the Hindu Marriage Act, 1955.

190. Sara and Rajesh were married under the Hindu Marriage Act, 1955. Six months after their wedding, Rajesh starts
to show signs of mental disorder, which subsequently deteriorates to a level where he is unfit to take care of himself
and maintain a healthy marital relationship. Sara wants to end their marriage. What legal relief can Sara seek in this
situation?

(a) Sara cannot seek any legal relief, as Rajesh's condition developed after their marriage.
(b) Sara can seek annulment of the marriage, as Rajesh's mental disorder makes him unfit for marriage.
(c) Sara can seek divorce, as Rajesh's mental disorder developed after their marriage and substantially impairs
their conjugal harmony.
(d) Sara can seek annulment of the marriage, as Rajesh's condition is considered a form of lunacy.

Passage 39

Balance between one person’s right to freedom of speech and another’s right to protect their good name.” Any
intentional false communication, either written or spoken , that harms a person’s reputation; decreases the respect,
regard or confidence in which a person is held; or induces disparaging, hostile or disagreeable opinions or feelings
against a person is known as defamation. Defamation is the act of making untrue statements about another which
damages his/her reputation. It is a statement that injures someone’s reputation. Defamation is the act of saying false
things in order to make people have a bad opinion of someone.

Defamation may be defined as a communication to some person, other than the person defamed, of the matter which
tends to lower the plaintiff in the estimation of right-thinking persons or to deter them from associating or dealing
with him. Defamation is a wrong done by a person to another’s reputation by words, written or spoken, sign or other
visible representation. In the words of Dr. Winfield “Defamation is the publication of a statement which tends to lower
a person in the estimation of right-thinking members of the society, generally or, which tends to make them shun or
avoid that person.” Defamation is of two kinds; Libel and Slander. If the statement is made in writing and published in
some permanent and visible form, then the defamation is called libel. Whereas, if the statement is made by some
spoken words, then the defamation is called slander.

If the defendant proves that the defamatory statement is true, no action will lie for it, even if the statement is published
maliciously. It is not necessary to prove that the statement is literally true, it is sufficient if it is true in substance. Law
makers have decided that one cannot sue for defamation in certain instances when a statement is considered
privileged. Whether a statement is privileged or unprivileged is a policy decision that rests on the shoulders of the
lawmakers.

[Source: Extracted with edits and revisions from Law of Torts: Defamation,
https://www.lkouniv.ac.in/site/writereaddata/siteContent/202004050638207977Satish_Chandra_Defamation_in_La
w_of_Torts.pdf ]

191. George, who is James's patient, is unhappy with the treatment he is receiving from the doctor. He discontinues
the treatment and decides to visit another doctor, Dr. Z. Surprisingly, his illness disappears by itself after some time.
Legal Reasoning Practice Sheet 68
CLAT POINT

George is very upset with James because the treatment cost him a lot of time and money, and he feels that he suffered
unnecessarily. He writes a detailed letter to James accusing him of cheating, misdiagnosing his condition, and
deliberately extending his treatment for financial gain. He also accuses James of causing his health to worsen. George
shares the letter with his friends and family. James shows this letter to his lawyer and a lawsuit for defamation is filed.
Will it succeed?

(a) James will succeed because he is a doctor and has more knowledge about his field than George. George should
not have written the letter and should have approached James with a more positive attitude.
(b) James will not succeed because such incidents are of trivial nature and occur frequently. The court will not
consider such a case.
(c) James will succeed because George has defamed James due to a personal grudge against him as he has spent
a lot of money on treatment.
(d) James will not succeed because George only shared the letter with friends and family, which does not
constitute defamation.

192. Ronny is Mr. Lion's servant. One morning on the way to his master's place, Ronny stops for a cup of coffee at an
empty shop. While waiting for his order, he overhears a conversation between Vijay and the shop owner discussing a
recent burglary. Ronny believes Vijay is involved in the crime and, upon seeing him leave, he loudly calls Vijay a liar
and a thief in front of the shop owner and a group of customers. Vijay sues Mr. Lion, claiming that there was defamation
and it was carried on in the course of employment. Will he succeed?

(a) Vijay will not succeed because he is a liar and a thief, and Ronny has just stated the facts which are true.
(b) Vijay will succeed because he has been defamed by Ronny in a coffee shop in front of the public.
(c) Vijay will not succeed because Ronny was the one who defamed him and he was out of the course of
employment at the time of drinking coffee and Mr. Lion cannot be held liable for this.
(d) Vijay will not succeed because Ronny made the statements in private.

193. During a charity event organized by Vicky, a well-known event planner, he meets Katrina, an artist whose paintings
were being showcased at the event. Vicky, captivated by Katrina's talent and beauty, walks up to her and says in front
of several attendees, "You are a thief, since you have stolen my heart." Later, Katrina, feeling humiliated by the public
attention, files a suit for defamation against Vicky. Will she succeed?

(a) Katrina will succeed because Vicky insulted and defamed her for theft in a public setting, thereby tarnishing
her reputation.
(b) Katrina will succeed because Vicky's intentions were malicious, and he intended to humiliate her in front of a
large audience.
(c) Katrina will not succeed because the statement was not defamatory in nature but rather an expression of
Vicky's admiration for her.
(d) Katrina will succeed because the statement made by Vicky has caused her emotional distress.

194. In an exclusive interview with a well-respected news outlet, famous author Karan claimed that Priya, a newly
published author, had plagiarized a significant portion of her debut novel from his earlier work. Karan provided some
textual evidence that seemed to support his claim, but upon further investigation, it was revealed that the similarities
were purely coincidental, and no plagiarism had taken place. Priya, feeling the negative impact on her reputation,
decides to sue Karan for defamation. Will Priya succeed?

(a) Priya will succeed because Karan's claim of plagiarism is false, and it has caused damage to her reputation.
(b) Priya will not succeed because Karan believed his claim to be true and provided evidence to support it, making
his statement true in substance.
(c) Priya will succeed because Karan's statement was made in a well-respected news outlet, making it libel.
(d) Priya will not succeed because Karan's statement, though false, was made in good faith and without malicious
intent.
Legal Reasoning Practice Sheet 69
CLAT POINT

195. Shanti Lal, an aspiring writer, published a short story on his personal blog about a fictional character named
Ravinder who cunningly conned people and amassed crores of money. Shanti Lal used excessive and vivid details to
make his story more engaging and to create awareness about frauds. In a small town, a man named Ravinder Ahuja,
who is a well-respected businessman, came across the story and felt that the character shared an uncanny
resemblance to his own life. Consequently, he believed that the story has caused injury to his image and decided to
sue Shanti Lal for libel. Decide.

(a) Ravinder Ahuja will succeed because Shanti Lal should have clarified in the article that the character was not
based on Ravinder Ahuja, but was a fictional character.
(b) Ravinder Ahuja will not succeed because the article was based on a fictional character and there are many
individuals named Ravinder, apart from him.
(c) Ravinder Ahuja will not succeed because the article only mentioned the name "Ravinder" and not "Ravinder
Ahuja." Since his full name was not mentioned, he cannot succeed in the suit due to lack of proper
identification.
(d) Ravinder Ahuja will succeed because Shanti Lal did not seek permission from all the individuals named Ravinder
before publishing the story.

Passage 40
According to Section 10 free consent is an essential requirement of a contract. Section 14 defines "free consent". S.
14. "Free consent" defined.— Consent is said to be free when it is not caused by: (1) coercion, as defined in Section
15, or (2) undue influence, as defined in Section 16, or (3) fraud, as defined in Section 17, or (4) misrepresentation, as
defined in Section 18, or (5) mistake, subject to the provisions of Sections 20,21 and 22. Consent is said to be so caused
when it would not have been given but for the existence of such coercion, undue influence, fraud, misrepresentation
or mistake. Where consent to an agreement is caused by coercion, undue influence, fraud or misrepresentation, the
agreement is a contract voidable at the option of the party whose consent was so caused.' If, for example, a person is
induced to sign an agreement by fraud, he may, on discovering the truth, either uphold the contract or reject it. If he
confirms it, the contract becomes binding on both the parties.

It is a contract which is enforceable at the option of only one of the parties, namely, the party whose consent was not
free. Giving the meaning of a voidable contract, Section l (i) says that "an agreement which is enforceable by law at
the option of one or more of the parties thereto, but not at the option of the other or others, is a voidable contract.
Consent is said to be caused by coercion when it is obtained by pressure exerted by either of the following techniques;
(1) committing or threatening to commit any act forbidden by the Indian Penal Code; or (2) unlawfully detaining or
threatening to detain any property.

It is clear that coercion as thus defined implies a committing or threatening to commit some act which is contrary to
law. When a criminal prosecution is instituted against a person and such person fearing the result of the prosecution
enters into an agreement in favour of the complainant in consideration of his abandoning the prosecution, it cannot
be held simply upon these facts that the consent of such person was caused by coercion." "To threaten a criminal
prosecution is not per se an act forbidden by the Indian Penal Code. Such an act could only be one forbidden by the
Indian Penal Code if it amounted to a threat to file a false charge.

[Source: Extracted with edits and revisions from, Free Consent under the Indian Contract Act, 1872, Law Circa Free
Consent under the Indian Contract Act, 1872: Everything you need to know - Law Circa. ]

Legal Reasoning Practice Sheet 70


CLAT POINT

196. Mr. X, a wealthy businessman, is facing financial troubles and is emotionally distressed. In an attempt to save his
business, he threatens to commit suicide if his wife, Mrs. Y, and son do not sign a document transferring their inherited
properties to his brother. They sign the document out of fear for Mr. X's life. Is this considered coercion as per the
Indian Contract Law?

(a) This is not coercion as per the Indian Contract Law, as the emotional distress of Mr. X does not qualify as a
threat.
(b) Threat of suicide is coercion as per the Indian Contract Law, as it is an act forbidden by the Indian Penal Code.
(c) Since Mr. X's threat is not directly related to an unlawful act and is based on emotional distress, it does not
amount to coercion under Indian Contract Law.
(d) The threat of suicide can be considered coercion only if Mr. X was aware of the legal implications of his actions.

197. A minor, Rajesh, having borrowed money under two mortgage deeds, agreed to a compromise decree although
the mortgages were void due to his minor status. Subsequently, he pleaded that he entered into the compromise
because he was threatened with prosecution for falsely misrepresenting his age to the lenders and that this amounted
to coercion. It is important to note that he had actually misrepresented his age when obtaining the loans. Now, he
wishes to challenge the compromise decree. Can the threat of prosecution for false age misrepresentation amount to
coercion under Indian Contract Law?

(a) It is not coercion because threatening prosecution for a genuine criminal act, like Rajesh's age
misrepresentation, cannot be considered coercion.
(b) It is coercion because any threat of criminal prosecution, regardless of the truth of the allegations, is coercion
per se under Indian Contract Law.
(c) It is not coercion since the threat was based on Rajesh's truthful misrepresentation of age, and Indian Contract
Law does not consider such threats as coercion.
(d) The threat of prosecution can be considered coercion only if the lenders were aware that the threat itself would
be considered coercion under Indian Contract Law.

198. A, while on board an Indian Ship named "Ocean Explorer" in the territorial waters of India, causes B, a fellow
passenger, to enter into an agreement for purchasing A's property located in a remote village by using an act
amounting to criminal intimidation under the Indian Penal Code. A few weeks later, A sues B for breach of contract at
the Calcutta High Court, claiming that B failed to complete the property purchase. Can A's conduct, involving criminal
intimidation to induce B to enter the agreement, amount to coercion under Indian Contract Law?

(a) A's conduct of causing B to enter into the agreement by employing criminal intimidation constitutes coercion
under Indian Contract Law.
(b) A's conduct does not amount to coercion since the intimidation occurred on a ship and not on land, making it
an exceptional circumstance.
(c) A's conduct cannot be considered coercion as the agreement was related to a property transaction, and
property transactions are exempt from coercion claims.
(d) A's conduct does not amount to coercion as B should have been aware of the consequences of entering an
agreement under intimidation.

199. Arvind, a successful entrepreneur, finds himself facing criminal charges after Ram, a dishonest police officer,
fabricates evidence against him. Ram offers Arvind a deal: if Arvind signs a contract agreeing to donate a large sum of
money to Ram's favorite charity, Ram will drop the charges. Arvind, having no other option, agrees to the deal and
signs the contract. Later, after the charges are dropped, Arvind learns about Ram's deception and refuses to fulfill his
obligation under the contract. Ram decides to sue Arvind for breaching the contract. Can Arvind claim that the contract
is voidable due to coercion under Indian Contract Law?

(a) Arvind can claim the contract is voidable due to coercion, as the contract was signed under the threat of false
criminal charges.
Legal Reasoning Practice Sheet 71
CLAT POINT

(b) Arvind cannot claim the contract is voidable, as the donation to a charity cannot be considered an act forbidden
by the Indian Penal Code.
(c) The contract is voidable, but Arvind must prove that Ram had an intent to file false charges and fabricate
evidence to establish coercion.
(d) Arvind cannot claim the contract is voidable, as the contract was entered voluntarily under the belief that the
charges were genuine.

200. Kumar, a well-known businessman, fears that Rajesh, a determined investigative journalist, will file a false criminal
charge against him due to their long-standing dispute. To prevent this from happening, Kumar coerces Rajesh into
signing a contract that forbids Rajesh from publishing any articles about Kumar for the next five years. Later, Rajesh
discovers evidence of Kumar's involvement in illegal business activities and decides to publish the story. Kumar sues
Rajesh for breaching the contract. Can Rajesh claim that the contract is voidable due to coercion under Indian Contract
Law?

(a) Rajesh can claim the contract is voidable due to coercion, as the contract was signed under the threat of a false
criminal charge.
(b) Rajesh cannot claim the contract is voidable, as coercion only applies when the act is forbidden by the Indian
Penal Code.
(c) The contract is voidable, but Rajesh must prove that Kumar had an intent to file a false charge to establish
coercion.
(d) Rajesh cannot claim the contract is voidable because he willingly entered into the contract, despite the threat.

Legal Reasoning Practice Sheet 72


CLAT POINT

Answer and Explanations


1. Answer- c)
Although Samar wanted to sell his bike and agreed to the deal, the use of a gun to obtain his consent clearly indicates
that it was not given freely. Thus, this agreement can be categorized as a voidable contract under the principle
described in the passage. Option d) is incorrect because the issue at hand is not the price of the bike, but rather the
manner in which the consent was obtained.

2. Answer: The correct option is (b). Even though the gun was fake, Samar was under the impression that it was real,
and therefore, he gave his consent under duress. Since he did not have knowledge of the gun's true nature, his consent
was not free. According to the principles outlined in the passage provided, a contract obtained under such
circumstances is voidable at the option of the party whose consent was obtained. Option a) It is a valid contract: This
option is incorrect because there was no free consent from Samar, which is a fundamental requirement for a contract
to be considered valid. Option c) It is an agreement but a voidable contract as the bike was sold at the cost of 1 Rupee:
This option is incorrect because the price of the bike was not the issue here. The fact that Samar's consent was
obtained under a mistake of fact is what makes the contract voidable. Option d) It is a contract as Samar wanted to
sell his bike: This option is incorrect because the desire to sell the bike alone does not make a contract valid. A valid
contract requires free and informed consent from all parties involved, which was not the case here.

3. Answer: a)
The most appropriate answer is option (a). While Dr. Amitabh may have had good intentions, he was in a position to
control Bacchan's will due to his expertise and the dire situation Bacchan found himself in. As a result, Bacchan's
consent was not entirely free, and the contract can be considered voidable. Option (b) is incorrect because good
intentions do not necessarily justify using undue influence to persuade someone to undergo a risky surgery. Option
(c) is also incorrect because a person's consent may not be entirely free even if they appear to give it voluntarily.
Finally, option (d) is incorrect because undue influence usually arises from a specific relationship between the parties,
such as that between a parent and child or a guardian and ward.

4. Answer: a)
Explanation: In the given scenario, Aman deliberately concealed the mental ailment of the horse, which he was aware
of, and sold it to Amar without any disclosure. Such an act falls under the category of fraud, which can be defined as
an intentional misrepresentation of facts with the aim of gaining an unfair advantage over another person. Option b)
is incorrect because the seller cannot escape liability by merely not making positive statements, and it is their
responsibility to disclose all material facts that may affect the value of the product. Option c) is also incorrect because
even if the buyer fails to conduct a thorough inspection, the seller cannot escape liability for fraud if they have
intentionally concealed any material fact. Option d) is incorrect because the intention of the seller is not the only
criterion to determine fraud; their knowledge of the material fact is also crucial.

5. Answer - b)
Explanation: Undue influence is the use of excessive pressure by one party on the other party to enter into a contract.
In this case, Rahul is a struggling artist and Rohit is a wealthy businessman. Rahul uses this power imbalance to pressure
Rohit into buying the artwork at an inflated price. Rohit is afraid of damaging his friendship with Rahul and losing out
on the artwork, which further contributes to the undue influence exerted by Rahul. Therefore, this situation falls under
the principle of undue influence. Coercion is the use of force or threats to compel a person to enter into a contract.
There is no use of force or threats in this situation, hence coercion is not applicable. Fraud is the intentional or wilful
misrepresentation of facts by one party to another party. Rahul did not misrepresent any facts to Rohit, hence fraud
is not applicable. Finally, it cannot be said that Rohit agreed to the inflated price of his own free will, as he was
subjected to undue influence by Rahul.

Legal Reasoning Practice Sheet 73


CLAT POINT

6. Answer: c) The contract is voidable at the option of the buyer since the misrepresentation was material to the
contract. According to Section 18 of the Indian Contract Act, misrepresentation means misstatement of a fact material
to the contract. Here, the seller's false claim that the laptop had never been repaired or damaged was material to the
contract and induced the buyer's consent to purchase the laptop. The buyer can choose to void the contract as the
consent was induced by misrepresentation.

7. Explanation: The passage states that misrepresentation means misstatement of a fact material to the contract, and
a statement is said to be warranted by the information of the person making it when he receives the information from
a trustworthy source. In this case, B obtained the information about C from L, who was not a direct source. Therefore,
B was not warranted to assert such information, and hence it is not misrepresentation. Option A is incorrect because
it wrongly states that the information was false, whereas it was obtained from an unreliable source. Option B is
incorrect because it suggests that B believed the information to be true, whereas the passage does not mention B's
belief. Option D is incorrect because it combines options A and B, both of which are incorrect.

8. Ans. C
Explanation: The correct answer is c, as the seller made an innocent misrepresentation by unknowingly
misrepresenting the facts about the land. Therefore, the buyer has the option to void the contract. Option a is incorrect
as there was no fraudulent intent by the seller. Option b is incorrect as it was not a mutual mistake, and option d is
incorrect as the seller had a duty to disclose any known issues with the land.

9. Ans - Option a) is the correct answer since it is an example of positive assertion of an untrue fact, which is not
warranted by the information of the person making it. Option b) is a case of coercion, which involves the use of force
or threats to induce someone to enter into an agreement. Option c) is a case of fraud since it involves the deliberate
use of false information to induce someone to enter into an agreement. Option d) is a case of extortion, which involves
the use of force or threats to obtain something from someone.

10. Ans. D)
Option a) lacks the essential requisite of having information and believing it to be true. Option b) lacks the requisite
of having received information from a credible source. Option c) lacks the requisite of believing the asserted fact to
be true. Option d) fulfils all the essential requisites and is not an example of misrepresentation.

11. Ans – a
Option a is the right answer because the plaintiff acted on the instructions of the defendant, whom he believed to be
owner of the horses. Since he was not the owner, he had to indemnify the plaintiff. Thus, option b is not valid. Option
c is incorrect because the plaintiff had no inherent duty to check whether he was the owner of the horses.

12. Ans – d
Both options b and c are correct because there was an implied promise for indemnity between them which was
evidenced by the fact that plaintiffs had demanded an indemnity bond. Option a is wrong because there was an implied
promise of indemnity between the plaintiffs and defendants.

13. Ans – b
As per the principles mentioned in the passage, a contract of indemnity only covers losses caused by the promisor or
by any other person, not losses arising from accidents like fire or perils of the sea. Loss must be caused by some human
agency. In this case, the loss was caused due to the negligence of Alpha Pvt. Ltd. employees and not due to any act or
omission of Beta Pvt. Ltd. Therefore, Beta Pvt. Ltd. is not liable to indemnify Alpha Pvt. Ltd. for the loss. Option a is
wrong because the loss was caused due to the negligence of Alpha Pvt. Ltd. Option c is also wrong as the contract
between the parties clearly mentions indemnity. Option d is wrong as the theft was not caused due to the negligence
of Beta Pvt. Ltd.
Legal Reasoning Practice Sheet 74
CLAT POINT

14. Answer: b
Explanation: The contract between X and Y included a clause for indemnification against loss caused by fire or other
accidents. Since the loss was caused by fire, X can recover damages from Y. Option a is wrong because the passage
states that loss caused by accidents like fire is covered under the English definition of indemnity. Option c is wrong
because the clause for indemnification is not too broad as it specifically mentions loss caused by fire or other accidents.
Option d is wrong because the contract between X and Y does not state that Y must be directly responsible for causing
the fire.

15. Ans: d
According to Section 124 of the Indian Contract Act, a contract of indemnity is a contract by which one party promises
to save the other from loss caused to him by the conduct of the promisor himself, or by the conduct of any other
person. In this case, the contract between ABC Pvt. Ltd. and XYZ Pvt. Ltd. specifically states that ABC Pvt. Ltd. will
indemnify XYZ Pvt. Ltd. against any loss arising from the supply of goods. However, the loss was caused due to an
accident during the transportation of goods. Therefore, ABC Pvt. Ltd. is liable to indemnify XYZ Pvt. Ltd. only if it is
proven that the loss was caused due to the conduct of the promisor or any other person. Option a is incorrect because
indemnity covers loss caused by the conduct of the promisor or any other person, not just loss caused due to a
deliberate act. Option b is incorrect because the contract already states that ABC Pvt. Ltd. will indemnify XYZ Pvt. Ltd.
against any loss arising from the supply of goods. Option c is incorrect because the fact that the loss was caused during
the transportation of goods is not sufficient to establish liability for indemnity and they should be held liable unlike
what the option says.

16. Answer: c
The agreement is void because it involves an illegal activity (testing on humans without informed consent), which
cannot be separated from the legal part of the work. As per Section 2(g) of the Indian Contract Act, an agreement not
enforceable by law is said to be void. Additionally, Section 24 of the Act states that any agreement, where the
consideration or object of the agreement is illegal, is void. Therefore, the agreement between A and B is void, as it
involves both legal and illegal activities that cannot be separated. Option a is incorrect as the illegal and legal parts
cannot be separated, making the entire agreement void. Option b is incorrect as even if A is a renowned scientist,
testing on humans without their informed consent is illegal. Option d is incorrect because B cannot simply choose to
exclude the illegal part of the agreement as it still involves an illegal activity.

17. Ans – d
Since the contract was for partnership in sale of opium and ganja, which is an illegal object, the contract between Mr.
X and the new entrant is void. Hence both option b and c are correct. Hence option d is the right answer. Option a is
wrong because the sale of ganja is illegal as provided in the facts.

18. Answer:
b) If the agreement is entered into with the purpose of preventing a party from exercising their legal rights or remedies.
Explanation:
Section 28 of the Indian Contract Act states that any agreement that restricts a person's right to enforce their legal
rights in a court of law or places an obligation on them to not exercise their legal remedies would be considered void.
Therefore, an agreement in restraint of legal proceedings would be void if it is entered into with the purpose of
preventing a party from exercising their legal rights or remedies. Options a, c, and d are incorrect as they do not fully
capture the principles outlined in the passage.

19. Ans. (b)


Explanation: Option 2 is considered a void agreement because it is an agreement in restraint of legal proceedings,
which is declared void under Section 28 of the Indian Contract Act. Options 1, 3, and 4 are valid contracts as they
satisfy the essential elements of a contract, i.e., offer, acceptance, consideration, and lawful object. Option 1 is an
Legal Reasoning Practice Sheet 75
CLAT POINT

example of a valid contract with a condition precedent, option 3 is an example of a gift, and option 4 is an example of
a sale at a price that is not prohibited by law.

20. Answer: B) An agreement between two artists not to exhibit their work in a particular gallery for a period of one
year.
Option B is an example of an agreement in restraint of trade, which is void under Section 27 of the Indian Contract
Act. Options A, C, and D are valid contracts as they do not fall under any of the categories of void agreements
mentioned in the passage.

21. Ans – a
Option a is correct because in English law, the contracts made by a person with unsound mind are voidable at his
option only if he was not completely incapable of understanding the contract at the time it was signed. Option b is
incorrect because even if Mr Tull understood the contract at the time of signing, his incapacity could still make the
contract voidable at his option. Option c is incorrect because the contract would not be void if Mr Johnson did not
know about Mr Tull's incapacity, and it would only be voidable at Mr Tull's option. Option d is incorrect because Mr
Tull's capacity at the time of signing is what determines the voidability of the contract, not Mr Johnson's actions.

22. Ans - d
Explanation: The agreement of a person of unsound mind is void in Indian law, and alcoholic psychosis can be
considered a form of unsound mind. Therefore, the sale deed is void ab initio, and it can be set aside. Option a is
incorrect because a sale agreement with a person of unsound mind is not a binding contract in Indian law. Option b is
incorrect because it does not matter whether the other party knew about the transferor's condition. Option c is
incorrect because the transferor's capacity to understand the terms of the sale is irrelevant since the sale is void ab
initio. Option e is incorrect because it assumes that the transferor has the option to affirm the sale, which is not
applicable in this case since the sale is void.

23. Ans - b
Option b is correct because Mr. Jacobs confirmed the contract during a period of lucidity, and a person who is usually
of unsound mind may make a contract when they are of sound mind. Option a is wrong because the fact that Mr.
Jacobs has a history of schizophrenia does not make the contract void. Option c is wrong because the contract is not
voidable at the option of Mr. Jacobs since he confirmed it during a period of lucidity. Option d is wrong because Mr.
Alex's knowledge of Mr. Jacobs' condition does not make the contract voidable.

24. Ans – b.
Under English law, a person of unsound mind is competent to contract, but the contract is voidable at his option if he
can prove that he was incapable of understanding the contract and the other party knew it. However, in this scenario,
John was of sound mind when he entered into the contract with Jane, and his later relapse cannot void the contract
as long as he was capable of understanding the contract at the time of execution. Therefore, option b is the correct
answer. Option a is incorrect because John's mental disorder does not automatically make the contract voidable.
Option c is also incorrect because there is no evidence that Jane knew about John's mental disorder at the time of the
contract. Option d is incorrect as option b is the correct answer.

25. Ans – b) The contract is void ab initio as it is illegal and Mr. Raghav's unsound mind cannot validate it.
The Indian Contract Act, 1872 states that an agreement made by a person of unsound mind is void. Moreover, as the
contract was formed to smuggle drugs, it is an illegal contract, and the court will not enforce such contracts. Therefore,
Mr. Raghav cannot enforce the contract or claim damages from Mr. Arun.

26. Explanation:

Legal Reasoning Practice Sheet 76


CLAT POINT

As per the principle of joint and several liability, when several persons commit a tortious act, they become joint
tortfeasors, and are jointly and severally liable for the same tort. This means that the aggrieved party has the option
to sue any one of them, some of them, or all of them in a single action, and can recover the full amount of
compensation from any of them, or all of them jointly. In this case, Mr. Z can sue both Mr. X and Mr. Y jointly in one
action, and can recover the full amount of compensation from either of them, or both of them jointly, if he wins the
case. Thus, option C is the correct option. Option A is incorrect because Mr. Z can sue both of them jointly in one
action, and not just one of them. Option B is also incorrect because Mr. Z cannot sue the joint tortfeasors separately
for the same tortious act. Option D is incorrect because Mr. Z can sue both the joint tortfeasors jointly in one action,
and does not have to file separate actions against each of them.

27. Answer:
Option d is correct. In cases where joint liability exists, all defendants are inherently liable up to the full amount of the
judgment awarded to the plaintiff, which rules out options a and b. A successful plaintiff can choose to collect the
entire amount from one defendant or from all defendants collectively. Therefore, both A and B are liable to pay
damages, but it is up to C to decide the proportion of damages to be paid by each defendant.

28. Ans- a
Even though the actual wrongdoer is the agent, if the plaintiff so elects, he may sue the principal for the whole of the
damage. As against the aggrieved party, the principal cannot take the defence that the actual wrongdoer was the
agent, although after making good the loss, the principal may hold the agent responsible to the extent of his (agent's)
fault. In this case, after the final payment, A can make B liable for his part of the injury.

29. Explanation:
The question now includes additional details that make it more challenging to determine liability. Jogindar was driving
recklessly, but Manindar's improper parking also contributed to the accident. Surindar's presence and failure to take
action to prevent the accident also adds complexity to the situation. Hence, option C is the most appropriate answer
as both Jogindar and Manindar are liable for Surindar's injuries as their actions contributed to the accident.

30. Answer: c
Explanation: Joint and severable liability is applicable when multiple parties act together and cause harm to someone
else. In this case, X and Y jointly built a faulty bridge that caused harm to Z. While both parties are jointly liable for the
harm caused, they are also severally liable, meaning that they can be held responsible for the full amount of the
damages if the other party is unable to pay. This principle is not applicable in the other scenarios listed. In a and b, the
parties acted independently of each other, and in d, the parties committed a crime together but did not cause harm
to anyone else.

31. Ans - b.
The correct answer is option b, which states that the promise made by Mr C is not covered under section 25(3) of the
Indian Contract Act, 1872. This is because while Mr C did make a promise in writing to pay the time-barred debt, he is
not the person who was originally liable for the debt. Section 25(3) only applies to a promise made by the person who
was originally liable for the debt or by their authorized agent. Option a is incorrect because it suggests that the promise
made by Mr C is covered under section 25(3), which is not the case due to the reasons stated above. Option c is
incorrect because it states that the Indian Contract Act, 1872 does not take cognisance of time-barred debts, which is
also not true. Section 25(3) of the Act specifically deals with promises to pay such debts. Option d is incorrect because
it does not provide a correct answer to the question.

32. Ans – c
Since it is expressly provided in the questions that Mr Tucker had promised, writing to execute a transfer of a car as a
payment for the past services which were rendered by his butler Mr Chris, this s entirely covered by section 25(2) of

Legal Reasoning Practice Sheet 77


CLAT POINT

the Indian Contract Act, 1872 and is valid. Option b is wrong because it is covered by section 25(2) due to it being a
payment for past services. Option c is wrong because the promise is a valid one not a voidable one.

33. Ans. (d)


The correct answer is d, as Mr Z's promise to pay a portion of the time-barred rent in writing is valid according to
section 25(3) of the Indian Contract Act, 1872. Option a, b, and c are incorrect as they do not consider the principle in
the passage, which is that a written promise to pay a time-barred debt is valid. The details added in the question, such
as the acknowledgement of obligation and the portion of the debt, are not relevant to the validity of the promise
under the Indian Contract Act, 1872.

34. Ans. (d)


The correct answer is option (d). The subsequent written agreement to extend the time for repayment creates a new
contract that replaces the original promissory note. Mr. X's failure to repay the amount on the extended due date
amounts to a breach of this new contract, and Mr. Y can enforce it. Option (a) is wrong because the subsequent
agreement does not make the original promissory note void. Option (b) is also wrong because the subsequent
agreement is not without consideration. Option (c) is wrong because the subsequent agreement is not a promise to
pay a time-barred debt, but a new contract that extends the time for repayment.

35. Explanation: All of the statements in options a, b, and c are true according to the passage. Therefore, option d, "All
of the above are true," is the correct answer.

36. Ans – d
Since it is provided in section 5 that a “marriage may be solemnized between any two Hindus” and Mr X converted to
Hinduism after the date of wedding, the marriage cannot be valid under section 5. Hence both b and c are correct.
Thus, d is the right answer. Option a is wrong because he had converted after the wedding ceremony so at the time
of marriage, he was still a Christian.

37. Answer: a) Yes, as neither party has a living spouse, both parties have given valid consent, and they have fulfilled
the age requirement for marriage under the Hindu Marriage Act.
Explanation:
Option (a) is correct as Rahul and Pooja fulfill the conditions for a legal marriage under the Hindu Marriage Act, and
the fact that their gotra is the same does not render their marriage void. Option (b) is incorrect because, under the
Hindu Marriage Act, parties cannot marry within the degrees of prohibited relationship, which includes sapindas or
persons who are related by blood or marriage up to the third generation. But here Pooja does not fall into that category
for marrying Rahul. Option (c) is incorrect because, under the Special Marriage Act, a marriage officer can refuse to
solemnize a marriage if he has reason to believe that any of the conditions mentioned in the act have not been fulfilled.
One such condition is that there should not be any pre-existing marriage of either party.

38. Answer: b) No, because Rohan is incapable of giving valid consent to the marriage due to his mental disorder,
which is a condition mentioned in the act.
Explanation: While the Special Marriage Act allows individuals of different religions to marry, it also requires both
parties to give valid consent to the marriage. In this case, Rohan's mental disorder makes him incapable of giving such
consent, which is a condition mentioned in the act. Hence, option (b) is the correct choice. Option (a) is incorrect
because fulfilling the age and other requirements is not enough if one party is incapable of giving valid consent. Option
(c) is also incorrect because while Art. 21 guarantees the right to marry, it is subject to reasonable restrictions imposed
by law. In this case, the restriction on the marriage is reasonable because it is based on Rohan's inability to give valid
consent.

Legal Reasoning Practice Sheet 78


CLAT POINT

39. Answer: a) No, because Ravi is still legally married, and the Hindu Marriage Act prohibits the solemnization of a
second marriage unless the first marriage has been legally dissolved.
Explanation:
Option (a) is correct as per the Hindu Marriage Act, a person cannot solemnize a second marriage if his/her first
marriage is still legally valid. Option (b) is incorrect because even though Meera and Ravi fulfill the age requirement
for marriage, Ravi's previous marriage still stands in the way of their legal marriage. Option (c) is incorrect as caste is
not a legal ground for refusal of marriage registration, and it is not considered a prohibited relationship under the
Hindu Marriage Act. Option (d) is incorrect as only option (a) is correct.

40. The correct option is (a) X and Y can marry under the Hindu Marriage Act, 1955, as the Act permits marriages
between Hindus of different castes, as long as the other conditions mentioned in Section 5 are met. The option (b) is
incorrect as the consent of parents is not required for a valid Hindu marriage under the Hindu Marriage Act, although
it is customary in some communities. Option (c) is incorrect as Article 16 of the Universal Declaration of Human Rights
does not override the specific provisions of Indian law. Option (d) is incorrect as the passage does not mention anything
about violating the fundamental group unit of society as stated in Article 16.

41. Ans. (d)


The correct answer is d. Mrs. Y can file for divorce on the grounds of cruelty under the Hindu Marriage Act, 1955
provided she can prove that the acts of cruelty have caused her mental or physical trauma. Option a is incorrect
because the act of cruelty does not need to be continuous and repetitive for a divorce to be granted on the grounds
of cruelty. Option b is incorrect because while cruelty is a punishable offense under the Indian Penal Code, Mrs. Y is
seeking a divorce under the Hindu Marriage Act. Option c is incorrect because the Hindu Marriage Act recognizes both
physical and mental cruelty as grounds for divorce.

42. Ans. (b)


Explanation for the options:
(a) This option is incorrect because the definition of cruelty under the Hindu Marriage Act does not have any provision
for considering the mental state of the offender at the time of the incident.
(b) This option is correct because Mrs. Z's actions, which caused the death of their children, fall under the definition
of cruelty under the Hindu Marriage Act.
(c) This option is incorrect because pregnancy is not a defense for committing an act of cruelty, and the mental state
of the offender is not considered under the Hindu Marriage Act.
(d) This option is incorrect because the incident was caused by Mrs. Z's actions and not Mr. X's, and the Hindu Marriage
Act does not require proof of his un-involvement in the incident for granting a divorce on the grounds of cruelty.

43. Ans. (c)


The correct answer is c. The petitioner must establish that the respondent has treated the petitioner with cruelty to
obtain a divorce on the grounds of cruelty. The relevant legal principle in this situation is the requirement for the
petitioner to establish that the respondent has treated the petitioner with cruelty. Option a is incorrect because the
act of cruelty does not need to be continuous and repetitive for a divorce to be granted on the grounds of cruelty.
Option b is incorrect because mental cruelty is a recognized ground for divorce under the Hindu Marriage Act, 1955.
Option d is incorrect because the Hindu Marriage Act, 1955 recognizes both physical and mental cruelty as grounds
for divorce.

44. Ans. (d)


The answer is option d. As per the passage, cruelty under the Indian Penal Code includes any willful conduct that can
drive a woman to suicide or cause grave injury or danger to life, limb, or health

Legal Reasoning Practice Sheet 79


CLAT POINT

(whether mental or physical). Harassment of a woman, where such harassment is with a view to coercing her or any
person related to her to meet any unlawful demand for any property or valuable security, would also constitute
cruelty. Therefore, the scenario presented in option d meets the requirements to be considered cruelty under the laws
mentioned in the passage. Option a presents a scenario where the husband is verbally abusive towards his wife, which
is undoubtedly a form of emotional cruelty. However, the passage does not specify whether emotional abuse is
considered cruelty under the laws mentioned. Option b presents a scenario where the wife refuses to have sexual
relations with her husband, which may be a cause for concern in a marriage. However, the passage does not indicate
that it constitutes cruelty under the laws mentioned. Option c presents a scenario where a couple frequently argues
over trivial matters. As mentioned in the passage, quarrels over trivial matters may not be considered cruelty.
Therefore, this option does not meet the requirements to be considered cruelty under the laws mentioned. In
conclusion, while the question of
what may or may not constitute cruelty under personal laws and the Indian Penal Code may not have a straightforward
answer, it is clear that harassment of a woman with a view to coercing her or any person related to her to meet any
unlawful demand for any property or valuable security, leading to her suicide or causing grave injury or danger to life,
limb, or health, is considered cruelty under the laws mentioned.

45. Ans. (b)


The correct answer is option b, which accurately reflects an act of cruelty as defined in section 498A of the Indian
Penal Code. The section states that cruelty includes any wilful conduct that can drive a woman to suicide or cause
grave injury or danger to life, limb, or health (whether mental or physical). Harassment of a woman, where such
harassment is with a view to coercing her or any person related to her to meet any unlawful demand for any property
or valuable security, would also constitute cruelty. Option a is intentionally wrong, as forgetting a birthday and not
buying a gift is not an act of cruelty under section 498A. Option c is also intentionally wrong, as calling someone a
derogatory name during an argument may be hurtful, but it does not necessarily constitute an act of cruelty under
section 498A. Option d is also intentionally wrong, as cooking different dishes does not constitute an act of cruelty
under section 498A. In conclusion, this question has been designed to test the examinee's understanding of the
definition of cruelty under section 498A of the Indian Penal Code. The correct answer, option b, accurately reflects an
act of cruelty as defined in the section. The intentionally wrong options have been included to make the question more
challenging and to differentiate between acts that may be hurtful but do not necessarily meet the legal definition of
cruelty.

46. Ans. (b)


The correct answer is b. The passage explains that if a spouse abandons the other in a state of temporary passion, for
example, anger or disgust without intending permanently to cease cohabitation, it will not amount to desertion.
Desertion requires the factum of desertion and the intention to bring cohabitation permanently to an end (animus
deserendi). Option a is wrong, as it suggests that Raj's behavior would be considered an act of desertion, but it does
not consider the possibility that he may have left in a state of temporary passion. Option c is also wrong, as it suggests
that Raj's behavior would not be considered an act of desertion because Reena made several attempts to contact him
and reconcile, but it does not consider the intention behind Raj's absence. Option d is wrong, as it suggests that Raj's
behavior would be considered an act of desertion because it caused Reena mental and emotional anguish, but it does
not consider the intention behind Raj's absence.

47. Ans. (c)


The correct answer is c. The passage states that in cases of desertion, the deserted spouse must prove the absence of
conduct giving reasonable cause to the spouse leaving the matrimonial home to form the necessary intention
aforesaid. In this scenario, Mr. Y's mental and physical torture would be considered conduct giving reasonable cause
to Mrs. X to form the necessary intention of deserting him. Additionally, the passage notes that desertion requires the
intention to bring cohabitation permanently to an end, and in this case, Mrs. X's absence was not permanent and she

Legal Reasoning Practice Sheet 80


CLAT POINT

expressed a willingness to resume cohabitation under certain conditions. Option a is wrong, as it suggests that Mrs.
X's intention to end cohabitation was the sole factor in determining whether she is liable for desertion, disregarding
the reason for leaving. Option b is also wrong, as it suggests that Mrs. X is automatically liable for desertion solely
because she left her husband, disregarding the reason for leaving. Option d is wrong, as it suggests that Mrs. X's liability
for desertion depends on whether she made a bona fide offer to resume cohabitation before the statutory period for
desertion had ended, which is not the only factor to consider in determining desertion. In conclusion, this question
has been designed to test the examinee's understanding of the legal principles related to desertion and the application
of those principles to a hypothetical scenario. The correct answer, option c, reflects the fact that Mrs. X is not liable
for desertion because Mr. Y's conduct was giving reasonable cause to her to form the necessary intention of deserting
him, and her absence was not permanent. The intentionally wrong options have been included to make the question
more challenging and to differentiate between acts that may be hurtful but do not necessarily meet the legal definition
of desertion.

48. Answer: (b) B's claim for desertion will not be successful as A left due to temporary passion and not with the
intention to end cohabitation permanently.
Explanation: As per the legal principles mentioned in the passage, desertion occurs when one spouse leaves the other
without consent and with the intention to permanently end cohabitation. In this scenario, A left in a state of temporary
passion, which means that A did not leave with the intention to permanently end cohabitation. Therefore, B's claim
for desertion will not be successful. Option A is wrong because A did leave without B's consent, but the intention to
end cohabitation permanently is not proven. Option C is incorrect because A did come back within the statutory period
and offered to resume matrimonial life. Option D is incorrect because A does not have to offer an explanation for
leaving, and contacting the deserted spouse during the period of separation is not a requirement for desertion.

49. Ans. (a)


The correct answer is a. The passage states that in proceedings for divorce, the plaintiff must prove the offense of
desertion like any other matrimonial offense, beyond all reasonable doubt. While corroboration is not required as an
absolute rule of law, the courts insist upon corroborative evidence, unless its absence is accounted for to the
satisfaction of the court. Option b is wrong, as it suggests that corroboration is a mandatory requirement in all cases
of desertion. Option c is wrong, as it suggests that corroboration is only required in cases where the deserting spouse
denies the act of desertion in court. Option d is wrong, as it suggests that corroboration is only required if the deserting
spouse has been charged with other matrimonial offenses.
50. Ans – a
Explanation:
Option a is correct, as the wife in this case had a reasonable cause to leave her husband, who was subjecting her to
persistent physical and mental cruelty, including harassment for dowry, which had led to a breakdown in their marital
relationship and posed a threat to her safety and well-being. Therefore, her act of leaving cannot be classified as
desertion. Option b is incorrect, as the wife's decision to leave her husband to go overseas and marry someone else
demonstrates a clear intention to permanently end her matrimonial ties with her husband, without any reasonable
cause or justification for such an action. This would meet the requirements of animus deserendi and absence of
consent, making it an act of desertion. Option c is also incorrect, as the wife's decision to live separately from her
husband, without attempting to resolve their issues or finding a reasonable cause for such a separation, exhibits a
clear intention to end their marriage, meeting the requirements of animus deserendi and absence of consent. Option
d is incorrect, as each of the options can potentially meet the requirements of desertion, depending on the facts and
circumstances of the case, as established by the aforementioned case law. Therefore, none of the options can be
excluded from the ambit of desertion without a thorough analysis of the facts and circumstances of each case.

51. Answer: a

Legal Reasoning Practice Sheet 81


CLAT POINT

Explanation: Since Mr Mahesh's earlier suit was dismissed on merits for failure to produce valid succession documents,
the doctrine of res judicata applies, and the subsequent suit is barred. The parties and the cause of action are the same
in both suits, and the decision given by one court should be considered final, subject to appeal or review. Option b is
incorrect because the lack of valid succession documents was the reason for the dismissal of the earlier suit and not
the only factor, and new evidence may not be considered in a subsequent suit. Option c is incorrect because the
principle of res subjudice applies only when the same cause of action is pending before another court, which is not
the case here.

52. Answer (a)


Explanation: The correct answer is a, the new suit is not barred as the parties involved are not identical in both cases.
The doctrine of Res Judicata applies when a party tries to file the same matter against another party in the same court
or in another competent court, and the decision of one court should be considered final, subject to any appeal, revision
or review. However, Res Judicata only applies in cases where the parties are identical. In this case, X is suing both Y
and Z for a declaration of his title to the land, which is not identical to the previous case where X was suing Y for rent.
As a result, Res Judicata is not applicable here, and the new suit is not barred. Option b is incorrect because the relief
sought is different, and option c is incorrect because the parties are not identical in both cases. Option d is incorrect
because the new suit is not barred as the parties involved are not identical in both cases

53. Ans. ( c )
The correct answer is c. The provided passage explains that if a writ petition filed by a party under Article 226 of the
Constitution of India is dismissed without passing a speaking order, then such dismissal cannot be treated as creating
a bar of res judicata. Therefore, Mr. A can file a subsequent petition under Article 32 of the Constitution of India in the
Supreme Court, but only if he can demonstrate that the dismissal of the writ petition by the High Court was not on the
merits. Option a is wrong, as it suggests that the dismissal of the writ petition by the High Court for lack of jurisdiction
does not create a bar of res judicata, without considering whether the dismissal was on the merits or not. Option b is
also wrong, as it suggests that the general doctrine of res judicata applies to all legal matters, regardless of the court's
jurisdiction, which is not the case. Option d is wrong, as it suggests that Mr. A cannot file a subsequent petition under
Article 32 of the Constitution of India because his petition under Article 226 of the Constitution of India was already
dismissed, without considering whether the dismissal was on the merits or not. In conclusion, the correct answer,
option c, reflects that Mr. A can file a subsequent petition under Article 32 of the Constitution of India if he can
demonstrate that the dismissal of the writ petition by the High Court was not on the merits.

54. Ans. (b)


The correct answer is b. The provided passage explains that the general doctrine of res judicata is based on public
policy and applies to all legal matters, regardless of the court's jurisdiction. Therefore, Mr. X cannot file a subsequent
petition under Article 32 of the Constitution of India on the same facts and for obtaining the same or similar orders or
writs, as his petition under Article 226 of the Constitution of India was already dismissed on the ground of res judicata.
Option a is wrong, as it suggests that the dismissal of the writ petition by the High Court on the ground of res judicata
does not create a bar to a subsequent petition under Article 32 of the Constitution of India, without considering the
general doctrine of res judicata. Option c is also wrong, as it suggests that Mr. X can file a subsequent petition under
Article 32 of the Constitution of India if he can demonstrate that the dismissal of the writ petition by the High Court
was not on the merits, without considering the general doctrine of res judicata. Option d is wrong, as it suggests that
the doctrine of res judicata does not apply to writ jurisdictions, which is not the case. In conclusion, the correct answer,
option b, reflects that Mr. X cannot file a subsequent petition under Article 32 of the Constitution of India on the same
facts and for obtaining the same or similar orders or writs, as his petition under Article 226 of the Constitution of India
was already dismissed on the ground of res judicata.

55. Answer: (c)


Legal Reasoning Practice Sheet 82
CLAT POINT

Explanation: The court in the criminal case only decided Mr. Smith's criminal liability and acquitted him of all charges.
However, this does not mean that he is not civilly liable for the injuries he caused to Ms. Jones. Therefore, Ms. Jones
can file a civil suit against Mr. Smith for damages, and if she is able to prove her case in the civil court, she can be
successful in her suit. Option a is incorrect because the standard of proof is not the only factor that determines the
success of a civil suit. Option b is incorrect because the acquittal in the criminal case does not necessarily mean that
Mr. Smith is not liable in the civil case. Option d is incorrect because one of the options is correct.

56. Ans. (b)


The correct answer is b. The provided passage explains that where art and obscenity are mixed, what must be seen is
whether the artistic, literary or social merit of the work in question outweighs its obscene content. The element of art
must overshadow the obscenity or make it so trivial and inconsequential that it can be ignored. Therefore, Mr. X
cannot be charged under section 292 of the Indian Penal Code for obscenity, as the element of art in the painting may
overshadow any obscenity or make it so trivial and inconsequential that it can be ignored. Option a is wrong, as it
suggests that Mr. X can be charged under section 292 of the Indian Penal Code for obscenity just because the painting
depicts nudity, without considering the element of art in the painting. Option c is also wrong, as it suggests that Mr. X
can be charged under section 292 of the Indian Penal Code for obscenity only if the painting has the tendency to
deprave and corrupt those who are likely to be exposed to it, without considering the element of art in the painting.
Option d is wrong, as it suggests that the government can only block websites on specific grounds, and the display of
a painting in an art exhibition does not create a public order problem, which is not relevant to the scenario.

57. Ans. (a)


The correct answer is a. The provided passage explains that section 292 of the Indian Penal Code deals with obscenity,
which is defined as that which is "lascivious or appeals to prurient interest" or which has the tendency to "deprave"
and "corrupt" those who are likely to be exposed to it. In this case, Mr. S was found to be listing video-clips of women
engaging in explicit sexual activities for sale and inviting people, including children, to purchase and view them. Since
these video-clips have the tendency to "deprave" and "corrupt" those who are likely to be exposed to them, Mr. S can
be charged under section 292 of the Indian Penal Code. Option b is wrong, as it suggests that the sale of video-clips
depicting sexual activities is protected under the fundamental right to freedom of speech and expression, without
considering the fact that obscenity is not protected under this fundamental right. Option c is also wrong, as it suggests
that the video-clips listed by Mr. S may serve as a means of sex education, without considering the fact that the sale
of video-clips depicting sexual activities to children cannot be considered as a means of sex education. Option d is
wrong, as it suggests that both options b and c are wrong, without providing any valid reasoning to support this claim.

58. Ans. (b)


The correct answer is b. The provided passage explains that section 292 of the IPC deals with obscenity, which is
defined as that which is lascivious or appeals to prurient interest or which has the tendency to deprave and corrupt
those who are likely to be exposed to it. In this case, the accused was found selling a packet of playing cards that
portrayed luridly obscene naked pictures of men and women in pornographic sexual postures on the reverse. Since
this amounts to obscenity, the accused can be charged under section 292 of the IPC. Option a is wrong, as it suggests
that section 69A of the IT Act can be used to punish the accused, without considering the fact that there is no use of
websites by the accused in this scenario. Option c is also wrong, as it suggests that both sections 69A of the IT Act and
292 of the IPC can be used to punish the accused, without considering the fact that the provisions of the IT Act are not
applicable in this scenario. Option d is wrong, as it suggests that the accused's actions do not come under the purview
of either section 69A of the IT Act or section 292 of the IPC, without considering the fact that the accused's actions
amount to obscenity under section 292 of the IPC.

59. Ans. (c )

Legal Reasoning Practice Sheet 83


CLAT POINT

The correct answer is c. The provided passage explains that the accused was found selling a packet of playing cards
that portrayed luridly obscene naked pictures of men and women in pornographic sexual postures on the reverse.
Upon conducting a raid of his shop, two more similar packets were recovered from him. Further investigation revealed
that the accused had a website on his computer, which he used to sell the cards online to various customers. It was
also found that over 500 orders of such cards had been already delivered by him to various customers. Since the
accused had made use of websites to commit the offence of obscenity under section 292 of the IPC, and that his act
had created a public order problem, both section 69A of the IT Act and section 292 of the IPC can be used to punish
the accused. Option a is partially correct, as it suggests that section 69A of the IT Act can be used to punish the accused
for using a website to sell obscene content. However, it fails to consider the fact that the accused can also be charged
under section 292 of the IPC for committing the offence of obscenity. Option b is also partially correct, as it suggests
that section 292 of the IPC can be used to punish the accused for selling obscene content. However, it fails to consider
the fact that the accused can also be charged under section 69A of the IT Act for using a website to commit the offence.
Option d is wrong, as it suggests that the accused's actions do not come under the purview of either section 69A of
the IT Act or section 292 of the IPC, without considering the fact that the accused's actions involved the use of a
website to sell obscene content and thus, both sections of the law are applicable in this scenario.

60. Ans. (a)


The correct answer is a. The performers can be charged under section 292 of the Indian Penal Code. While the right
to freedom of speech and expression is guaranteed under the Indian Constitution, this right is not absolute and is
subject to reasonable restrictions. One such restriction is the restriction on obscenity, as outlined in section 292 of the
Indian Penal Code. The definition of obscenity includes material that is lascivious or appeals to prurient interest, or
material that has the tendency to deprave and corrupt those who are likely to be exposed to it. The street performers'
show involves nudity and simulated sexual acts, which can be considered lascivious and appeal to prurient interest.
Therefore, the performers can be charged under section 292 of the Indian Penal Code. Option b is incorrect, as it
suggests that the performers cannot be charged under section 292 of the Indian Penal Code, without considering the
specific details of the case. Option c is also incorrect, as it suggests that the performers can be charged under section
292 of the Indian Penal Code, but only if they failed to consider the impact of their performance on public decency.
While public decency is a consideration in determining whether something is obscene, it is not a requirement for
charging someone under section 292 of the Indian Penal Code. Option d is incorrect, as it suggests that the answer
depends on the specific details of the case, without considering the general legal principles that apply in such
situations.

61. Ans C
The Indian Penal Code, Section 124A punishes any speech, sign, or representation that brings or attempts to bring
hatred or contempt towards the government established by law in India. In order to be charged under this section,
the act must be aimed against the State. In this case, the charge framed against Mr. Z did not contain any allegations
of him acting against the Government of India or any State government. Hence, option b is the correct answer. Option
a is incorrect since there is no evidence that Mr. Z acted against the Government of India or any State government.
Option c is incorrect since the facts provided in the question are sufficient to make a decision. Finally, option d is
incorrect since there is a correct answer to the question. In conclusion, Mr. Z cannot be charged under section 124A
of the IPC since the charge framed against him did not contain any allegations of him acting against the Government
of India or any State government.

62. Ans A
The correct answer is a. Section 153A of the IPC prohibits and punishes the promotion of enmity between different
groups on grounds of religion, race, place of birth, residence, language, etc. In this scenario, Ms X was charged under
Section 153A for promoting enmity between different religious groups on grounds of religion through a video she
posted on her social media page. As the video allegedly had inflammatory content that was derogatory towards a
particular religious group, Ms X can be held guilty under Section 153A of the IPC.
Legal Reasoning Practice Sheet 84
CLAT POINT

Option b is incorrect because it suggests that freedom of expression allows an individual to express their views and
opinions, even if they may be derogatory towards a particular religious group. However, freedom of expression is not
absolute and is subject to reasonable restrictions, such as incitement to violence or hatred.
Option c is incorrect because it suggests that the charge under Section 153A of the IPC can only be upheld if Ms X had
the intention of promoting enmity between different religious groups on grounds of religion. However, the charge can
be upheld if the video posted by Ms X had content that promoted enmity between different religious groups on
grounds of religion, regardless of her intention.
Option d is also incorrect because it suggests that the answer depends on whether the video posted by Ms X had a
real likelihood of causing public disorder or violence. However, as per the legal principles related to Section 153A of
the IPC, the charge can be upheld if the video had content that promoted enmity between different religious groups
on grounds of religion, regardless of whether there was a likelihood of causing public disorder or violence

63. Ans a
The correct answer is a. Mr X can be prosecuted under both Sections 124A and 153A of the Indian Penal Code as he
has committed offences under both sections. Section 124A of the Indian Penal Code prohibits and punishes sedition,
which involves incitement of violence or rebellion against the Government of India. Mr X's messages endorsing the
idea of waging a war against the Government of India and Indian Army to separate his native state from the territory
of India fall under this section. Section 153A of the Indian Penal Code prohibits and punishes promoting enmity
between different groups on grounds of religion, race, place of birth, residence, language, etc. Mr X's messages that
incited violence against people of different religions fall under this section. Option b is incorrect because it suggests
that Mr X cannot be prosecuted under both sections as the offences committed by him fall under only one section.
However, Mr X has committed offences under both Sections 124A and 153A of the Indian Penal Code Option c is
incorrect because it suggests that Mr X can be prosecuted under Section 124A of the Indian Penal Code, but not under
Section 153A. However, Mr X has committed offences under both Sections 124A and 153A of the Indian Penal Code
Option d is also incorrect because it suggests that Mr X can be prosecuted under Section 153A of the Indian Penal
Code, but not under Section 124A. However, Mr X has committed offences under both Sections 124A and 153A of the
Indian Penal Code.

64. Ans a
The correct answer is a. This is not sedition as Mr. X is exercising his freedom of speech and expression and is only
requesting his supporters to vote against the current government in the upcoming elections. Section 124A of the
Indian Penal Code prohibits and punishes sedition, which involves incitement of violence or rebellion against the
government. Mr. X's speech does not fall under this section as he is not inciting his supporters to use force or violence
to overthrow the government. Option b is incorrect because it suggests that Mr. X's speech is sedition as he is making
derogatory remarks against the government and inciting his supporters to overthrow the government by use of force
or violence. However, Mr. X's speech does not incite his supporters to use force or violence to overthrow the
government. Option c is incorrect because it suggests that the charge of sedition can only be upheld if Mr. X had the
intention of inciting violence or rebellion against the government and if his words had a real likelihood of causing
public disorder or violence. However, Mr. X's speech does not have the intention of inciting violence or rebellion
against the government. Option d is also incorrect because it suggests that the answer depends on whether Mr. X had
the intention of inciting violence or rebellion against the government and if his words had a real likelihood of causing
public disorder or violence. However, as per the legal principles related to sedition, Mr. X's speech does not incite
violence or rebellion against the government.

65. Ans D
The correct answer is d. None of the above. While Mr. Z's message was false and baseless, none of the legal provisions
mentioned in the passage can be used to prosecute him. Section 295A of the Indian Penal Code pertains to outrage of
religious feelings, which is not relevant to this scenario. Section 153A of the Indian Penal Code pertains to promoting

Legal Reasoning Practice Sheet 85


CLAT POINT

enmity between different groups, which is also not relevant to this scenario. Section 79 of the Information Technology
Act pertains to the liability of intermediaries such as search engines, but Mr. Z is not an intermediary in this case.
Option a is incorrect because it suggests that Section 295A of the Indian Penal Code can be used to prosecute Mr. Z
for deliberately and maliciously outraging religious feelings of any class by insulting its religion or religious beliefs,
which is not relevant to this scenario. Option b is incorrect because it suggests that Section 153A of the Indian Penal
Code can be used to prosecute Mr. Z for promoting enmity between different groups on grounds of religion, race,
place of birth, residence, language, etc., which is also not relevant to this scenario. Option c is incorrect because it
suggests that Section 79 of the Information Technology Act can be used to prosecute Mr. Z for failing to remove
objectionable content pursuant to takedown notices by law.

66. Ans B
The correct answer is b. Section 375(e) of the Indian Penal Code for having sexual intercourse with Ms. Y with her
consent, when at the time of giving such consent, by reason of unsoundness of mind or intoxication, she was unable
to understand the nature and consequences of the act to which she gave consent. In this scenario, Mr. X spiked Ms.
Y's drink with a sedative, which caused her to become intoxicated and unable to understand the nature and
consequences of the sexual act to which she gave consent. Option a is incorrect because it suggests that Mr. X had
sexual intercourse with Ms. Y with her consent, when he knew that he is not her husband and that her consent was
given because she believed that he is another man to whom she is or believes herself to be lawfully married. However,
in this scenario, Mr. X did not deceive Ms. Y about his identity or marital status. Option c is incorrect because it suggests
that Mr. X had sexual intercourse with Ms. Y, who is under 16 years of age. However, in this scenario, there is no
mention of Ms. Y's age. Option d is incorrect because it suggests that none of the legal provisions mentioned in the
passage can be used to prosecute Mr. X. However, as explained above, Section 375(e) of the Indian Penal Code can be
used to prosecute Mr. X for having sexual intercourse with Ms. Y with her consent, when she was unable to understand
the nature and consequences of the act to which she gave consent due to intoxication caused by Mr. X spiking her
drink.

67. Ans D
The correct answer is d. Section 375(e) of the Indian Penal Code for having sexual intercourse with a woman with her
consent, when at the time of giving such consent, by reason of unsoundness of mind or intoxication, she was unable
to understand the nature and consequences of the act to which she gave consent. In this scenario, Mr. X informed Ms.
Y that the sexual act was a method for improving her violin skills, which she agreed to. However, Ms. Y was not
informed of the true nature of the act, which makes her consent invalid under Section 375(e) of the Indian Penal Code.
Option a is incorrect because it suggests that Mr. X had sexual intercourse with Ms. Y against her will, which is not
mentioned in the scenario.
Option b is incorrect because it suggests that Mr. X had sexual intercourse with Ms. Y without her consent, which is
also not mentioned in the scenario.
Option c is incorrect because it suggests that Mr. X had sexual intercourse with Ms. Y with her consent, which was
obtained by putting her or any person in whom she is interested in fear of death or of hurt, which is not relevant to
this scenario.

68. Ans B
The correct answer is b. No, Mr. X is not guilty of committing the offence of rape under Section 375 of the Indian Penal
Code, as the victim was his wife and above the age of 18. Under Section 375, sexual intercourse by a man with his own
wife who is above the age of 18 is not considered sexual assault or rape.
Option a is incorrect because it suggests that Mr. X is guilty of committing the offence of rape, even if the victim was
his wife. However, as explained above, Section 375 provides an exception for sexual intercourse between a husband
and wife, as long as the wife is above the age of 18.

Legal Reasoning Practice Sheet 86


CLAT POINT

Option c is incorrect because it suggests that Mr. X is guilty of committing rape because the sexual intercourse was
forceful and against the wife's will. However, the scenario does not mention whether the wife consented to the sexual
act or not, and under Section 375, forceful sexual intercourse is not sufficient to constitute rape if the victim is the
man's wife and above the age of 18.
Option d is incorrect because it suggests that none of the options are correct, but as explained above, option b is the
correct answer.

69. Ans C
The correct answer is c. Section 376(2) of the Indian Penal Code for having sexual intercourse with a woman when she
is in a state of intoxication or under the influence of drugs and unable to give her consent. In this scenario, Mr. B spiked
Ms. A's drink with a sedative, which caused her to become intoxicated and unable to give her consent for the sexual
act that followed.
Option a is incorrect because it suggests that Mr. B had sexual intercourse with Ms. A with her consent, when he knew
that he is not her husband and that her consent was given because she believed that he is another man to whom she
is or believes herself to be lawfully married. However, in this scenario, Mr. B did not deceive Ms. A about his identity
or marital status.
Option b is incorrect because it suggests that Mr. B had sexual intercourse with Ms. A with her consent, when at the
time of giving such consent, by reason of unsoundness of mind or intoxication, she was unable to understand the
nature and consequences of the act to which she gave consent. However, in this scenario, Ms. A did not give her
consent for the sexual act, as she was unable to give her consent due to the sedative that Mr. B had spiked her drink
with.
Option d is incorrect because it suggests that none of the legal provisions mentioned in the passage can be used to
prosecute Mr. B. However, as explained above, Section 376(2) of the Indian Penal Code can be used to prosecute Mr.
B for having sexual intercourse with a woman when she is in a state of intoxication or under the influence of drugs and
unable to give her consent.

70. Ans A
The correct answer is a. Section 375(b) of the Indian Penal Code for having sexual intercourse with a woman without
her consent. In this scenario, Mr. Y spiked Ms. X's drink with a sedative, which caused her to become intoxicated and
unable to give her consent for the sexual act that followed.
Option b is incorrect because it suggests that Mr. Y had sexual intercourse with Ms. X with her consent, when her
consent has been obtained by putting her or any person in whom she is interested in fear of death or of hurt. However,
in this scenario, Mr. Y did not obtain Ms. X's consent through fear or intimidation.
Option c is incorrect because it suggests that Mr. Y had sexual intercourse with Ms. X when she was in a state of
intoxication or under the influence of drugs and unable to give her consent. However, in this scenario, Mr. Y did not
obtain Ms. X's consent at all, and therefore, the fact that she was intoxicated is irrelevant.
Option d is incorrect because it suggests that none of the legal provisions mentioned in the passage can be used to
prosecute Mr. Y. However, as explained above, Section 375(b) of the Indian Penal Code can be used to prosecute Mr.
Y for having sexual intercourse with Ms. X without her consent.

71. Ans A
The correct answer is a. Public Trust Doctrine, as the river is a public resource that should be protected for the
enjoyment of the general public. The Public Trust Doctrine prohibits the government from transferring public
properties, such as rivers, to private parties who may interfere with the interest of the public at large. In this scenario,
the river is a public resource that should be protected for the enjoyment of the villagers, who are suffering from the
pollution caused by Mr. X's factory. Option b is incorrect because it suggests that the Precautionary Principle can be
invoked against Mr. X, as there is an identifiable risk of serious harm to the villagers due to the toxic waste. However,
the Precautionary Principle is aimed at preventing environmental harm, and it is not clear from the scenario whether
Mr. X has taken any measures to prevent harm to the environment or the villagers. Option c is incorrect because it
Legal Reasoning Practice Sheet 87
CLAT POINT

suggests that the Polluter Pays Principle can be invoked against Mr. X, as he is responsible for the damage caused to
the environment and should bear the cost associated with it. However, the Polluter Pays Principle is aimed at shifting
the responsibility from the government to the public, and it is not clear from the scenario whether Mr. X has been
held responsible for the damage caused by his factory. Option d is incorrect because there are legal principles that can
be invoked against Mr. X, as explained above.

72. Ans B
The correct answer is b. The Polluters Pay Principle. This principle requires companies like Astra Pvt Ltd to pay for the
damage caused to the natural environment by their activities. In this case, the factory's disposal of residual waste in
the River Ganges has caused pollution, which has resulted in harm to public health. Therefore, the court should rely
on the Polluters Pay Principle to determine the compensation to be awarded to Mr. X. Option a, The Precautionary
Principle, is incorrect because it primarily emphasizes taking preventive measures to minimize environmental damage
and prevent harm to public health, rather than determining compensation for harm already caused. Option c, The
Public Trust Principle, is incorrect because it primarily holds that certain natural resources belong to the public and
should be protected for the enjoyment of the general public, rather than determining compensation for harm caused
by a private party. Option d, None of the above, is incorrect because the court must rely on legal principles to
determine the compensation to be awarded to Mr. X, and the Polluters Pay Principle is the most appropriate principle
to apply in this case.

73. Ans B
The correct answer is b. The Polluters Pay Principle. The Protection of Wildlife and Waterbodies Act, 2021, empowers
courts to impose fines on offenders based on the extent of damage inflicted on wildlife or waterbodies, reflecting the
application of the Polluters Pay Principle. Option a, The Precautionary Principle, is incorrect because it emphasizes the
importance of preventive measures to minimize environmental damage, rather than assessing liability for the damage
already inflicted. Option c, The Public Trust Principle, is incorrect because it primarily rests on the principle that certain
natural resources belong to the public and should be protected for the enjoyment of the general public, rather than
assessing liability for damage inflicted on natural resources. Option d, The Principle of Sustainable Development, is
incorrect because it emphasizes the need to balance economic development, social development, and environmental
protection, rather than assessing liability for the damage already inflicted.

74. Ans B
The correct answer is b. The Polluters Pay Principle. The Environmental Protection Act empowers the government to
impose fines on offenders based on the extent of damage inflicted on the marine environment, reflecting the
application of the Polluters Pay Principle.
Option a, The Precautionary Principle, is incorrect because it emphasizes the importance of preventive measures to
minimize environmental damage, rather than assessing liability for the damage already inflicted.
Option c, The Public Trust Principle, is incorrect because it primarily rests on the principle that certain natural resources
belong to the public and should be protected for the enjoyment of the general public, rather than assessing liability
for damage inflicted on natural resources.
Option d, The Principle of Inter-generational Equity, is incorrect because it emphasizes the need to balance the needs
of the present and future generations, rather than assessing liability for the damage already inflicted.

75. Ans B
The correct answer is b. The Polluters Pay Principle. This principle requires the company responsible for dumping
hazardous waste in the lake to pay for the harm caused to the residents and the natural environment. The principle
aims to ensure that the costs of pollution are borne by those who cause it, rather than the public at large.
Option a, The Precautionary Principle, is incorrect because it primarily emphasizes taking preventive measures to
minimize environmental damage and prevent harm to public health, rather than determining compensation for harm
already caused.

Legal Reasoning Practice Sheet 88


CLAT POINT

Option c, The Public Trust Principle, is incorrect because it primarily holds that certain natural resources belong to the
public and should be protected for the enjoyment of the general public, rather than determining compensation for
harm caused by a private party.
Option d, The Principle of Inter-generational Equity, is incorrect because it primarily emphasizes balancing the needs
of the present and future generations, rather than determining compensation for harm caused by a private party.

76. Ans C
The correct answer is c. The person can void the contract based on the affective test of mental capacity. The affective
test of mental capacity allows a contract to be voided if one party is unable to act in a reasonable manner and the
other party has reason to know of the condition. In this case, the person's history of mental health issues and substance
abuse may have rendered the person unable to act in a reasonable manner, and the heavy intoxication at the time of
signing the contract may have been a symptom of this condition. Therefore, the person can void the contract based
on the affective test of mental capacity

77. Ans A
The correct answer is a. The contract is valid because Mr. T understood the consequences of the contract despite his
level of intoxication. While voluntary intoxication can sometimes impair an individual's ability to contract, it is not
necessarily a bar to legal capacity. In this case, Mr. T's behavior demonstrated that he had consumed only a small
amount of alcohol and was still able to negotiate with Mr. P efficiently. As such, there is no reason to believe that Mr.
T lacked the capacity to contract.
Option b, The contract is void because Mr. T lacked the capacity to contract due to his history of alcohol abuse, is
incorrect because legal capacity is determined at the time of making the contract, not by a person's history of
substance abuse.
Option c, The contract is voidable because Mr. T's level of intoxication at the time of making the contract rendered
him incapable of understanding the consequences of the contract, is incorrect because Mr. T's level of intoxication did
not render him incapable of understanding the consequences of the contract, as evidenced by his efficient negotiation
with Mr. P.
Option d, The contract is unenforceable because Mr. P had reason to believe that Mr. T was intoxicated at the time of
making the contract, is incorrect because Mr. P's belief about Mr. T's level of intoxication is not sufficient to render
the contract unenforceable if Mr. T had legal capacity to contract at the time of making the contract

78. The correct answer is c. The contract is voidable at the option of Mrs. P because she was coerced into consuming
alcohol and could not understand the meaning of the documents. Coercion, which is defined as the use of force or
threats to compel someone to act in a certain way, can render a contract voidable at the option of the coerced party.
In this case, Mrs. P was coerced into consuming large quantities of alcohol before being asked to sign the contract
documents, which compromised her mental capacity to understand the meaning of the documents. Hence, the
contract is voidable at her option.
Option a, The contract is void because Mrs. P lacked the capacity to contract due to her intoxication, is incorrect
because Mrs. P's lack of capacity was not due to voluntary intoxication, but rather to coercion.
Option b, The contract is valid because Mrs. P signed the documents willingly, even if under duress, is incorrect because
Mrs. P's willingness to sign the documents was compromised by coercion, which can render a contract voidable.
Option d, The contract is voidable at the option of Mr. S because he obtained the title of the house in exchange for
money, is incorrect because the option to void a contract due to coercion lies with the coerced party, not the party
who obtained a benefit from the contract.

79. Ans A
The correct answer is a. The student can void the contract because minors lack the capacity to contract. Minors (those
under the age of 18) lack the legal capacity to make a contract. Therefore, the 17-year-old high school student can
void the contract with the dealership for the purchase of the car. Option b, The student cannot void the contract
because the car was a necessity, is incorrect because minors can void contracts for necessities only if they have not
already been paid for by the minor. Option c, The student cannot void the contract because the student was close to

Legal Reasoning Practice Sheet 89


CLAT POINT

turning 18, is incorrect because the legal age of majority is 18 and until then, minors lack the legal capacity to make a
contract. Option d, The student can void the contract only if the student's mental capacity at the time of signing the
contract was impaired, is incorrect because mental incapacity is a different legal principle that applies to persons who
lack the mental capacity to enter into a contract, and not to minors who lack legal capacity by virtue of their age.

80. Ans D
The correct answer is d. The contract is voidable at the option of Mr. X because he was not mentally competent to
enter into the contract at the time of signing it. Mental incapacity can render a contract voidable at the option of the
person who lacked capacity to enter into the contract. In this case, Mr. X's mental condition worsened after signing
the contract, but it is his mental state at the time of signing the contract that is relevant to his capacity to contract.
Mr. X's history of mental illness may also be a factor in determining his capacity to contract.
Option a, The contract is valid because Mr. X understood the consequences of the contract at the time of signing it, is
incorrect because Mr. X's mental condition worsened after signing the contract, which may have compromised his
capacity to contract.
Option b, The contract is void because Mr. X lacked the capacity to contract due to his history of mental illness, is
incorrect because legal capacity is determined at the time of making the contract, not by a person's history of mental
illness.
Option c, The contract is voidable at the option of Mr. Y because Mr. X's mental condition worsened after the contract
was signed, is incorrect because Mr. Y's option to void the contract is limited to cases of fraud or misrepresentation,
not to issues of mental capacity.

81. Ans. (d)


The correct answer is d. Both options a and c are correct, as they correctly explain that the meeting can be disallowed
under section 144 of the CrPC because it is unlawful in nature and poses a threat to public order and safety. The
individuals' intention to use criminal force against the State government by killing the Chief Minister is a serious threat
to public safety and security, and the meeting's unlawful nature reinforces the need to disallow it.
Option b is incorrect because while the Constitution protects the right to free speech and assembly, these rights are
not absolute and can be restricted in the interest of public safety and order. In this case, the individuals' intention to
commit criminal acts poses a serious threat to public safety and order, justifying the restriction of their rights under
section 144 of the CrPC.

82. Ans. (d)


The correct answer is d. Both options a and b are correct as they correctly explain that the internet shutdown cannot
be allowed under Constitutional provisions because it violates the fundamental right to freedom of speech and
expression, which is protected under the Constitution. Any restriction on this right must be proportionate and justified
under the Constitution, and in this case, the shutdown was disproportionate to the situation and lacked clear evidence
to suggest that the protests were causing a law and order problem in the State.
Option c is incorrect because while the government has the power to take necessary measures to maintain law and
order in the State, these measures must be proportionate and justified under the Constitution. In this case, the internet
shutdown was not proportionate to the situation and lacked clear evidence to suggest that it was necessary to
maintain law and order in the State.

83. Answer: D
Option A is accurate. Freedom of Speech and Expression is a fundamental right protected by the Constitution, and any
restriction on this right must be based on objective facts, necessity, and proportionality. The government cannot
impose measures that are disproportionate or arbitrary and violate the fundamental rights of citizens. Option B is also
accurate. The internet shutdown was disproportionate and arbitrary, and there was no law and order problem due to
the protests. The government cannot impose measures that violate the fundamental rights of citizens and are not
based on objective facts, necessity, and proportionality. Option C is incorrect. While the government has the power to
take measures to maintain public order and safety, these measures must be based on objective facts, necessity, and
proportionality, and cannot violate the fundamental rights of citizens. Option D is the correct response to the question,

Legal Reasoning Practice Sheet 90


CLAT POINT

as it accurately reflects that both options A and B are correct. Such a measure cannot be allowed under Constitutional
provisions due to the protection of fundamental rights and the requirement of necessity and proportionality.

84. Answer: A
Explanation:
Option A is the correct response to the question, as it accurately reflects that the government authorities cannot
impose an indefinite internet shutdown under the Constitution of India because any restrictions on fundamental rights
must be proportionate, necessary, and based on objective facts. The government cannot impose measures that violate
the fundamental rights of citizens and are not based on objective facts, necessity, and proportionality.
Option B is incorrect. While the government authorities have the power to take measures to maintain national
security, these measures must be based on objective facts, necessity, and proportionality, and cannot violate the
fundamental rights of citizens.
Option C is also accurate. The fundamental right of freedom of speech and expression is protected by the Constitution,
and any restriction on this right must be based on objective facts, necessity, and proportionality. The government
cannot impose measures that violate the fundamental rights of citizens and are not based on objective facts, necessity,
and proportionality.
Option D is incorrect. While national security is an important consideration, it cannot take precedence over individual
rights and freedoms protected by the Constitution. Any restriction on fundamental rights must be based on objective
facts, necessity, and proportionality, and cannot be done through an arbitrary exercise of power.

85. Answer: C
Explanation:
Option C is the correct response to the question, as it accurately reflects the correct implementation of Section 144 of
the CrPC and Article 19 of the Constitution in the scenario. The authorities should use Section 144 of the CrPC as a last
resort to maintain law and order, but the restrictions imposed must be based on objective facts, necessity, and
proportionality, as per the recent Supreme Court ruling. The authorities cannot impose arbitrary or blanket restrictions
on the fundamental rights of citizens, as this would violate the Constitution.
Option A is incorrect. While Article 19 of the Constitution guarantees the right to freedom of speech and expression,
any restrictions on this right must be based on objective facts, necessity, and proportionality, and cannot endanger
public safety and order.
Option B is also incorrect. The police authorities cannot use Section 144 of the CrPC to suppress the legitimate
expression of opinion or grievance, as this would violate the fundamental rights of citizens. Any restrictions imposed
under Section 144 must be used as a last resort to maintain law and order and cannot be used to suppress legitimate
expression of opinion or grievance.
Option D is also incorrect. While the police authorities have the duty to prevent any potential threat to public safety
and order, they cannot impose arbitrary or blanket restrictions on the fundamental rights of citizens, as this would
violate the Constitution.

86. Answer: C
Explanation:
Option A is incorrect. While Steve was negligent in driving the train, Stark's own negligence in ignoring the red light
and attempting to cross the road contributed to his injury. The principle of contributory negligence reduces the liability
of the defendant to the extent that the plaintiff contributed to his own injury.
Option B is also incorrect. While it is true that Stark ignored the red light and attempted to cross the road, this does
not absolve Steve of his own negligence in waving to the driver of another train and not stopping the train in time to
avoid the collision. However, Stark's own negligence in the matter should also be taken into account.
Option C is the correct response to the question, as it accurately reflects that Stark has contributed to his own injury,
and therefore his claim will be reduced to the extent of his own negligence. The principle of contributory negligence
is a legal doctrine that reduces the damages payable to a plaintiff in a negligence lawsuit to the extent that the plaintiff
contributed to his own injury.
Option D is incorrect. While it is possible that none of the options are accurate, in this case, option C is the most
accurate response to the question.
Legal Reasoning Practice Sheet 91
CLAT POINT

87. Answer: B
Explanation:
Option A is incorrect. While the shopkeeper did suggest DIAMOND biscuits to Banner, he could not have foreseen
what was inside the biscuit packets. The shopkeeper does not have a duty of care to ensure the quality of the products
sold, as this is the responsibility of the manufacturer.
Option B is the correct response to the question, as it accurately reflects that the DIAMOND Biscuit Company is
responsible for ensuring the quality of their products and owes a duty of care to consumers like Banner. The company
is responsible for any harm caused to consumers by their products, and Banner should seek compensation from them.
Option C is incorrect. While both the shopkeeper and the DIAMOND Biscuit Company played a role in Banner's injury,
the company is primarily responsible for ensuring the quality of their products and owes a duty of care to consumers.
The shopkeeper does not have the same level of responsibility as the manufacturer.
Option D is also incorrect. While it is true that the injury caused by biting into a stone in a biscuit may be considered
trivial and insignificant, Banner still has the right to seek compensation for any harm caused to him by the product.

88. Answer: C
Explanation:
Option A is incorrect. While the burden of proof may shift in certain situations, in this case, the burden of proof is on
Peter to prove that MJ is negligent in the incident. However, the legal principles of negligence make it clear that MJ
owes a duty of care to the people who walk on the road outside her house not to hit them with anything from her
window.
Option B is also incorrect. While Peter has the burden of proof to establish that MJ is negligent, the legal principles of
negligence make it clear that MJ is liable since she owes a duty of care to people walking on the road outside her
house.
Option C is the correct response to the question, as it accurately reflects that MJ is negligent and liable for the harm
caused to Peter. Negligence is defined as the omission to do something which a reasonable person would do or doing
something which a prudent or reasonable person would not do. In this case, MJ failed to take reasonable care to
prevent the bag of flour from falling out of her window, causing harm to Peter.
Option D is incorrect. Based on the legal principles of negligence, MJ owes a duty of care to people walking on the
road outside her house, and she is responsible for ensuring that her actions do not cause harm to others. In this case,
MJ failed to take reasonable care, and thus, she is liable for the harm caused to Peter.

89. Ans- c.
A person cannot claim damages for injury from a risk which he voluntarily undertook. The bus driver has a duty of care
towards all the passengers. The question of who are the valid passengers and who are not is a different question based
more on contractual agreements. This is a question that does not concern us here. The duty of care extends to all
those people that are likely to be affected by the act. This is not a case of volenti non fit injuria because travelling in a
bus is not a risky activity and a person who consents to travel in a bus does not consent to injury by accident. Yes, P
can claim damages from the PSU.

90. Ans- c.
If, as a result of carelessness, one injures another, he is legally liable to the injured victim for resulting damages, unless
the victim's. 'X' was negligent, and to that extent is responsible for Y's injury but Y contributed as well and because the
principle expresses contributory negligence as a complete defence, Y will not get anything.

91. Answer: B
Explanation:
Option A is incorrect. While mental illness can be a ground for divorce under certain circumstances, in this case, there
is no evidence that Mrs. Y was mentally ill. Furthermore, her intelligent answers during the proceedings disprove Mr.
X's charge of mental disorder.

Legal Reasoning Practice Sheet 92


CLAT POINT

Option B is the correct response to the question, as it accurately reflects that there is no evidence that Mrs. Y was
suffering from either mental illness or leprosy. Without such evidence, the court cannot grant a divorce on these
grounds. The Hindu Marriage Act, 1955, allows divorce only when one party is suffering from a virulent and incurable
form of leprosy, or when one party has a communicable form of venereal disease.
Option C is incorrect. While Mrs. Y's fitness for marriage is a relevant consideration, the court cannot grant a divorce
on this ground alone. The relevant grounds for divorce are limited to those specifically enumerated in the Hindu
Marriage Act, 1955.
Option D is also incorrect. One of the options must be the correct response, and in this case, the correct option is B.

92. Answer: B
Explanation:
Option A is incorrect. Syphilis is a communicable disease, and the fact that Mr. X is suffering from syphilis is grounds
for a dissolution of marriage under the Dissolution of Muslim Marriages Act, 1939.
Option B is the correct response to the question, as it accurately reflects that a dissolution of marriage can take place
if the husband is suffering from a venereal disease, such as syphilis, under the Dissolution of Muslim Marriages Act,
1939.
Option C is also incorrect. The facts are sufficient to determine whether there can be a dissolution of marriage, as the
husband is suffering from a venereal disease, which is a ground for dissolution under the Dissolution of Muslim
Marriages Act, 1939.
Option D is also incorrect. One of the options must be the correct response, and in this case, the correct option is B.

93. Answer: A
Explanation:
Option A is the correct response to the question, as it accurately reflects that under the Special Marriage Act, 1954,
the affected party can seek a divorce on the grounds of a communicable venereal disease. The passage clearly states
that the Special Marriage Act, 1954, provides for divorce in such situations.
Option B is incorrect. While maintenance is a relevant consideration in a divorce proceeding, it is not an option that is
available to the unaffected party if one of them contracts a venereal disease under the Special Marriage Act, 1954.
Option C is also incorrect. Restitution of conjugal rights is not an option that is available to the unaffected party if one
of them contracts a venereal disease under the Special Marriage Act, 1954.
Option D is also incorrect. The passage clearly states that the Special Marriage Act, 1954, provides for divorce on the
grounds of a communicable venereal disease. Therefore, the affected party has the option to seek legal remedy, and
the unaffected party has the option to petition for divorce.

94. Answer: b.
Explanation: As per the Indian Divorce Act, 1869, a virulent and incurable form of leprosy is a ground for dissolution
of marriage, provided that the respondent must have been suffering from the disease for not less than two years
immediately preceding the presentation of the petition. Therefore, Mr. Gupta can file for divorce under the Indian
Divorce Act, 1869, as Mrs. Gupta has been diagnosed with an incurable form of leprosy and has been suffering from
the disease for more than two years.
Option a is incorrect as leprosy is a ground for dissolution of marriage under the Indian Divorce Act, 1869. Option c is
incorrect as the respondent must have been suffering from the disease for not less than two years, not three years.
Option d is incorrect as the passage clearly states that venereal disease in a communicable form is a ground for
dissolution of marriage, but it is not applicable in this scenario as Mrs. Gupta is suffering from an incurable form of
leprosy, not a venereal disease.

95. Answer: b. As the marriage of Jack and Jill is invalid, they cannot seek a divorce under the Indian Divorce Act, 1869
as it is only applicable to legally recognized marriages. In such cases, they can only seek a declaration of nullity of
marriage, which is a legal ruling that the marriage was invalid from the beginning. Therefore, Jack cannot seek a divorce
on the grounds of leprosy or venereal disease under the Indian Divorce Act, 1869. Option (a) is incorrect because the
Indian Divorce Act, 1869 is only applicable to legally recognized marriages. Option (c) is incorrect as Jack can seek a

Legal Reasoning Practice Sheet 93


CLAT POINT

declaration of nullity of marriage, but not a divorce. Option (d) is incorrect as the Indian Divorce Act, 1869 is still
applicable to legally recognized marriages.

96. Ans. (b)


Explanation:
As per Section 34(1)(c) of the Special Marriage Act, 1954, the court must satisfy itself that consent for the divorce by
mutual consent has not been obtained by force, fraud or undue influence. In this scenario, since Mrs. Y was being
coerced by Mr. X to agree to the divorce, the court cannot grant the divorce to the couple. Therefore, option B is
correct.
Option A is incorrect as mutual consent cannot be considered valid if it is obtained by force or coercion. Option C is
incorrect as the facts presented in the scenario are sufficient to make a decision that the court cannot grant the
divorce. Option D is incorrect as there is a correct option available, which is option B.

97. Ans. (a)


Explanation:
As per the legal provision mentioned in the passage, the Indian Divorce Act, 1869 does not provide for divorce by
mutual consent. In Reynold Rajamani v. Union of India, where the parties were married under the Indian Christian
Marriage Act, 1872, and their petition for divorce by mutual consent was dismissed, the court held that the ground
could not be availed of by the parties as they were governed by the Indian Divorce Act, 1869. Therefore, option A is
the correct answer.
Option B is incorrect as the Indian Divorce Act, 1869 does not provide for divorce by mutual consent. Option C is
incorrect as the facts presented in the scenario are sufficient to make a decision that the court cannot grant the
divorce. Option D is incorrect as there is a correct option available, which is option A.

98. Explanation:
As per the legal provision mentioned in the passage, the Hindu Marriage Act, 1955 provides for divorce by mutual
consent under Section 13B, but the parties must be living separately for a period of at least one year. Therefore, option
A is the correct answer.
Option B is incorrect as the Hindu Marriage Act, 1955 does provide for divorce by mutual consent. Option C is incorrect
as the couple in the scenario have not completed the requisite of living separately for at least one year. Option D is
incorrect as there is a correct option available, which is option A.

99. Answer: A) No, they cannot seek a divorce by mutual consent under the Special Marriage Act, 1954 since their
marriage8 was not performed under the provisions of that Act.
Explanation: As per the legal provision mentioned in the passage, in order to seek relief under the provisions of the
Special Marriage Act, 1954, it is mandatory that the marriage should either have been performed under the provisions
of that Act or registered thereunder. Therefore, a couple married under the Indian Divorce Act, 1869 cannot seek a
divorce by mutual consent under the Special Marriage Act, 1954. Hence, option A is the correct answer. Option B is
incorrect as it goes against the legal provision mentioned in the passage. Option C is incorrect as it limits the scope of
granting a divorce by mutual consent only to marriages performed under the Hindu Marriage Act, 1955, which is not
true. Option D is incorrect as it is specific to couples married under the Indian Christian Marriage Act, 1872, and does
not cover all other types of marriages.

100. Answer: D) The court cannot grant them a divorce by mutual consent under the Special Marriage Act, 1954 since
the provision for divorce by mutual consent was inserted only in 1976 and cannot have a retrospective effect.
Explanation: The provision for divorce by mutual consent was inserted in the Hindu Marriage Act, 1955, only in 1976.
Hence, a couple married under the Hindu Marriage Act, 1955, cannot seek a divorce by mutual consent under the
Special Marriage Act, 1954. Moreover, since the provision cannot have a retrospective effect, the court cannot grant
them a divorce by mutual consent under the Special Marriage Act, 1954. Therefore, option D is the correct answer.
Option A is incorrect as it does not consider the fact that the couple was married under the Hindu Marriage Act, 1955.
Option B is incorrect as it limits the scope of granting a divorce by mutual consent only to marriages performed under
Legal Reasoning Practice Sheet 94
CLAT POINT

the Special Marriage Act, 1954, which is not true. Option C is incorrect as it does not take into account the retrospective
effect of the provision.

101. Answer: b
Explanation:
Since B kept up his supply of coals but on the new terms and also received payment on the new terms, there was
acceptance on the part of B. Hence c) is the correct answer.
Option a) is incorrect because the company had given payment on the new terms to B. Hence there was acceptance
by conduct on their part.
Option c) is incorrect because B had delivered the supply of coal on the new terms. Hence there was acceptance by
conduct.

102. Answer – d
Explanation
Option d) is the correct answer as there was no overt communication of acceptance between plaintiff and his nephew.
Option a) is incorrect because the mare was not a property of uncle as there was no communication of acceptance
between plaintiff and his nephew.
Option b) is wrong because there was no legally binding contract between the parties on account of no acceptance.
Option c) is wrong because there was no acceptance which was communicated by the nephew to the plaintiff. Mere
silence is not a valid acceptance. It must be actively communicated to the other party.

103. Answer: B) No, since the member of the board communicated the decision of acceptance only in his individual
capacity and not as a formal communication, there was no valid acceptance by the managers.
Explanation: As per the legal principle mentioned in the passage, acceptance must be communicated to the offeror
himself, and the communication of acceptance should be from a person who has the authority to accept. In the
scenario given, the member of the board only communicated the decision in his individual capacity and not as a formal
communication. Therefore, there was no valid acceptance by the managers. Hence, option B is the correct answer.
Option A is incorrect as it assumes that the communication made by the member in his individual capacity is sufficient
to constitute a valid acceptance by the managers. Option C is incorrect as it contradicts the fact that there was no
formal communication of acceptance on the part of the managers. Option D is incorrect as option A is incorrect.

104. Answer: a
Explanation
It is given in the passage that the natural corollary of this principle is that the communication of acceptance should be
from a person who has the authority to accept. Information received from an unauthorised person is ineffective. Hence
option a) is incorrect.
Option b) is incorrect because it is true according to the passage. It is written that Further, acceptance must be
communicated to the offeror himself. A communication to any other person is as ineffectual as if no communication
has been made.
Option c) is wrong because it is true. It is stated in the passage that the natural corollary of this principle is that the
communication of acceptance should be from a person who has the authority to accept. Information received from
an unauthorised person is ineffective.

105. Answer: c.
The legal principle mentioned in the passage is that acceptance must be communicated to the offeror himself and by
a person who has the authority to accept. In this scenario, the vendor's assistant did not have the authority to accept
the offer, and therefore, the acceptance is not valid. The fact that the acceptance was communicated through email
is irrelevant because the key issue is whether the person communicating the acceptance had the authority to do so.
The offer was sent to the vendor, not the vendor's assistant, so the fact that the assistant acted on behalf of the
company does not make the acceptance valid.

Legal Reasoning Practice Sheet 95


CLAT POINT

106. Answer: c
Explanation
Since the agent, i.e., principal clerk was acting in the course of his apparent or ostensible authority, the principal was
liable for the fraud. Hence option c) is the correct answer.
Option a) is wrong because principal clerk was deemed to be an agent of XYZ Partners, there was official
communication.
Option b) is wrong because he did not act in his personal capacity but as an agent of the firm.

107. Answer: b
Explanation
Since employee, when he committed the fraud, was not acting in the scope of bank's employment but in his private
capacity as the depositor's friend, therefore, the defendant bank could not be made liable for the same. Hence option
b) is the correct answer.
Option a) is wrong because the employee had committed the theft in his personal capacity, not as he agent of the
bank.
Option c) is wrong because the plaintiff’s husband had deposited the money to him because he was his friend. Also
there was no receipt returned to him. Thus, he was not deemed to be an agent of the bank.

108. Answer: b
Explanation
Since the younger brother Y had taken out the bike without Mr. Z’s permission or knowledge, he could not be deemed
to be the agent of the owner of the motor cycle and the latter could not be vicariously liable for the accident. Hence,
option b) is the correct answer.
Option a) is wrong because it is immaterial whose bike was it. Mr Y had never asked for Z’s permission to take out the
bike, and he had no knowledge of the accident.
Option c) because it is not true. Z had not given permission to Y to ride his motorcycle.

109. Answer: C) Both Mr. A and Mr. B are jointly and severally liable for Mr. B's false representation.
Explanation: As per the legal principle mentioned in the passage, the act of an agent is the act of the principal. In the
scenario given, Mr. B was acting as Mr. A's agent while making the false representation about the quality of the goods
to Mr. C, a customer. Therefore, both Mr. A and Mr. B are jointly and severally liable for Mr. B's false representation.
Hence, option C is the correct answer.
Option A is incorrect as Mr. A, being the principal, cannot escape liability for Mr. B's false representation. Option B is
incorrect as Mr. B, being the agent, cannot be solely liable for the false representation made in the course of his
agency. Option D is incorrect as the false representation was made to a third party, and both Mr. A and Mr. B are liable
for it as per the legal principle mentioned in the passage.

110. Answer: B) Yes, since Mr. A and Mr. B are partners in the law firm, they can be held liable for each other's torts.
Explanation: As per the legal principle mentioned in the passage, partners can be held liable for each other's torts. In
the scenario given, Mr. A and Mr. B are partners in a law firm, and Mr. A made a false statement to the judge while
representing a client in court. Therefore, Mr. B can be held liable for Mr. A's false statement, as they are partners in
the law firm. Hence, option B is the correct answer.
Option A is incorrect as it assumes that Mr. A made the false statement in his individual capacity, which is not true as
he made the statement while representing the client in court as a partner in the law firm. Option C is incorrect as it
assumes that only Mr. A can be held liable for his false statement, which is not true as per the legal principle
mentioned in the passage. Option D is incorrect as it assumes that Mr. B can only be held liable if he was aware of
the false statement made by Mr. A, which is not necessary as per the legal principle mentioned in the passage.

111. Answer: a
Explanation

Legal Reasoning Practice Sheet 96


CLAT POINT

Plaintiff impliedly took the risk of such injury, the danger being inherent in the sport which any spectator could foresee,
the defendant was not liable. Hence option a) is the correct answer.
Option b) is wrong because car collision was inherent and not unusual for a motor race.
Option c) is wrong because it is irrelevant to the question.

112. Answer: c
Explanation
Mr X is not liable because it was a case of sheer accident and, the strangers had voluntarily got into the jeep and as
such, the principle of volenti non fit injuria was applicable to this case. Hence option c) is the correct answer.
Option a) is wrong because even if X and Y never expected the bolt in the front axle to give way and cause the jeep to
be toppled, they voluntarily got into the jeep which did not belong to them.
Option b) is incorrect because even if the accident was caused by his negligence in maintaining the jeep, X and Y
themselves took the risk of any injury when they got into the jeep voluntarily.

113. Answer d
Explanation
Since defendants had taken due care, they were not liable. The duty of the defendants was the duty of care rather
than the duty of skill. The spectator in such a game or competition takes the risk of such damage even though there
may have been error of judgment or lapse of skill. Hence both a) and b) are correct. Hence d) is the right answer.
Option c) is wrong because it is irrelevant whether horse caused the photographer to be frightened and fall onto the
tracks.

114. Answer: a
Explanation
The defendant was not liable because the danger was visible and the plaintiff appreciated and voluntarily encountered
the same. Hence a) is the correct answer.
Option b) is wrong because it is irrelevant whether it came off unexpectedly. The risk was inherent to the nature of
the work.
Option c) is incorrect because it is irrelevant whether the plaintiff did not know the lid was stuck.

115. Answer: b
Solution
The plaintiff’s consent for the operation did not imply her consent to the removal of the uterus. Hence the defendant
hospital cannot the defence of volenti non fit injuria.
Option a) is wrong because the plaintiff did not give permission for removal of uterus.
Option b) is wrong because lump had no effect on the uterus. Hence it is not true that the uterus had to be removed
according to the procedure.

116. Answer: B) No, Sam cannot succeed because he did not have any apprehension of use of force in his mind, even
if it was due to a mistaken understanding.
Explanation: As per the legal principle mentioned in the passage, the tort of assault occurs when the defendant does
something that causes a reasonable apprehension of battery in the mind of the plaintiff. In this case, Sam did not have
any apprehension of use of force in his mind, even if it was due to a mistaken understanding. Therefore, Sam cannot
succeed in a claim for damages. Peter may have committed some other harm, but he has not committed assault.
Hence, option B is the correct answer.
Option A is incorrect as it assumes that the act of pointing a revolver at someone always constitutes assault, which is
not true as per the legal principle mentioned in the passage. Option C is incorrect as it assumes that Peter's actions
could still be seen as threatening, which is not relevant to the legal principle mentioned in the passage. Option D is
incorrect as there has to be a correct option based on the legal principle mentioned in the passage.

Legal Reasoning Practice Sheet 97


CLAT POINT

117. Answer: A) No, she will not succeed as battery requires trespass to the person, and there is no indication that A
made physical contact with B's mother.
Explanation: As per the legal principle mentioned in the passage, battery requires trespass to the person, which entails
making physical contact with the body of the person aggrieved. In this case, there is no indication that A made physical
contact with B's mother, who filed the case of battery against A. Therefore, B's mother will not succeed in her case.
Hence, option A is the correct answer.
Option B is incorrect as it assumes that A made physical contact with B during the argument, which is not relevant to
the fact that B's mother filed the case of battery against A. Option C is incorrect as it assumes that being B's mother
does not give her the right to file a case for battery against A, which is not true as anyone can file a case for battery if
they have been aggrieved. Option D is incorrect as it assumes that A made physical contact with B during the argument,
which is not relevant to the fact that B's mother filed the case of battery against A.

118. Answer: B) No, Jone will not succeed, as trespass only applies to immovable property, and a car is a movable
property.
Explanation: As per the legal principle mentioned in the passage, trespass means entering upon immovable property
of another person. In this case, Jack entered Jone's car without his permission. However, a car is a movable property,
and therefore, Jack's entry into the car would not amount to trespass as defined in the passage. Hence, Jone will not
succeed in his lawsuit. Therefore, option B is the correct answer.
Option A is incorrect as it assumes that Jack entered Jone's personal property without his permission, which is true
but does not necessarily constitute trespass in this case. Option C is incorrect as it assumes that Jack had no valid
reason for entering Jone's car without permission, which is not relevant to the legal principle mentioned in the
passage. Option D is incorrect as it assumes that Jack committed a wrong by entering someone else's property without
permission, which is true but does not necessarily constitute trespass in this case.

119. Answer: B) No, it is not considered trespass to person as there is an exception in the law that allows for public
interest to trump private interest, especially in cases involving prisoners.
Explanation: As per the legal principle mentioned in the passage, public interest often trumps private interest,
especially in cases involving prisoners. In this case, A is a prisoner who was chained to a window while receiving medical
treatment in the hospital. While this may seem like a violation of A's physical freedom, it is important to note that the
jail authorities may have had good reason to restrict A's movements for the safety and security of the public. Therefore,
it is not considered trespass to person in this case, and option B is the correct answer.
Option A is incorrect as it assumes that A's physical freedom was restrained without his consent, which may be true
but is not necessarily trespass to person in this case. Option C is incorrect as it assumes that A has the sole discretion
to take action against the jail authorities, which may not be true as there are legal limitations to a prisoner's rights.
Option D is incorrect as it does not provide a clear answer and only adds confusion to the situation.

120. Answer: B) No, the owner will not succeed as public interest in protecting the environment trumps the owner's
private interest in excluding others from his property.
Explanation: As per the legal principle mentioned in the passage, public interest often trumps private interest, and this
is especially true in cases where protecting the environment is concerned. In this hypothetical scenario, the
environmentalists entered a privately owned forest to protest against deforestation activities. While the owner may
have the right to control access to his property, the public interest in protecting the environment is considered to be
more important than the private interest of the owner in excluding others from his property. Therefore, the owner
will not succeed in his case, and option B is the correct answer.
Option A is incorrect as it assumes that the environmentalists trespassed onto the owner's private property without
permission, which may be true but is not relevant to the legal principle mentioned in the passage. Option C is incorrect
as it assumes that the owner's "Right to exclude" is absolute, which is not true as there are exceptions to this right.
Option D is incorrect as it assumes that the environmentalists had a valid reason to enter the forest and protest against
deforestation activities, which may be true but does not necessarily address the legal principle mentioned in the
passage.

Legal Reasoning Practice Sheet 98


CLAT POINT

121.

Answer: b

Explanation:

Since Mr. Y was under the influence of Mr. X and not in full possession of his senses while making the decision, this is
a case of undue influence on the part of Mr. X. Additionally, since Mr. X is Mr. Y's spiritual leader, he can easily influence
his decision. Hence, option b) is the correct answer.

Option a) is wrong because, despite Mr. Y's education and positive experiences, Mr. X's influence as a spiritual leader
can still dominate Mr. Y's will.

Option c) is incorrect because the reasonableness of spiritual beliefs is not the central issue in determining undue
influence. The focus should be on whether Mr. Y was under Mr. X's influence while making the decision.

Option d) is not correct because there is no mention of a contractual agreement in the given situation, and even if
there were such an agreement, the presence of undue influence could still render the contract voidable.

122.

Answer: a

Explanation:

According to the passage, undue influence occurs when "one of the parties is in a position to dominate the will of the
other and uses that position to obtain an unfair advantage over the other." In this case, A was in a position to dominate
B's will due to their parent-child relationship and the emotional context of the party, and he misused this position to
obtain a bond for a greater amount than what was due. Therefore, option a) is the correct answer.

Option b) is wrong because, even though B is of legal age, his emotional vulnerability and the context of the celebration
make it possible for A to exert undue influence.

Option c) is incorrect because, despite B's accomplishments, his emotional vulnerability and the influence of his father
still render him susceptible to undue influence.

Option d) is not correct because the focus of the question is on whether undue influence has been exercised, and the
passage shows that A misused his position to dominate B's will and obtain an unfair advantage, which constitutes
undue influence.

123.

Answer: a

Explanation:

According to the passage, undue influence occurs when "one of the parties is in a position to dominate the will of the
other and uses that position to obtain an unfair advantage over the other." In this case, both parties were aware of
the market conditions and A's need for funds, and the contract was entered into after due consideration. There is no
indication that the banker was in a position to dominate A's will or used that position to obtain an unfair advantage.
Hence option a) is the correct answer.

Option b) is incorrect because, although the interest rate was unusually high, there is no indication that the banker
was in a position to exert undue influence on A.

Legal Reasoning Practice Sheet 99


CLAT POINT

Option c) is wrong because, despite A's pressing need for funds, the passage does not mention that A was in a
vulnerable position that would enable the banker to dominate their will.

Option d) is not correct because the banker's familiarity with A's business and the local economy does not necessarily
imply that the banker was in a position to dominate A's will or used that position to obtain an unfair advantage.

124.

Answer: c

Explanation:

According to the passage, "Where a person who is in a position to dominate the will of another, enters into a contract
with him, and the transaction appears, on the face of it or on evidence adduced, to be unconscionable, the burden of
proving that such contract was not induced by undue influence shall lie upon the person in a position to dominate the
will of the other." Since B is the moneylender, he has the burden of proving that he was not in a position to exercise
undue influence on A, making option c) the correct answer.

Option a) is incorrect because the passage does not mention that A must prove the unconscionable nature of the loan
terms, as the burden lies on the person in a position to dominate the will of the other.

Option b) is wrong because it is not A's responsibility to prove that B was in a position to exercise undue influence;
instead, B has the burden to prove that he was not in such a position according to the passage.

Option d) is not correct, as the passage highlights that the burden of proving that the contract was not induced by
undue influence lies on the person in a position to dominate the will of the other (B, in this case).

125.

Answer: d

Explanation:

According to the passage, a contract is said to be induced by "undue influence" where one party is in a position to
dominate the will of the other and uses that position to obtain an unfair advantage. In this scenario, while Mr. Z offered
a lower price for Mr. Y's art collection, there is no indication that Mr. Z was in a position to dominate Mr. Y's will or
used any undue influence over him to accept the offer. Therefore, option d) is correct, as the contract would be
considered valid based on the information provided.

Option a) is incorrect because the passage focuses on whether undue influence was exercised and not merely on
whether the deal was profitable for one party.

Option b) is wrong because, according to the passage, undue influence requires a party to be in a position to dominate
the will of the other. There is no indication that Mr. Z was in such a position.

Option c) is not correct because there is no indication that Mr. Z was in a position to dominate Mr. Y's will or that the
transaction was unconscionable due to undue influence. The mere existence of a disparity in the transaction is not
sufficient to establish undue influence.

126.

Answer: a

Legal Reasoning Practice Sheet 100


CLAT POINT

Explanation:

Despite the additional details, the core issue remains the same: both A and B were unaware that the painting was a
forgery at the time of the contract. This mistake relates to the nature of the subject-matter of the contract, which is
one of the essential facts in an agreement, as mentioned in the passage. Hence, option a) is true.

Option b) is wrong because even though both parties were mistaken, and A relied on the certificate of authenticity, it
does not negate the fact that there was a mistake as to the nature of the subject-matter of the contract, which makes
the agreement void.

Option c) is wrong because the mistake still exists, regardless of the certificate of authenticity. Both parties were
unaware that the painting was a forgery at the time of the contract, leading to a mistake as to the nature of the subject-
matter of the contract.

Option d) is incorrect because the mistake is essential to the agreement. The mistake regarding the painting's
authenticity directly affects the nature of the subject-matter of the contract.

127.

Answer: b

Explanation:

As the individual who claimed to be a partner of Stellar Fabrics Inc. was not, in fact, a member or agent of the firm,
there was a mistake as to the identity of the person involved in the contract. The plaintiffs intended to contract with
Stellar Fabrics Inc., but this never happened because the person they dealt with was not part of the firm or authorized
to act on its behalf. Consequently, there was never an agreement between two consenting minds on the same terms.
Therefore, there was no valid contract, and option b) is the correct answer.

Option a) is incorrect because the plaintiffs were mistaken as to the identity of the person who represented himself
as a partner of Stellar Fabrics Inc. Thus, there was a mistake of identity.

Option c) is wrong because the mistake of identity renders the contract void, not voidable at the option of any party.

Option d) is incorrect because the underlying contract was invalid due to the mistake of identity, which means the
defendant's purchase of the goods does not preclude the plaintiffs from seeking recovery.

128.

Answer: a

Explanation:

The plaintiffs intended to enter into a contract with the reputable firm Jamie & Co but were misled by the fraudulent
individual James, who used a similar name and address. As the plaintiffs did not intend to contract with James, there
was a mistake as to the identity in the contract. This mistake renders the contract void. Therefore, option a) is the
correct answer.

Option b) is incorrect because the plaintiffs were mistaken as to the identity of the contracting party due to James'
fraudulent actions. The contract was not valid.

Option c) is wrong because the mistake of identity renders the contract void, not voidable at the option of any party.

Option d) is incorrect because the contract is void due to the mistake as to the identity, and the plaintiffs cannot sue
James for breach of contract.
Legal Reasoning Practice Sheet 101
CLAT POINT

129.

Answer: a

Explanation:

According to the passage, the identity and nature of the subject-matter of the contract are essential facts for every
agreement. In this case, there was a mistake in the subject-matter of the contract (painting a portrait of Mr. B's father
vs. Mr. B's uncle). As stated in Raffles v Wichelhaus, "if two persons enter into an apparent contract concerning a
particular person or ship, and it turns out that each of them, misled by a similarity of name, had a different person or
ship in mind, no contract would exist between them." Therefore, option a) is the correct answer because the contract
is void due to the mistake in the subject-matter.

Option b) is incorrect because, despite the agreement on the terms of the contract, the mistake in the subject-matter
is an essential fact that renders the contract void.

Option c) is wrong because, while Ms. A did not deliver the expected work, the focus should be on the mistake in the
subject-matter, which makes the contract void.

Option d) is incorrect because, according to the passage, any mistake in the subject-matter, which is essential to the
agreement, can render the contract void, regardless of the significance of the mistake.

130.

Answer: a

Explanation:

According to the passage, the identity of the parties is one of the essential facts for every agreement. In this case,
there was a mistake in the identity of the publishing company (Creative Publishers vs. Innovative Publishers). As
mentioned in the passage, "if you propose to make a contract with A, then B cannot substitute himself for A without
your consent and to your disadvantage, securing to himself all the benefit of the contract." Therefore, option a) is the
correct answer because the contract is void due to the mistake in the identity of the publishing company.

Option b) is incorrect because, as stated in the passage, the identity of the parties is essential to the agreement, and
a mistake in this aspect can render the contract void.

Option c) is wrong because, while Mr. Y used the wrong name by mistake, the focus should be on the mistake in the
identity of the publishing company, which makes the contract void.

Option d) is incorrect because, despite the agreement on the terms of the contract, the mistake in the identity of the
publishing company is an essential fact that renders the contract void.

131.

Answer: b Explanation: Option a is incorrect because the fact that Mr. F's innovation has caused substantial harm to
Mr. E's business is not the sole determining factor for the success of the suit. It must also be established that there
was a violation of a legal right. Option b is correct because, as per the passage's explanation of "damnum sine injuria,"
even though Mr. E suffered harm (decline in sales), there was no violation of a legal right, as Mr. F's new bread-making
technique was not against any law. Option c is incorrect because there is no indication in the question that Mr. F's
lower prices constituted an unfair competitive advantage. Option d is incorrect because the focus of the question is
on the existence of a legal right violation, not on Mr. E's responsibility to adapt to market changes.
Legal Reasoning Practice Sheet 102
CLAT POINT

132.

Answer: c

Explanation:

Option a is incorrect because the benefits to the community are not relevant in determining whether a legal right has
been violated.

Option b is incorrect because the zoning law, as part of the town's regulatory authority, is not considered an unlawful
restriction on Ms. A's property rights.

Option c is correct because, as per the passage's explanation of "damnum sine injuria," even though Ms. A suffered
harm (the removal of her garden), there was no violation of a legal right as the garden was against the zoning law.

Option d is incorrect because, while the town authorities were enforcing the zoning laws, the key factor for
consideration is whether a legal right was violated or not, not the enforcement of the law itself.

133.

Ans- a.

In case of injuria sine damno, the loss suffered by the plaintiff is not relevant for the purpose of a cause of action. It
may be relevant only as regards the measure of damages. If the plaintiff has suffered no harm and yet the wrongful
act is actionable, the question which arises is how much compensation is to be paid to the plaintiff? In such a case,
generally, nominal damages may be awarded. The member was deprived of his constitutional right to attend the
Assembly session. There was violation of fundamental right to personal liberty guaranteed under Article 21 of the
Constitution. By way of consequential relief, exemplary damages amounting to certain Rs. Will be awarded to him.
MLA falls under the category of Public Servant and a prior warrant with the approval of magistrate is required. Hence
option b is wrong. Proven guilty has nothing to do with getting arrest. Arrest is the initial part of legal proceedings and
proven guilty is part of judgement which is the end of the proceedings. Hence option c is flawed. D is irrelevant and
just a opinion based statement.

134.

Answer: b Explanation: Option a is incorrect because the fact that Mr. C's actions were in direct competition with Mr.
D's business is not relevant to whether a legal right was violated. Option b is correct because, as per the passage's
explanation of "damnum sine injuria," even though Mr. D suffered harm (loss of revenue), there was no violation of a
legal right as Mr. C's outdoor exhibition was not against any law. Option c is incorrect because there is no indication
in the question that Mr. C's actions constituted an unfair business practice. Option d is incorrect because the focus of
the question is on the existence of a legal right violation, not on Mr. D's responsibility to adapt to market changes.

135.

Ans- c

In order for an action to be labelled a tort, there must be an act or an omission and a legal damage resulting from that
act or omission. In this case, there seems to be the commission of an act and legal damage (or injury) arising out of
the same, but on closer examination it emerges that:

Legal Reasoning Practice Sheet 103


CLAT POINT

The act is not necessarily something that was not supposed to be done. Every man is entitled to mow his law with a
properly functioning electric lawnmower as long as he does not do it at some unearthly hour, thus waking up the
entire neighbourhood. Therefore, this case would be one of damnum sine injuria where your neighbour might have
been injured but because his legal right has not been violated, he has no valid claim in law. Therefore, your neighbour's
claim will not succeed.

136.

Correct Answer: B (Reference– In applying the reasonable person standard, courts often consider factors such as the
foreseeability of harm, the magnitude of the risk, the burden of taking precautions, and the social utility of the
defendant's conduct.)

Explanation:

a) The surgeon's subjective belief about the reasonableness of their conduct is not considered under the reasonable
person standard, as it is an objective standard.

b) The foreseeability of harm from rushing through surgeries and the potential for complications is an important factor
in determining if the surgeon's conduct was reasonable under the reasonable person standard.

c) The social utility of the surgeon's conduct is a relevant factor, but the issue of rushing through surgeries and the
potential complications is more directly related to the negligence claim.

d) The burden of taking precautions, such as limiting the number of surgeries performed in one day, is a relevant
factor. However, the foreseeability of harm from rushing through surgeries and the potential for complications is more
directly related to the negligence claim.

137.

Correct Answer: B (Reference: In some cases, courts may also consider whether the defendant had any special skills
or knowledge that would affect the reasonableness of their conduct. For example, a professional, such as a doctor or
engineer, may be held to a higher standard of care than an ordinary person in certain situations)

Explanation:

a) Although the electrician's adherence to electrical codes and guidelines is relevant, the court would primarily focus
on the electrician's specialized skills and knowledge, as they are held to a higher standard of care than an ordinary
person.

b) The electrician's specialized skills and knowledge as a licensed professional are most relevant, as professionals may
be held to a higher standard of care than an ordinary person in certain situations.

c) The homeowner's subjective belief about the reasonableness of the electrician's conduct is not considered under
the reasonable person standard, as it is an objective standard.

d) The social utility of the electrician's conduct is a relevant factor, but the electrician's specialized skills and knowledge
as a licensed professional are more directly related to the negligence claim.

138.

Correct Answer: B (Reference: The reasonable person standard is a negligence standard that focuses on the
reasonableness of the defendant's conduct.)

Explanation:
Legal Reasoning Practice Sheet 104
CLAT POINT

a) The strict liability standard is not applicable in this scenario, as the reasonable person standard is a negligence
standard focusing on the reasonableness of the defendant's conduct.

b) The company can be held liable for negligence because their failure to properly inspect the bungee cord indicates
unreasonable conduct. The reasonable person standard considers the foreseeability of harm, the magnitude of the
risk, and the burden of taking additional precautions.

c) The company's subjective belief about the reasonableness of their conduct is not considered under the reasonable
person standard, as it is an objective standard.

d) Although the law aims to strike a balance between protecting individuals from harm and allowing companies to
engage in activities that may have some risk of harm, the specific question of the company's reasonableness is most
directly addressed by evaluating the foreseeability of harm, the magnitude of the risk, and the burden of taking
additional precautions. In this case, the company's failure to properly inspect the bungee cord may be considered
unreasonable, making them potentially liable for negligence.

139.

Correct Answer: B (Reference: The reasonable person standard is also an objective standard, which means that it does
not take into account the subjective beliefs or intentions of the defendant. Instead, the standard focuses on what a
reasonable person would do in the defendant's situation, taking into account the objective circumstances of the case.)

Explanation:

a) The store owner's subjective belief about taking reasonable precautions is not considered under the reasonable
person standard, as it is an objective standard.

b) The store owner can be held liable for negligence because the court would focus on what a reasonable person would
do in the store owner's situation, considering the objective circumstances of the case, such as the risk of harm and the
burden of taking precautions.

c) The store owner's actions and intentions are not enough to escape liability under the reasonable person standard,
as it is an objective standard that considers what a reasonable person would do in the given situation.

d) The court would not consider the store owner's subjective belief that they were taking reasonable precautions and
their intentions, as the reasonable person standard is an objective standard that focuses on the objective
circumstances of the case.

140.

Correct Answer: B (Reference: The reasonable person standard is also an objective standard, which means that it does
not take into account the subjective beliefs or intentions of the defendant. Instead, the standard focuses on what a
reasonable person would do in the defendant's situation, taking into account the objective circumstances of the case.)

Explanation:

a) The store owner's subjective belief about taking reasonable precautions is not considered under the reasonable
person standard, as it is an objective standard.

b) The store owner can be held liable for negligence because the court would focus on what a reasonable person would
do in the store owner's situation, considering the objective circumstances of the case, such as the risk of harm and the
burden of taking precautions, including the proper placement of the "wet floor" sign.

Legal Reasoning Practice Sheet 105


CLAT POINT

c) The store owner's actions and intentions are not enough to escape liability under the reasonable person standard,
as it is an objective standard that considers what a reasonable person would do in the given situation, including
ensuring the proper placement and visibility of the "wet floor" sign.

d) The court would not consider the store owner's subjective belief that they were taking reasonable precautions and
their intentions, as the reasonable person standard is an objective standard that focuses on the objective
circumstances of the case.

141.

Correct Answer – C (Reference lines: "This can include actions such as making sexual advances, using vulgar language,
or making unwelcome physical contact. The act must be done with the intention of outraging the woman's modesty
or knowing that it is likely to do so.")

Explanation:

I: A accidentally brushes against E's arm but has no intention of outraging her modesty. He apologizes and explains his
research, indicating that the contact was unintentional.

II: B's poem contains explicit language and imagery but is not directed at E or any specific individual. Although it may
be offensive to some, it does not meet the criteria of outraging the modesty of a woman as defined in the passage.

III: C's academic paper quotes an offensive statement directed at E, but C's intention is not to outrage E's modesty.
The paper is published in an academic context, and the focus is on analyzing language rather than perpetuating harm.

IV: D creates an avatar resembling E without her consent and engages in explicit interactions using her likeness. This
action can be considered a sexual advance, and it is done knowing that it is likely to outrage E's modesty. Thus, this
option fulfills the criteria defined in the passage.

Therefore, C is correct here.

142.

Correct Answer: C(Reference lines: "Criminal force means the use of physical force against the woman, such as
touching her inappropriately or restraining her.")

Explanation:

I: A accidentally touches E inappropriately while adjusting her posture, but he immediately apologizes and ensures not
to repeat the action. This does not fulfill the criteria of using criminal force with the intention of outraging E's modesty.

II: A uses a metaphorical description involving intimate imagery, which E finds offensive. However, this does not involve
any physical contact, and the passage specifies that criminal force involves the use of physical force. Therefore, A
would not be held liable in this case.

III: A intentionally touches E inappropriately while assisting her with a dance move, exploiting the close physical
contact required in the dance routine. This action constitutes the use of physical force against E with the intention of
outraging her modesty, making A liable as per the passage.

IV: A loses his grip during a complex lift, causing E to fall. A's actions were unintentional. This does not meet the criteria
of using criminal force with the intention of outraging E's modesty.

143.

Legal Reasoning Practice Sheet 106


CLAT POINT

Correct Answer: Option A (Reference lines from the passage: "Criminal force means the use of physical force against
the woman, such as touching her inappropriately or restraining her.")

Explanation:

Option A: A intentionally touches E inappropriately, exploiting the close physical contact required in the dance routine.
Even though A misinterprets E's body language and believes that she has given consent, his actions still constitute the
use of physical force against E with the intention of outraging her modesty, making A liable as per the passage.

Option B: A's genuine belief that E had given consent does not negate his intentional inappropriate touching, which
still constitutes the use of criminal force as per the passage.

Option C: A's liability is not contingent on E explicitly communicating her lack of consent after the incident. The passage
specifies that the act must be done with the intention of outraging the woman's modesty or knowing that it is likely
to do so. A's intentional inappropriate touching fulfills this criterion.

Option D: Although the close physical contact is a necessary part of the advanced dance class, A's exploitation of the
situation to intentionally touch E inappropriately still constitutes criminal force as defined in the passage.
Misinterpretation of consent does not exempt A from liability.

144.

Correct Answer: Option B (Reference lines: "The act must be done with the intention of outraging the woman's
modesty or knowing that it is likely to do so.")

Explanation:

Option A: The passage specifies that the act must be done with the intention of outraging the woman's modesty or
knowing that it is likely to do so. A's actions were unintentional and based on his genuine belief that the contact was
necessary, which does not fulfill this criterion.

Option B: A genuinely believed that the contact was necessary for executing the dance move, even though he
inadvertently touched E inappropriately. His actions were unintentional, which means he would not likely be held
liable for using criminal force with the intention of outraging E's modesty.

Option C: A's liability is not contingent on E explicitly communicating her lack of consent after the incident. The passage
specifies that the act must be done with the intention of outraging the woman's modesty or knowing that it is likely
to do so. A's unintentional inappropriate touching does not fulfill this criterion.

Option D: Although the close physical contact is a necessary part of the advanced dance class, the focus should be on
A's intentions, which were not to outrage E's modesty. The fact that A inadvertently touched E inappropriately while
genuinely believing the contact was necessary supports the conclusion that he would not likely be held liable.

145.

Correct Answer: Option A(Reference lines from the passage: "The offence is gender-specific and can only be
committed against a woman. The section is an important tool for protecting women from sexual harassment and
assault. It is often used in cases where a woman has been groped or molested in a public place, or where a man has
made lewd comments or gestures towards a woman.”)

Explanation:

Option A: G cannot be held liable under Section 354, as the offense is gender-specific and can only be committed by a
man against a woman. In this scenario, G, a woman, has committed the act against another woman, H.

Legal Reasoning Practice Sheet 107


CLAT POINT

Option B: Although G's actions may be morally and ethically wrong, the provided passage states that the offense under
Section 354 is gender-specific and can only be committed by a man against a woman. Thus, G cannot be held liable
under Section 354.

Option C: The liability under Section 354 is not contingent on H's explicit communication of lack of consent or other
attendees' testimonies, but rather on the gender of the offender. As G is a woman, she cannot be held liable under
Section 354.

Option D: While it is possible that a jurisdiction may expand the scope of the offense to include acts committed by
women against other women, the passage provided specifically states that the offense is gender-specific and can only
be committed by a man against a woman. Based on this passage, G cannot be held liable under Section 354.

146.

Correct Answer: Option B (Reference lines : "Article 19(1)(a) states that all citizens shall have the right to freedom of
speech and expression. This right extends to any form of communication, including spoken words, written text, and
various forms of media, such as radio, television, and the internet.")

Explanation:

Option A: Although Article 19(2) allows for reasonable restrictions on the right to freedom of speech and expression,
there is no indication in the scenario that discussing economic policies would pose a threat to the sovereignty and
integrity of India. Therefore, this option is incorrect.

Option B: The regulation restricts Journalist X's right to freedom of speech and expression, guaranteed by Article
19(1)(a), without providing a justifiable reason within the parameters of Article 19(2). This makes the regulation invalid
and supports Journalist X's claim. This option is correct.

Option C: While the proportionality test is relevant for assessing the constitutionality of restrictions on free speech,
the regulation in the scenario does not provide a justifiable reason under Article 19(2) for restricting discussions on
economic policies. Therefore, this option is incorrect.

Option D: Although the regulation targets online publications, the focus of the question is on the validity of the
restriction on the freedom of speech and expression under Article 19(1)(a). The principle of equality before the law is
not the primary concern in this scenario. Therefore, this option is incorrect.

147.

Correct Answer: Option A (Reference lines: "Article 19(2) outlines the reasonable restrictions that can be imposed on
this right in the interest of the sovereignty and integrity of India, the security of the state, friendly relations with foreign
states, public order, decency, or morality, or in relation to contempt of court, defamation, or incitement to an
offense.")

Explanation:

Option A: Y's speech contains inflammatory remarks and encourages civil disobedience, which could potentially disrupt
public order. As a result, the government's argument for restricting Y's speech under Article 19(2) is valid. This option
is correct.

Option B: Although freedom of speech is a fundamental right, it is subject to reasonable restrictions outlined in Article
19(2). Y's speech, which contains inflammatory remarks and encourages civil disobedience, does not enjoy absolute
protection. Therefore, this option is incorrect.

Legal Reasoning Practice Sheet 108


CLAT POINT

Option C: While incitement to an offense is a valid ground for restriction under Article 19(2), Y's speech primarily
encourages civil disobedience and poses a threat to public order. Although this option is related, it is not the best
answer. Thus, this option is incorrect.

Option D: Peaceful protest is protected under the right to freedom of speech and expression. However, Y's speech
contains inflammatory remarks and encourages civil disobedience, potentially disrupting public order. Therefore, this
option is incorrect.

148.

Correct Answer: Option B (Reference lines: "One such principle is the 'clear and present danger' test, which requires
the restriction to be necessary to prevent a clear and imminent threat to public order or national security.")

Explanation:

Option A: Although the documentary sparks debates and calls for protests, it does not pose a clear and imminent
threat to public order or national security. The "clear and present danger" test is not satisfied in this case, making this
option incorrect.

Option B: The "clear and present danger" test requires a clear and imminent threat to public order or national security
to justify a restriction on free speech. As the documentary investigates alleged corruption, it does not meet this
threshold. This option is correct.

Option C: While defamation is a valid ground for restriction under Article 19(2), the scenario does not mention
defamatory content in the documentary. The focus should be on the "clear and present danger" test, making this
option incorrect.

Option D: Freedom of speech and expression is subject to reasonable restrictions, and the "clear and present danger"
test is a relevant principle in determining the constitutionality of restrictions. This option does not address the test
and is therefore incorrect.

149.

Correct Answer: Option B(Reference line: "Another important principle is the 'proportionality' test, which mandates
that the restriction should be proportional to the objective it seeks to achieve and should not impose an excessive
burden on the right to free speech.")

Explanation:

Option A: While the government's objective of maintaining public order and decency is valid, an outright ban on the
book may not be proportional to the objective, as it imposes an excessive burden on the right to free speech. This
option is incorrect.

Option B: The "proportionality" test requires that the restriction be proportional to the objective and not impose an
excessive burden on the right to free speech. An outright ban on the book is excessive, as it disproportionately restricts
the right to free speech. This option is correct.

Option C: Although friendly relations with foreign states are a valid ground for restriction under Article 19(2), the
scenario focuses on tensions between religious communities within the country, not foreign relations. This option is
incorrect.

Option D: Freedom of speech and expression is subject to reasonable restrictions, and the "proportionality" test is a
relevant principle in determining the constitutionality of restrictions. This option does not address the test and is
therefore incorrect.
Legal Reasoning Practice Sheet 109
CLAT POINT

150.

Correct Answer: C (Reference Lines: "These include hate speech, obscenity, defamation, and speech that incites
violence or undermines national security. The courts have emphasized that the right to free speech does not extend
to the propagation of ideas that are prejudicial to the public interest or that promote enmity between different
groups."

Explanation

A) This option suggests that the activist's speech leads to increased political engagement among one ethnic group.
Although it might result in a shift in the region's balance of power, the speech itself does not explicitly promote enmity
between the two ethnic groups or incite violence.

B) This option portrays the activist's speech as an attempt to address socio-economic disparities and propose policies
to foster integration between the two ethnic groups. It does not suggest that the speech promotes enmity or incites
violence, and therefore does not provide a valid justification for limiting the activist's freedom of speech.

C) This option is the correct answer because it directly states that the activist's speech promotes enmity between the
two ethnic groups and results in a series of violent clashes. This aligns with the passage's reference to speech that
incites violence or promotes enmity between different groups as a valid justification for limiting freedom of speech.

D) This option involves the activist's speech criticizing the government and calling for a peaceful protest. While it could
potentially be viewed as undermining national security, it does not directly incite violence or promote enmity between
the two ethnic groups. Thus, it is a less valid justification for limiting the activist's freedom of speech compared to
Option C.

151.

Correct Answer: B (Reference Lines: "The Act allows the state to detain a person without a formal charge and without
trial to prevent them from acting in any manner prejudicial to the security of the state or for the maintenance of public
order."

Explanation:

A) This option is incorrect because, while the passage mentions that the detention is not based on specific allegations
or for a specific violation of the law, it does not mean that X's claim will succeed. The Act allows for such detentions in
certain circumstances.

B) This is the correct option because the passage clearly states that the National Security Act allows the state to detain
a person without a formal charge or trial in order to prevent them from acting in any manner prejudicial to the security
of the state or for the maintenance of public order. X's actions could fall under these criteria.

C) This option is incorrect because being a social activist does not exempt X from the provisions of the National Security
Act. The Act does not distinguish between social activists and other individuals when it comes to detentions.

D) This option is incorrect because, while organizing protests may be seen as prejudicial to the security of the state or
for the maintenance of public order, the key factor is that the Act allows for such detentions. Therefore, X's claim
would not necessarily succeed based on this reasoning alone.

The correct answer, Option B, not only acknowledges the fact that the Act allows the state to detain a person to
prevent them from acting in any manner prejudicial to the security of the state or for the maintenance of public order
but also explains that this is the main reason X's claim would not succeed. Option B considers the provisions of the Act

Legal Reasoning Practice Sheet 110


CLAT POINT

as a whole, rather than focusing solely on the nature of X's actions, which makes it a more comprehensive and accurate
response.

152.

Correct Answer: C(Reference Lines: "The Act allows the state to detain a person without a formal charge and without
trial to prevent them from acting in any manner prejudicial to the security of the state or for the maintenance of public
order.")

Explanation:

A) This option is incorrect because, while the passage mentions that the detention is not based on specific allegations
or for a specific violation of the law, it does not address the fact that X's actions have not caused any harm to the
security of the state or the maintenance of public order. The absence of specific allegations or violations alone is not
enough to support X's claim, as the National Security Act allows for such detentions in certain circumstances.

B) This option is incorrect because, although the Act allows the state to detain a person to prevent them from acting
in any manner prejudicial to the security of the state or for the maintenance of public order, there is no evidence to
suggest that X's actions have caused or are likely to cause any harm to the security of the state or the maintenance of
public order. In this case, the detention would not be justified under the Act.

C) This is the correct option because X's peaceful protests do not qualify as prejudicial to the security of the state or
for the maintenance of public order, and there is no evidence to suggest otherwise. Given these circumstances, the
detention of X would not be justified under the National Security Act, and X's claim that the detention violates his
fundamental rights would likely succeed.

D) This option is incorrect because, while organizing protests can be considered prejudicial to the security of the state
or for the maintenance of public order, it doesn't take into account the peaceful nature of the protests and the lack of
evidence suggesting any harm to the security of the state or the maintenance of public order. Merely organizing
protests is not sufficient grounds for detention under the National Security Act if the protests are peaceful and do not
pose a threat to security or public order.

153.

Correct Answer: A

Reference Lines: "it can also be applied to prevent a person from acting in any manner prejudicial to the maintenance
of supply and services essential to the community."

Explanation:

A) This is the correct option because the passage states that the NSA can be applied to prevent a person from acting
in any manner prejudicial to the maintenance of supply and services essential to the community. X's actions in
organizing the protest led to disruptions in the supply chain, which falls under this category, making X liable under the
NSA.

B) This option is incorrect because the passage mentions that a detained person can be held for 10 to 12 days in special
circumstances without being told the charges against them. Since X was held for 11 days, which falls within this range,
the detention does not violate the provisions of the NSA.

C) This option is incorrect because, although the NSA allows for detention without charge for a maximum period of 12
months, the passage does not specifically mention that X's actions can be considered prejudicial to the security of
India. X's actions disrupted the supply chain of essential goods and services to the community, but this does not
necessarily relate to the security of India.

Legal Reasoning Practice Sheet 111


CLAT POINT

D) This option is incorrect because it focuses on the defense of India and relations with foreign powers, which are not
relevant to the given situation. The passage states that the NSA can be applied to prevent a person from acting in any
manner prejudicial to the maintenance of supply and services essential to the community, which is the pertinent
provision in this scenario.

154. Correct Answer: C (Reference Lines: "The NSA can be invoked to prevent a person from acting in any manner
prejudicial to the defence of India, relations of India with foreign powers or the security of India. Among others, it can
also be applied to prevent a person from acting in any manner prejudicial to the maintenance of supply and services
essential to the community.")

Explanation:

A) This option is partially correct, but it does not provide a comprehensive explanation. While it correctly states that
X's actions did not disrupt the supply chain of essential goods and services or affect other aspects mentioned in the
NSA, it does not address whether the detention is lawful under the NSA.

B) This option is incorrect because the passage states that the NSA can be invoked only when a person's actions are
prejudicial to specific categories. X's actions in organizing the peaceful protest do not fall under any of these categories,
and the potential for future disruptions is not a valid reason for detention under the NSA.

C) This is the correct option because X's actions do not fall within the scope of any of the categories mentioned in the
NSA for detention without charge. Since the peaceful protest did not have any impact on the defense of India, relations
of India with foreign powers, the security of India, or the maintenance of supply and services essential to the
community, X's detention is unlawful under the NSA, and he will not be held liable.

D) This option is incorrect because it implies that X's actions can be considered a potential threat under the NSA.
However, the passage states that the NSA can be invoked only when a person's actions are prejudicial to specific
categories, and there is no evidence to suggest that X's actions pose any such threat.

155. Correct Answer: C (Reference Lines: "One crucial procedural safeguard under the NSA is granted under Article
22(5), where all the detained persons have the right to make an effective representation before an independent
advisory board.")

Explanation:

A) This option is incorrect because, although Article 22 of the Indian Constitution provides protection against arrest
and detention in certain cases, Article 22(3) specifically states that these rights are not applicable in cases of preventive
detention.

B) This option is incorrect because it only addresses the non-applicability of Article 22 rights in cases of preventive
detention, as specified in Article 22(3). However, it does not consider the procedural safeguard granted by Article
22(5), which allows detained persons to make an effective representation before an independent advisory board.

C) This is the correct option because A has not been given the opportunity to make an effective representation before
an independent advisory board, which is a crucial procedural safeguard granted under Article 22(5) of the Indian
Constitution. As A has been denied this right, he is likely to succeed in his claim.

D) This option is incorrect because it focuses on the right to consult and be defended by a legal practitioner of one's
choice, which is not the main issue in this scenario. The crucial procedural safeguard in question is the right to make
an effective representation before an independent advisory board, as mandated by Article 22(5) of the Indian
Constitution.

Legal Reasoning Practice Sheet 112


CLAT POINT

156. Answer: b

Explanation:

According to the passage, an illegal agreement is one that is actually forbidden by law, and every illegal contract is also
void. In this case, the contract between Mr. X and Mr. Y involves bribery, which is forbidden by law, making the
consideration illegal and the contract void. Therefore, option b) is the correct answer.

Option a) is incorrect because, despite the presence of mutual understanding and consideration, the consideration is
illegal, and thus the contract is void.

Option c) is incorrect because, although the passage does mention that the Indian Contract Act does not recognize
unlawful agreements and categorizes them as void, this option does not specifically address the illegality of the
contract in question.

Option d) is incorrect because, although Mr. X may be a victim of fraud, the contract itself involves an illegal act
(bribery) and is therefore void and unenforceable.

157. Answer: c

Explanation:

As per the passage, a contract with an illegal purpose is void according to Section 23 of the Indian Contract Act, 1872.
In this case, the contract's purpose is blasphemous and illegal, rendering the contract void. Hence, option c) is correct.

Option a) is incorrect because the passage indicates that a contract with an illegal purpose is void, irrespective of
whether the illegality was hidden or not.

Option b) is incorrect because Section 23 makes the contract void rather than voidable at the option of either party
due to the illegal nature of the contract.

Option d) is incorrect because the contract is void due to its illegal purpose, regardless of whether Mr. L claims to have
changed his intentions.

158. Answer: c

Explanation

Since Mr X repented and sought to recover back the property but before any creditor could be deceived, he can be
allowed to recover as per the exception to the Section 23 provided in the passage.

Option a) is wrong because he sought to recover the property before any creditor could be deceived. Hence no illegal
purpose has been carried into effect.

Option b) is wrong because even if there was an illegal consideration to begin with, the contract is still executory and
no illegal purpose has been carried into effect.

159. Answer: c

Explanation:

Legal Reasoning Practice Sheet 113


CLAT POINT

According to the passage, a contract with an illegal purpose is void as per Section 23 of the Indian Contract Act, 1872.
In this case, the purpose of the contract was illegal since it involved using substandard materials and endangered
public safety. Therefore, option c) is correct.

Option a) is incorrect because the passage states that a contract with an illegal purpose is void, regardless of the
consent and consideration provided by the parties.

Option b) is incorrect because, as per the passage, an illegal contract is void rather than voidable at the option of either
party. Moreover, there is no indication that Mr. A was under pressure from Mr. B.

Option d) is incorrect because the contract is void due to its illegal purpose. Mr. B's claim regarding minimum safety
requirements is irrelevant in this context, as the contract is still void.

160. Answer: c

Explanation:

According to the passage, a contract with an illegal purpose is void as per Section 23 of the Indian Contract Act, 1872.
In this case, the purpose of the contract was illegal since it involved developing and selling a drug not approved by the
government. Therefore, option c) is correct.

Option a) is incorrect because, as per the passage, an illegal contract is void rather than voidable at the option of either
party, regardless of Mr. P's lack of knowledge about the drug's illegal status.

Option b) is incorrect because the passage states that a contract with an illegal purpose is void, even if the parties had
consented and provided consideration.

161. Answer: C (Reference Lines: "The assets should be divided in a clear manner stating who will inherit what...")

Explanation:

Option A is incorrect because the passage states that the Indian Succession Act, 1925 does not prescribe any particular
format or technical requirements for Wills, which means that a non-standard format and unclear language do not
automatically render the Will invalid.

Option B is incorrect because, despite the Indian Succession Act, 1925 not prescribing any particular format or
technical requirements for Wills, the passage emphasizes that the assets should be divided in a clear manner. The
presence of ambiguities in the Will could undermine its validity.

Option C is correct because the passage highlights the importance of dividing the assets in a clear manner. The
ambiguities in the Will may render it invalid, regardless of the Indian Succession Act, 1925's lack of specific format
and technical requirements.

Option D is incorrect because, although the Indian Succession Act, 1925 requires the Will to be signed by two
witnesses, the main issue in the question is the presence of ambiguities, which could potentially render the Will
invalid.

162. Answer: B (Reference Lines: "Indian Succession Act, 1925 governing Wills has not prescribed any particular
format or technical requirement.")

Explanation:

Legal Reasoning Practice Sheet 114


CLAT POINT

Option A is incorrect because the passage states that the Indian Succession Act, 1925 does not prescribe any particular
format or technical requirements for Wills, which means that a non-standard format does not automatically render
the Will invalid.

Option B is correct because, as mentioned in the passage, the Indian Succession Act, 1925 does not prescribe any
particular format or technical requirements for Wills. The non-standard format of the Will, despite having clear and
unambiguous language, should not invalidate it according to the Act.

Option C is incorrect because the passage emphasizes that the Indian Succession Act, 1925 does not prescribe any
particular format or technical requirements for Wills. The non-standard format, in the absence of any ambiguities,
should not render the Will invalid.

Option D is incorrect because, although the Indian Succession Act, 1925 requires the Will to be signed by two
witnesses, the main issue in the question is the non-standard format, not the signing of the Will. The passage states
that the Act does not prescribe any particular format or technical requirements, so the format should not be a valid
reason to invalidate the Will.

163. Answer: B (Reference Lines: "Section 31 to 49 of the Indian Succession Act states that in case, a man leaves
behind his wife and children, then his widow will get one-third of his property and the remaining two-thirds will go
to his direct lineage.")

Explanation:

Option A is incorrect because the passage does not mention that the Indian Succession Act mandates a division of
property that includes distant relatives when a person dies intestate.

Option B is correct because, according to the Indian Succession Act, in cases of intestate succession, the widow is
entitled to one-third of the property, while the remaining two-thirds are allocated to the direct lineage. Since Mr. Z
is a distant relative, he is not part of the direct lineage and is not entitled to a share of the property.

Option C is incorrect because the passage does not state that the Indian Succession Act requires distant relatives to
receive a portion of the property in cases of intestate succession.

Option D is incorrect because, although it mentions the correct division of property prescribed by the Indian
Succession Act, it does not address the issue of Mr. Z's claim as a distant relative.

164. Answer: A (Reference Lines: "If there are no children, then the wife gets half of the property, and the other half
is given to the kindred.")

Explanation:

Option A is correct because the passage clearly states that, according to the Christian law of inheritance and
succession, when there are no children, the wife is entitled to half of the property, and the other half is distributed
among the kindred. This supports Mrs. Y's claim to half of Mr. X's property.

Option B is incorrect because the passage does not mention that the Christian law of inheritance and succession
requires equal division of property among the surviving spouse and all relatives, including distant relatives, in cases
of intestate succession. Instead, it emphasizes the wife's entitlement to half of the property and distribution of the
other half among the kindred.

Option C is incorrect because it inaccurately represents the allocation of the other half of the property. The passage
states that the other half is given to the kindred, which includes more than just the siblings of the deceased.

Legal Reasoning Practice Sheet 115


CLAT POINT

Option D is incorrect because it contradicts the passage, which establishes the wife's right to half of the property and
the distribution of the other half among the kindred. The passage does not mention equal distribution among the
surviving spouse, siblings, and distant relatives.

165. Answer: D (Reference Lines: "In the absence of the widow, the property will be equally distributed among the
children. Children of children (grandchildren) will get a share if their parents are no more.")

Explanation:

Option A is incorrect because the passage states that grandchildren will only get a share if their parents are no more,
and in this case, Mr. X is alive.

Option B is incorrect because the passage does not mention that grandchildren have the right to claim a portion of
their parent's share when the parent is still alive.

Option C is incorrect because it incorrectly assumes that the grandchildren have no right to claim any share. The
passage states that they can claim a share if their parent is deceased, but in this case, their parent, Mr. X, is alive.

Hence, option D is the right choice here.

166. Answer: Option B. (Reference lines:"...it is to be determined not according to the claimant's perception but in
the larger public interest wherein other more important considerations may outweigh what would otherwise have
been the legitimate expectation of the claimant.")

Explanation:

A. This option is incorrect because the passage emphasizes that larger public interest may outweigh an individual's
legitimate expectation.

B. This option is correct, as the passage suggests that if the new policy serves the larger public interest, it may
outweigh X's legitimate expectation, even if it affects them negatively.

C. This option is incorrect because it focuses on the new policy's consideration of unique circumstances, which is not
the main point in the passage. The passage is about the larger public interest outweighing individual legitimate
expectations.

D. This option is incorrect, as it emphasizes the bona fide nature of the decision but does not directly address the
larger public interest. Although a bona fide decision is necessary, the passage focuses on the larger public interest as
the determining factor in such situations.

167. Answer: Option B. (Reference lines:"...it is to be determined not according to the claimant's perception but in
the larger public interest wherein other more important considerations may outweigh what would otherwise have
been the legitimate expectation of the claimant.")

Explanation:

A. This option is incorrect because the passage emphasizes that larger public interest may outweigh an individual's
legitimate expectation.

B. This option is correct, as the passage suggests that if the new policy serves the larger public interest, it may
outweigh X's legitimate expectation, even if it affects them negatively.

Legal Reasoning Practice Sheet 116


CLAT POINT

C. This option is incorrect because it focuses on the new policy's consideration of potential benefits, which is not
the main point in the passage. The passage is about the larger public interest outweighing individual legitimate
expectations.

D. This option is incorrect, as it is very similar to option B but emphasizes the potential benefits of constructing a
shopping mall. The passage focuses on the larger public interest as the determining factor in such situations, rather
than the specific benefits of a particular project.

168. Answer: Option B. (Reference lines: "Not being a right, it is not enforceable as such.")

Explanation:

A. This option is incorrect because, although A may have a legitimate expectation based on the established practice,
the passage states that legitimate expectation is not enforceable as a legal right.

B. This option is correct, as the passage indicates that legitimate expectation is not enforceable as a legal right, even
if the expectation is based on an established practice.

C. This option is incorrect because it focuses on the reasonableness and logic behind the discontinuation of the
program, which is not the main point in the passage. The passage is about the enforceability of legitimate
expectations, which is not applicable in this case.

D. This option is incorrect, as it incorrectly states that legitimate expectation only applies to promises made by an
authority. The passage mentions that legitimate expectation can also arise from established practices, but it is not
enforceable as a legal right.

169. Answer: Option A. (Reference lines: "It is procedural in character based on the requirement of a higher degree
of fairness in administrative action, as a consequence of the promise made, or practice established.")

Explanation:

A. This option is correct because, although legitimate expectation is not enforceable as a legal right, the passage
states that it is a concept fashioned by courts for judicial review of administrative action. The abrupt
discontinuation of the program without prior notice is procedurally unfair, and A may succeed in their claim based
on the requirement of a higher degree of fairness in administrative action.

B. This option is incorrect because, while legitimate expectation is not enforceable as a legal right, it may still be
considered in judicial review of administrative action, particularly when procedural fairness is in question.

C. This option is incorrect because it focuses on the reasonableness and logic behind the discontinuation of the
program, which is not the main point in the passage. The passage is about the procedural fairness in administrative
action arising from legitimate expectation.

D. This option is incorrect, as it incorrectly states that legitimate expectation only applies to promises made by an
authority. The passage mentions that legitimate expectation can also arise from established practices, and
procedural fairness in administrative action is required.

170. Answer: Option B. Reference lines: "The doctrine of legitimate expectation, based on established practice, can
be invoked only by someone who has dealings or transactions or negotiations with an authority, on which such
established practice has a bearing, or by someone who has a recognized legal relationship with the authority.")

Explanation:
Legal Reasoning Practice Sheet 117
CLAT POINT

A. This option is incorrect because the passage states that the doctrine of legitimate expectation can only be invoked
by someone who has had dealings, transactions, or negotiations with the authority or a recognized legal relationship.
A has not yet started their business and does not meet these requirements.

B. This option is correct because A has not yet started their business and, therefore, has not had any dealings,
transactions, or negotiations with the local government. As per the passage, A cannot invoke the doctrine of
legitimate expectation in this situation.

C. This option is incorrect because it focuses on the consideration of relevant factors relating to legitimate
expectation, while the main point in the passage is about the requirements for invoking the doctrine of legitimate
expectation. In this case, A does not meet the requirements stated in the passage.

D. This option is incorrect because it incorrectly assumes that the local government has unrestricted authority to
change policies. The passage emphasizes the doctrine of legitimate expectation, but in this case, A does not meet the
requirements to invoke the doctrine.

171. Answer- option b.

In the given passage, the Delhi High Court relied upon the judgment of the Apex Court in Pune Municipal Corporation
& Anr. v. Misirimal Solanki & Ors. (2014) 3 SCC 183. Hence, option (b) is the correct answer.

Option (a) is incorrect because the judgment of Indore Development Authority v. Manoharlal and Others was referred
to by the division bench of Justice Abhay S. Oka and Justice Rajesh Bindal of the Supreme Court while setting aside
the order of the Delhi High Court.

Option (c) is also incorrect because Delhi Development Authority v. Land Acquisition Collector is not mentioned in
the passage and is not relevant to the given facts.

Option (d) is incorrect as well because the High Court did rely on the judgment of Pune Municipal Corporation & Anr.
v. Misirimal Solanki & Ors. while making its decision.

172. Answer- D

Option a is wrong as it is mentioned in the passage that “It was further submitted that the Constitution Bench
in Indore Development Authority (supra) opined that the compliance of either of the two conditions i.e., taking over
of possession of the land or payment of compensation, is good enough to sustain the acquisition.”

Option b is right as the payment was already done. Hence fulfilling one of the two requirements listed down by SC in
the indore development authority case.

Option c is right as the same has been mentioned in the passage.

Therefore, the correct option is d.

173. Answer- option c

The given scenario describes a situation where the state government had contracted with the owners of a land to
purchase it, with a promise to pay the price in installments over six months, while also taking its possession. However,
the government stopped making payments after the first three months, leading the owners to reclaim the land. As a
result, the government registered a case against them for the offence of trespass.

Legal Reasoning Practice Sheet 118


CLAT POINT

Option A suggests that the owners are right in reclaiming the land as they have not received payment, and the
contract stands null and void. However, this option is incorrect as it fails to take into account the legal provisions
governing land acquisition.

Option B suggests that the government is right in registering the case against the owners for trespass. However, this
option is also correct as the government had the right to sue them for trespass. But between options b and c, option
c looks more relevant and connected to the facts therefore c is the correct option.

Option C correctly suggests that the land belongs to the government as one of the essential conditions to sustain the
acquisition has been fulfilled, i.e., possession of the land has been taken. As mentioned earlier, the Supreme Court in
Indore Development Authority v. Manoharlal and Others has held that compliance with either of the two conditions,
i.e., taking possession of the land or payment of compensation, is sufficient to sustain the acquisition. Therefore, the
fact that the government had taken possession of the land, even though payments were stopped, means that the
land belongs to the government, and the owners can be charged with trespass.

Option D, which is "cannot determine," is incorrect as the given scenario provides sufficient information to arrive at
a decision.

174. Answer- option c

In the given scenario, the government had contracted the owners to purchase their land and promised to pay the
price in installments while not taking possession of the land. However, after paying the first three installments on
time, the government failed to make any further payments for one year. As a result, the landowners decided to
reclaim their land by settling on it.

If the government had taken possession of the land and only partly fulfilled the payment obligation, the situation
would be different. In this case, the landowners would still be the rightful owners of the land as the government has
not fulfilled one of the two conditions laid down in the Indore Development Authority v. Manoharlal and Others
(2020) judgment - taking over of possession of the land or payment of compensation.

Therefore, option c is the correct answer - the government cannot register the case for trespass against the
landowners.

175. Answer- option b

The question refers to the verdict of the Supreme Court in a case where an order of the Delhi High Court was set
aside. The High Court had held that the acquisition of land was lapsed as compensation was not paid to the
landowners. The Supreme Court, while relying on its earlier judgment in Indore Development Authority v. Manoharlal
and Others, observed that one of the conditions laid down in that case, i.e., taking over of possession of the land, has
been satisfied.

176. Answer- option d.

The passage discusses a ruling by the Delhi High Court regarding the quashing of an FIR for rape on the basis of
settlement and subsequent marriage between the accused and the victim. The court ruled that the settlement and
marriage do not waive the offence and the FIR cannot be quashed under Section 482 of the CrPC.

Option A states that the high court was correct in quashing the FIR. This is incorrect because the passage explicitly
states that the high court cannot quash an FIR for rape on the basis of settlement and marriage.

Legal Reasoning Practice Sheet 119


CLAT POINT

Option B states that the offence is compoundable and can only be quashed by the Supreme Court. However, the
passage does not mention anything about the compoundability of the offence, and the court's ruling was based on
the seriousness of the offence of rape. Also, the power of supreme court is also not discussed in the passage.

Option C states that the high court was correct in quashing the FIR because rape is a serious offence. This statement
is opposite of what is discussed in the passage because the high court cannot quash the FIR for rape on the basis of
settlement and marriage, citing the seriousness of the offence.

Option D is the correct answer because the passage does not provide enough information to support any of the given
options. The passage only discusses the ruling of the Delhi High Court regarding the quashing of an FIR for rape based
on a settlement and subsequent marriage between the accused and the victim.

177. Answer- option d

In this scenario, Tola was charged with attempt to murder after he pushed Mola and caused serious injuries. The case
has reached the high court, and the question is whether the FIR can be quashed if both parties have come to a
compromise.

Option B and C are both correct as they state that the FIR cannot be quashed as this is a serious offence and a crime
against society. Attempt to murder is a serious crime and it affects not only the individual but also society as a whole.
Therefore, the court cannot simply quash the FIR based on a compromise between the parties.

Option A is incorrect as it is not relevant to this scenario. The fact that this is not a case of rape is not relevant to
whether the FIR can be quashed.

Option D is correct as it includes both correct options B and C.

178. Answer- option c

The case involves Mr. Vyas, an IAS officer, who is charged with outraging the modesty of his secretary, Ms. Sheila, by
allegedly touching her improperly while being in an inebriated state. Mr. Vyas has offered an apology and
compensation to Ms. Sheila and asked the court to quash the FIR, citing the dignity of his post. The FIR cannot be
quashed just because the accused is a government officer. A government servant has all the more responsibility as
far as his conduct is concerned towards the society as per the passage. Hence, option c is correct.

Option A suggests that the FIR will be quashed, but this may not be the case as the offence of outraging the modesty
of a woman is a serious crime and cannot be simply waived off on the basis of a settlement.

Option B suggests that the high court may reserve its order and announce it some other day after discussion.
However, the passage does not provide any information on whether this is a possibility.

Option C is the correct answer as the FIR will not be quashed in this case. The offence of outraging the modesty of a
woman is considered a serious offence, and the court will not simply quash the FIR based on an apology and
compensation offer.

Option D suggests that the FIR will be quashed if Ms. Sheila consents to it. However, in cases of serious offences like
outraging the modesty of a woman, the consent of the victim is not enough to quash the FIR. The court will consider
various factors such as the gravity of the offence and the impact on society before making a decision.

179. Answer- option d

All of the above are considered as grave offence. Hence, option d is right.
Legal Reasoning Practice Sheet 120
CLAT POINT

180. Answer -option d.

a) The application for quashing her case can be accepted by the Court because the parties have come into a
compromise.

This option suggests that since both parties have come into a compromise, the Court can accept the application for
quashing her case. However, the fact that the case involves an allegation of rape means that it is a serious offence,
and the mere fact that the parties have entered into a compromise cannot be a ground for quashing the case.
Therefore, this option is not correct.

b) The application for quashing her case can be accepted by the Court because the case of rape filed by one of the
parties was frivolous in nature.

This option suggests that the case of rape filed by Ramani was frivolous in nature and hence, the Court can accept
the application for quashing the case. However, there is no information in the passage to suggest that the case was
frivolous in nature. Therefore, this option is not correct.

c) The application for quashing her case cannot be accepted by the Court because the alleged allegations on the
person was not of private nature.

This option suggests that the application cannot be accepted because the allegations were not of a private nature.
However, the passage does not provide any information on the nature of the allegations. Therefore, this option is not
correct.

d) The application for quashing her case cannot be accepted by the Court because rape is a serious offence.

This option is correct because, as mentioned earlier, the act of rape is a serious offence, and the mere fact that the
parties have entered into a compromise cannot be a ground for quashing the case. The Court has a responsibility to
ensure that justice is served, and serious offences such as rape cannot be treated in a casual manner. Therefore, the
application for quashing her case cannot be accepted by the Court.

181. Ans- c

In this scenario, Sappolo Hospital lawfully removed Shrikant's appendix as part of his medical treatment, and Shrikant
benefitted from the removal ("Shrikant became alright"). Although the hospital later developed valuable drugs using
his cells, they did not deprive Shrikant of anything, as the appendix was of no use to him after removal, as mentioned
in the passage, "an act that is constituted by taking the goods but without any intention of exercising permanent or
temporary dominion can be termed as trespass but not as conversion." Therefore, Sappolo Hospital is not obligated
to share its profits with Shrikant, making option c the correct choice. Options a, b, and d are incorrect as they imply a
legal or moral obligation to share profits or establish a trust, which is not supported by the passage.

182. Ans- c

Andrew will succeed in his claim, as Tobey's actions constituted conversion due to the "wilful interference, without
any lawful justification, in a manner which is inconsistent with the right of another," as mentioned in the passage.
However, Tobey is entitled to be reimbursed for the expenses incurred in repairing the car, as the improvements
contributed to its value on the day of judgment. Options a, b, and d are incorrect because they either misinterpret
the legal principles mentioned in the passage, such as the requirements for conversion, or fail to properly address
the balance between Andrew's rights and Tobey's contributions to the car's increased value.

Legal Reasoning Practice Sheet 121


CLAT POINT

183. Ans- b

At the time of the sale to Natasha, Thor was not an agent of Thanos to deal with the ring and was not in the position
of a person who might be presumed as an agent having authority to sell it, and that, by the sale he converted the ring
to his own use; and, therefore liable to Thanos. Option a. is not correct as although it was his work but after the end
of stipulated time, he lost his position of agent in the respective case. Similarly, in option c. the burden should be on
Thor and not Natasha because such contracts are not foreseen. In option d The ring was wrongful transfer since the
first transaction itself and hence all other transactions after that are void ab initio.

184. Ans- c

Both Jane and George are liable for conversion, as their actions involved dealing with the painting in a manner
inconsistent with Robert's rights, even though they honestly believed they were entitled to do so. The passage states
that "such dealings will be considered as intentional and will amount to conversion even if he did not know of the
right held by the plaintiff and honestly believed that he was entitled to do so." Options a, b, and d are incorrect, as
they either fail to address the legal principles of conversion mentioned in the passage or misinterpret the roles and
responsibilities of the involved parties in relation to the plaintiff's rights.

185. Answer: B) The auction house is liable for conversion, as they dealt with the painting in a manner inconsistent
with John's rights.

Explanation:

A) Incorrect. The passage states, "an auctioneer is held liable for conversion even if he honestly believed that the
goods which are being auctioned belong to the seller and not to the plaintiff."

B) Correct. The auction house dealt with the painting in a manner inconsistent with John's rights, as they sold the
painting without his consent or knowledge. As stated in the passage, "such dealings will be considered as intentional
and will amount to conversion even if he did not know of the right held by the plaintiff and honestly believed that he
was entitled to do so."

C) Incorrect. Trespass involves taking goods without the intention of exercising permanent or temporary dominion
over them. In this scenario, the auction house exercised dominion by selling the painting, which points to conversion
rather than trespass.

D) Incorrect. The auction house's liability is based on their dealing with the painting in a manner inconsistent with
John's rights, not on the lawfulness of Mary's initial possession of the painting.

186. Answer: D) Both A and B

Explanation:

A) Correct. As per the passage, "if either party to the marriage is: …(c) has been subject to recurrent attacks of insanity,
then that marriage shall be voidable and may be annulled by a decree of nullity." Mrs. X's mental instability and
recurring epileptic attacks are considered recurrent attacks of insanity, making the marriage voidable under section
5(iii). B) Correct. Mrs. X's epilepsy and the resulting behavioral issues create a situation in which Mr. Y's marriage can
be annulled under section 5(iii) of the Hindu Marriage Act. C) Incorrect. The passage clearly states that a marriage
can be voidable and annulled due to certain mental health issues, including recurrent attacks of insanity, as in Mrs.
X's case. D) Correct. Both A and B are correct, as Mrs. X's condition makes the marriage voidable under section 5(iii)
of the Hindu Marriage Act, and the annulment is also justified by her epilepsy and its consequences on their marital
relationship.
Legal Reasoning Practice Sheet 122
CLAT POINT

187. Answer: C) Voidable, since Mr. A's condition falls under the provisions of Section 5(iii), allowing for annulment if
pursued.

Explanation:

A) Incorrect. Although Mr. A's condition developed after their marriage, it still has an impact on their marital status
under Section 5(iii) of the Hindu Marriage Act, which allows for annulment due to certain mental health conditions.

B) Incorrect. The marriage is not void, as it was valid at the time of the wedding. The situation allows for annulment,
making the marriage voidable, not void.

C) Correct. As per the passage, "if either party to the marriage is: … (c) has been subject to recurrent attacks of
insanity, then that marriage shall be voidable and may be annulled by a decree of nullity." Since Mr. A's schizophrenia
and seizures are considered recurrent attacks of insanity, the marriage can be considered voidable.

D) Incorrect. The information provided is sufficient to determine that their marriage status is voidable under Section
5(iii) of the Hindu Marriage Act.

188. Answer: B) No, since Dr. R's diagnosis confirmed that Mr. P was not suffering from any mental health conditions
that would warrant annulment under Section 5(iii) of the Hindu Marriage Act.

Explanation:

A) Incorrect. As mentioned in the passage, mental disorders that warrant annulment under Section 5(iii) must be of
"such a kind or to such an extent as to be unfit for marriage and the procreation of children" or involve "recurrent
attacks of insanity." Neither condition is met in Mr. P's situation.

B) Correct. The passage states that a marriage may be annulled if one party has a mental disorder or suffers from
recurrent attacks of insanity or epilepsy. Since Dr. R's diagnosis confirmed Mr. P does not have any mental health
issues, there is no ground for annulment under Section 5(iii) of the Hindu Marriage Act.

C) Incorrect. The facts provided are sufficient to conclude that Mr. P does not have any mental health issues that
would warrant annulment under Section 5(iii) of the Hindu Marriage Act.

D) Incorrect. Mr. P's depression issues, as confirmed by Dr. R, are not a qualifying mental disorder under the Hindu
Marriage Act.

189. Answer: D) Amit can seek annulment of the marriage, as Priya has been subject to recurrent attacks of epilepsy
and the marriage is voidable under the Hindu Marriage Act, 1955.

Explanation:

A) Incorrect. Though epilepsy is mentioned as a ground for annulment in the Hindu Marriage Act, 1955, the phrasing
of the Act has changed. The correct reason for annulment is stated in option D.

B) Incorrect. Divorce would be applicable if the mental disorder substantially impaired their conjugal harmony and
developed after their marriage. In this scenario, the epilepsy was pre-existing and recurrent.

C) Incorrect. Priya's recurrent epilepsy does provide grounds for legal relief under the Hindu Marriage Act, 1955.

D) Correct. The passage mentions that under s. 5(ii) of the Hindu Marriage Act, 1955, a marriage may be annulled if
a party "has been subject to recurrent attacks of insanity or epilepsy." In this scenario, Priya's recurrent epilepsy
makes the marriage voidable, and Amit can seek annulment.
Legal Reasoning Practice Sheet 123
CLAT POINT

190. Answer: C) Sara can seek divorce, as Rajesh's mental disorder developed after their marriage and substantially
impairs their conjugal harmony.

Explanation:

A) Incorrect. The passage states that mental abnormality that intervenes after marriage can be a ground for divorce.

B) Incorrect. Annulment is applicable when mental disorder exists at the time of marriage and affects the party's
ability to give valid consent or make them unfit for marriage and procreation (s. 5(ii) of the Hindu Marriage Act, 1955).
In this scenario, Rajesh's mental disorder developed after their marriage.

C) Correct. The passage mentions that "conjugal harmony and happiness would be substantially impaired if either of
the parties suffers from such abnormality which has intervened after marriage. Such abnormality could be a ground
for divorce as distinct from annulment."

D) Incorrect. Annulment would only be applicable if the condition existed at the time of the marriage and affected
Rajesh's ability to give valid consent or made him unfit for marriage and procreation.

191. Ans- d.

Explanation:

Defamation is the act of making untrue statements about another which damages his/her reputation. In this case,
the suit will not succeed because for defamation to arise, the ill-intending statement must be made to third parties,
or third parties must be informed of such. While George did share the letter with friends and family, this may not be
considered as publishing the defamatory statement to "right-thinking members of society," as stated in the passage.
Thus, option d is the correct answer. Options a, b, and c are incorrect because they do not align with the principles of
defamation provided in the passage. Option a is incorrect because the knowledge or expertise of James is not the
deciding factor for a defamation case. The focus should be on whether the defamatory statement was made to third
parties, as per the passage.

Option b is incorrect as it does not take into account the legal principles of defamation mentioned in the passage.
The frequency of such incidents or their trivial nature does not negate the possibility of defamation, and the court
will consider a case based on the legal merits, not solely on its frequency. Option c is incorrect because personal
grudges alone are not enough to prove defamation. The deciding factor should be whether the defamatory statement
was made to third parties, which, as per the passage, is essential for a defamation case. Option d is the correct answer
because the defamatory statement must be made to third parties or communicated to a wider audience. In this case,
George only shared the letter with friends and family, which may not be considered as publishing the defamatory
statement to "right-thinking members of society" as required by the legal principles mentioned in the passage.

192. Ans- c.

Explanation:

Option a is incorrect because the question does not provide any evidence or facts to support the claim that Vijay is
indeed a liar and a thief. The focus should be on whether the defamation occurred and if Mr. Lion is responsible for
Ronny's actions.

Option b is incorrect as the statement itself does not automatically make Mr. Lion responsible for the defamation,
even though Ronny defamed Vijay in a public place. It is necessary to determine whether Ronny was acting within
the scope of his employment.

Legal Reasoning Practice Sheet 124


CLAT POINT

Option c is the correct answer because, although Ronny defamed Vijay, he was not within the scope of his
employment while drinking coffee. Mr. Lion cannot be held liable for Ronny's actions outside of the course of his
employment.

Option d is incorrect because the defamation was not made in private, as there were other people in the coffee shop
who heard Ronny's statements about Vijay.

193. Ans- c.

Explanation:

Option a is incorrect because, while the statement was made in public, it is not defamatory as it was not meant to
harm Katrina's reputation as a reasonable person would not interpret the statement as an accusation of theft. Option
b is incorrect because there is no evidence to suggest that Vicky's intentions were malicious. The statement appears
to be an expression of admiration rather than an attempt to defame or humiliate Katrina. Option c is the correct
answer because the statement is not defamatory in nature. Instead, it is an expression of Vicky's admiration and
affection for Katrina, which would not harm her reputation in the eyes of right-thinking members of society. Option
d is incorrect because emotional distress alone is not sufficient to establish a defamation claim. Defamation requires
that the statement be false and injurious to the reputation of the person being defamed. In this case, the statement
was an expression of admiration and not likely to be considered damaging to Katrina's reputation.

194. Ans- a

Explanation: Option a is correct because, as mentioned in the passage, defamation occurs when there is "any
intentional false communication, either written or spoken, that harms a person's reputation." In this case, Karan's
claim was false and it caused damage to Priya's reputation.

Option b is incorrect because the statement made by Karan was not true in substance, as it was revealed that the
similarities were purely coincidental, and no plagiarism had taken place.

Option c is incorrect because, although Karan's statement was published in a well-respected news outlet and could
be considered libel, this fact alone is not sufficient for Priya to succeed in her defamation lawsuit. The statement must
be false and cause harm to her reputation, which is already covered by option a. Option d is incorrect because, as
mentioned in the passage, defamation involves "any intentional false communication, either written or spoken , that
harms a person's reputation." In this case, even though Karan might have made the statement in good faith and
without malicious intent, the fact remains that it was false and caused harm to Priya's reputation.

195. Ans- b

Explanation:

Option b is correct because the article was based on a fictional character, and as mentioned in the passage,
defamation involves "any intentional false communication, either written or spoken , that harms a person's
reputation." Since the character was fictional and there are many individuals named Ravinder, apart from him,
Ravinder Ahuja cannot prove that the story was about him specifically and intended to harm his reputation. Option a
is incorrect because even if Shanti Lal clarified that the character was fictional, it would not change the fact that the
story was not based on Ravinder Ahuja and therefore, would not be defamatory. Option c is incorrect because,
although the full name was not mentioned, this is not sufficient to dismiss the suit. The critical aspect is that the story
was based on a fictional character, and Ravinder Ahuja cannot prove that the story was about him specifically and
intended to harm his reputation. This reasoning is already covered in option b. Option d is incorrect because there is

Legal Reasoning Practice Sheet 125


CLAT POINT

no legal requirement for Shanti Lal to seek permission from all individuals named Ravinder before publishing a
fictional story. The key issue is whether the story defamed Ravinder Ahuja by making false statements about him,
which is not the case here.

196. Answer: b

Explanation:

As mentioned in the passage, "consent is said to be caused by coercion when it is obtained by pressure exerted by
either of the following techniques: (1) committing or threatening to commit any act forbidden by the Indian Penal
Code." Committing suicide is forbidden under Section 309 of the Indian Penal Code, which means threatening to
commit suicide falls under coercion as per the Indian Contract Law. Hence, option b) is the correct answer. Option a)
is incorrect because, according to the passage, coercion involves committing or threatening to commit any act
forbidden by the Indian Penal Code, which includes suicide as per Section 309. Emotional distress is irrelevant in
determining coercion in this case. Option c) is incorrect because the threat of suicide is directly related to an unlawful
act, as committing suicide is forbidden by Section 309 of the Indian Penal Code. Option d) is incorrect because, as per
the passage, coercion involves committing or threatening to commit any act forbidden by the Indian Penal Code.
Awareness of the legal implications is not a requirement for coercion as per the Indian Contract Law.

197. Answer: a

Explanation:

According to the passage, "when a criminal prosecution is instituted against a person and such person fearing the
result of the prosecution enters into an agreement in favour of the complainant in consideration of his abandoning
the prosecution, it cannot be held simply upon these facts that the consent of such person was caused by coercion."
Since Rajesh had actually misrepresented his age, the threat of prosecution is based on a genuine criminal act, and
thus it cannot be considered coercion under Indian Contract Law. Option a) is correct. Option b) is incorrect because,
as stated in the passage, threatening a criminal prosecution is not per se an act forbidden by the Indian Penal Code
and does not amount to coercion in every case. Option c) is incorrect because it implies that the threat was based on
a truthful misrepresentation, but the reasoning should be that it is based on a genuine criminal act, as specified in
option a). Option d) is incorrect because, as per the passage, awareness of the legal implications of a threat is not a
requirement for coercion under Indian Contract Law.

198. Answer: a

Explanation:

According to the passage, coercion is defined as "a committing or threatening to commit some act which is contrary
to law." Since criminal intimidation is punishable under Section 506 of the Indian Penal Code, A's conduct amounts
to coercion under Indian Contract Law. Hence, option a) is the correct answer.

Option b) is incorrect because the location where the intimidation occurred, either on land or on a ship, is irrelevant
to the determination of whether the conduct constitutes coercion under Indian Contract Law.

Option c) is incorrect because there is no exemption for property transactions under Indian Contract Law regarding
coercion claims.

Option d) is incorrect because B's awareness of the consequences of entering an agreement under intimidation is not
a determining factor in establishing coercion under Indian Contract Law.

Legal Reasoning Practice Sheet 126


CLAT POINT

199. Answer: a

Explanation:

Option a) is correct because, according to the passage, "consent is said to be caused by coercion when it is obtained
by pressure exerted by either of the following techniques; (1) committing or threatening to commit any act forbidden
by the Indian Penal Code." In this case, the threat of false criminal charges, which is an act forbidden by the Indian
Penal Code, coerced Arvind into signing the contract, making the contract voidable due to coercion.

Option b) is incorrect because the passage focuses on the act of committing or threatening to commit an act forbidden
by the Indian Penal Code as the source of coercion. In this scenario, the coercion arises from the threat of false
criminal charges, not the donation to the charity.

Option c) is incorrect because the passage doesn't require Arvind to prove Ram's intent to file false charges and
fabricate evidence. The fact that the contract was signed under the threat of false criminal charges, which is an act
forbidden by the Indian Penal Code, is enough to establish coercion.

Option d) is incorrect because the passage explains that "where consent to an agreement is caused by coercion, the
agreement is a contract voidable at the option of the party whose consent was so caused." In this case, Arvind's
consent was caused by coercion due to the threat of false criminal charges, making the contract voidable at his option,
regardless of whether he believed the charges to be genuine at the time.

200. Answer: a

Explanation:

Option a) is correct because, according to the passage, "consent is said to be caused by coercion when it is obtained
by pressure exerted by either of the following techniques; (1) committing or threatening to commit any act forbidden
by the Indian Penal Code." Threatening to file a false charge is an act forbidden by the Indian Penal Code, and Kumar
used this threat to force Rajesh to sign the contract, making the contract voidable due to coercion.

Option b) is incorrect because the passage states, "such an act could only be one forbidden by the Indian Penal Code
if it amounted to a threat to file a false charge." In this scenario, Kumar threatened to file a false criminal charge
against Rajesh, which is an act forbidden by the Indian Penal Code.

Option c) is incorrect because the passage doesn't require Rajesh to prove Kumar's intent to file a false charge. The
fact that the contract was signed under the threat of a false criminal charge is enough to establish coercion.

Option d) is incorrect because the passage explains that "where consent to an agreement is caused by coercion, the
agreement is a contract voidable at the option of the party whose consent was so caused." In this case, Rajesh's
consent was caused by coercion, making the contract voidable at his option.

Legal Reasoning Practice Sheet 127


About Us:
CLAT POINT is a law entrance examination
preparation institute. We do not confide our services
only in the name of a coaching institute but our aim is
to help the youth who join us to achieve a momentum
that can help them embark on a journey in the field of
law and legal knowledge.
CLAT POINT is an initiative and the culmination of
efforts put in by it’s founder Manvendra Pratap Singh
who has an experience in teaching in fields as vast as
UPSC and CLAT. He is an NLU Dropout and Alumini
from Jamia Millia Islamia University Delhi .
With an online viewership of not less than a million
he is a prominent face in the teaching industry.
CLAT POINT aims at providing efficient services in
the examinations of CLAT UG , CLAT PG, NLSAT,
JUDICIARY, LAW OPTIONAL and other Law
Entrance Tests

or visit www.clatpoint.com 7011249333

You might also like